├── main.pdf ├── main - 2A.pdf ├── main - 2B.pdf ├── README.md ├── preface.tex ├── main.tex ├── main - 2A.tex ├── main - 2B.tex ├── 5.tex ├── 2.tex └── 6.tex /main.pdf: -------------------------------------------------------------------------------- https://raw.githubusercontent.com/jamesfang8499/math2/HEAD/main.pdf -------------------------------------------------------------------------------- /main - 2A.pdf: -------------------------------------------------------------------------------- https://raw.githubusercontent.com/jamesfang8499/math2/HEAD/main - 2A.pdf -------------------------------------------------------------------------------- /main - 2B.pdf: -------------------------------------------------------------------------------- https://raw.githubusercontent.com/jamesfang8499/math2/HEAD/main - 2B.pdf -------------------------------------------------------------------------------- /README.md: -------------------------------------------------------------------------------- 1 | # 中学数学实验教材 第二册(上、下册) 2 | 3 | 本项目对1980年代初出版发行的《中学数学实验教材 第二册》进行了重排。 4 | 5 | 这一套中学数学实验教材是教育部委托北京师范大学、中国科学院数学研究所、人民教育出版社、北京师范学院、北京景山学校等单位组成的领导小组组织“中学数学实验教材编写组”,根据美国加州大学伯克利分校数学系项武义教授的《关于中学实验数学教材的设想》编写的。 6 | 7 | 本重排本,将原教材的上下两册合排成一本,当中涉及的部分内容从现在看来已经超纲或过时,请阅读时仔细甄别。 8 | 9 | 书中的矢量图片使用Tikz和Tkz-euclide制作。其余的点阵图则是来自于扫描版电子文档(限于作者的能力和精力,无法将所有内容均以矢量图全部重绘)。 10 | 11 | **main - 2A.pdf和main - 2B.pdf分别是经过拆分后的上、下两册。** 12 | 13 | **注意:本项目的内容勿用于商业目的。** 14 | 15 | 16 | # 目录 17 | 18 | ## 第1章 实验几何 19 | * 点、直线和平面 20 | * 方向、角度与平行 21 | * 全等和叠合 22 | * 相似和相似比 23 | * 基本作图 24 | 25 | ## 第2章 集合与简易逻辑 26 | * 集合 27 | * 简易逻辑 28 | 29 | ## 第3章 直线形 30 | * 三角形 31 | * 平行线与内角和定理 32 | * 特殊四边形 33 | * 面积与勾股定理 34 | * 相似形 35 | 36 | ## 第4章 圆 37 | * 圆的基本性质 38 | * 圆与直线的位置关系 39 | * 与圆有关的角 40 | * 圆与正多边形 41 | 42 | ## 第5章 轨迹与作图 43 | * 轨迹 44 | * 作图 45 | 46 | ## 第6章 三角比与角边关系 47 | * 锐角三角比 48 | * 解直角三角形 49 | * 任意三角形中的边角关系 50 | 51 | 52 | -------------------------------------------------------------------------------- /preface.tex: -------------------------------------------------------------------------------- 1 | \chapter{前~~言} 2 | 3 | 这一套中学数学实验教材,内容的选取原则是精简实 4 | 用,教材的处理力求深入浅出,顺理成章,尽量作到使人人 5 | 能懂,到处有用. 6 | 7 | 本教材适用于重点中学,侧重在满足学生将来从事理工 8 | 方面学习和工作的需要. 9 | 10 | 本教材的教学目的是:使学生切实学好从事现代生产、 11 | 特别是学习现代科学技术所必需的数学基础知识;通过对数 12 | 学理论、应用、思想和方法的学习,培养学生运算能力,思 13 | 维能力,空间想象力,从而逐步培养运用数学的思想和方法 14 | 去分析和解决实际问题的能力;通过数学的教学和学习,培 15 | 养学生良好的学习习惯,严谨的治学态度和科学的思想方 16 | 法,逐步形成辩证唯物主义世界观. 17 | 18 | 根据上述教学目的,本教材精选了传统数学那些普遍实 19 | 用的最基础的部分,这就是在理论上、应用上和思想方法上 20 | 都是基本的、长远起作用的通性、通法.比如,代数中的数 21 | 系运算律,式的运算,解代数方程,待定系数法;几何中的 22 | 图形的基本概念和主要性质,向量,解析几何;分析中的函 23 | 数,极限,连续,微分,积分;概率统计以及逻辑、推理论 24 | 证等知识.对于那些理论和应用上虽有一定作用,但发展余 25 | 地不大,或没有普遍意义和实用价值,或不必要的重复和过 26 | 于繁琐的内容,如立体几何中的空间作图,几何体的体积、 27 | 表面积计算,几何难题,因式分解,对数计算等作了较大的 28 | 精简或删减. 29 | 30 | 全套教材共分六册.第一册是代数.在总结小学所学自 31 | 然数、小数、分数基础上,明确提出运算律,把数扩充到有 32 | 理数和实数系.灵活运用运算律解一元一次、二次方程,二 33 | 元、三元一次方程组,然后进一步系统化,引进多项式运 34 | 算,综合除法,辗转相除,余式定理及其推论,学到根式、 35 | 分式、部分分式.第二册是几何.由直观几何形象分析归纳 36 | 出几何基本概念和基本性质,通过集合术语、简易逻辑转入 37 | 欧氏推理几何,处理直线形,圆、基本轨迹与作图,三角比 38 | 与解三角形等基本内容.第三册是函数.数形结合引入坐 39 | 标,研究多项式函数,指数、对数、三角函数,不等式等. 40 | 第四册是代数.把数扩充到复数系,进一步加强多项式理论, 41 | 方程式论,讲线性方程组理论,概率(离散的)统计的初步 42 | 知识.第五册是几何.引进向量,用向量和初等几何方法综 43 | 合处理几何问题,坐标化处理直线、圆、锥线,坐标变换与 44 | 二次曲线讨论,然后讲立体几何,并引进空间向量研究空间 45 | 解析几何初步知识.第六册是微积分初步.突出逼近法,讲 46 | 实数完备性,函数,极限,连续,变率与微分,求和与积分. 47 | 48 | 本教材基本上采取代数、几何、分析分科,初中、高中 49 | 循环排列的安排体系.教学可按初一、初二代数、几何双科 50 | 并进,初三学分析,高一、高二代数(包括概率统计)、几 51 | 何双科并进,高三学微积分的程序来安排. 52 | 53 | 本教材的处理力求符合历史发展和认识发展的规律,深 54 | 入浅出,顺理成章.突出由算术到代数,由实验几何到论证 55 | 几何,由综合几何到解析几何,由常量数学到变量数学等四 56 | 个重大转折,着力采取措施引导学生合乎规律地实现这些转 57 | 折,为此,强调数系运算律,集合逻辑,向量和逼近法分别 58 | 在实现这四个转折中的作用.这样既遵循历史发展的规律, 59 | 又突出了几个转折关头,缩短了认识过程,有利于学生掌握 60 | 数学思想发展的脉络,提高数学教学的思想性. 61 | 62 | 这一套中学数学实验教材是教育部委托北京师范大学、 63 | 中国科学院数学研究所、人民教育出版社、北京师范学院、 64 | 北京景山学校等单位组成的领导小组组织“中学数学实验教 65 | 材编写组”,根据美国加州大学伯克利分校数学系项武义教 66 | 授的《关于中学实验数学教材的设想》编写的.第一版印出 67 | 后,由教育部实验研究组和有关省市实验研究组指导在北 68 | 京景山学校、北京师院附中、上海大同中学、天津南开中 69 | 学、天津十六中学、广东省实验中学、华南师院附中、长春 70 | 市实验中学等校试教过两遍,在这个基础上编写组吸收了实 71 | 验学校老师们的经验和意见,修改成这一版《中学数学实验 72 | 教材》,正式出版,内部发行,供中学选作实验教材,教师 73 | 参考书或学生课外读物.在编写和修订过程中,项武义教授 74 | 曾数次详细修改过原稿,提出过许多宝贵意见. 75 | 76 | 本教材虽然试用过两遍,但是实验基础仍然很不够,这 77 | 次修改出版,目的是通过更大范围的实验研究,逐步形成另 78 | 一套现代化而又适合我国国情的中学数学教科书.在实验过 79 | 程中,我们热忱希望大家多提意见,以便进一步把它修改好. 80 | 81 | \begin{flushright} 82 | 中学数学实验教材编写组\\ 83 | 一九八一年三月 84 | \end{flushright} 85 | 86 | 87 | 88 | 89 | 90 | 91 | 92 | -------------------------------------------------------------------------------- /main.tex: -------------------------------------------------------------------------------- 1 | \documentclass[b5paper, openany]{ctexbook} 2 | 3 | 4 | \usepackage[margin=2.5cm]{geometry} 5 | 6 | 7 | \usepackage{pifont} 8 | \usepackage[perpage,symbol*]{footmisc} 9 | \DefineFNsymbols{circled}{{\ding{192}}{\ding{193}}{\ding{194}} 10 | {\ding{195}}{\ding{196}}{\ding{197}}{\ding{198}}{\ding{199}}{\ding{200}}{\ding{201}}} 11 | \setfnsymbol{circled} 12 | 13 | \usepackage{ulem} 14 | 15 | \usepackage{amsmath,amsfonts,mathrsfs,amssymb} 16 | \usepackage{graphicx} 17 | 18 | \usepackage[font=bf,labelfont=bf,labelsep=quad]{caption} 19 | 20 | \usepackage{tikz} 21 | 22 | 23 | \usepackage{ntheorem} 24 | \theoremseparator{\;} 25 | 26 | 27 | 28 | \usepackage{blkarray} 29 | \usepackage{bm} 30 | \usepackage[colorlinks=true, linkcolor=black]{hyperref} 31 | 32 | \usepackage{enumerate} 33 | 34 | 35 | \theoremstyle{plain} 36 | \theoremheaderfont{\normalfont\bfseries} 37 | \theorembodyfont{\normalfont} 38 | 39 | 40 | \usepackage[framemethod=tikz]{mdframed} 41 | 42 | 43 | \newtheorem{example}{\bf 例}[chapter] 44 | \newenvironment{solution}{\noindent {\bf 解:}}{} 45 | \newenvironment{analyze}{\noindent {\bf 分析:}}{} 46 | \newenvironment{rmk}{\noindent {\bf 注意:}}{} 47 | \newenvironment{note}{\noindent {\bf 说明:}}{} 48 | 49 | 50 | 51 | \renewcommand{\proofname}{\bf 证明:} 52 | \newenvironment{proof}{{\noindent \bf 证明:}}{}%{\hfill $\square$\par} 53 | 54 | \newcommand{\E}{\mathbb{E}} 55 | \renewcommand{\Pr}{\mathbb{P}} 56 | \newcommand{\EP}{\mathbb{E}^{\mathbb{P}}} 57 | \newcommand{\EQ}{\mathbb{E}^{\mathbb{Q}}} 58 | \newcommand{\dif}{\,{\rm d}} 59 | \newcommand{\Var}{{\rm Var}} 60 | \newcommand{\Cov}{{\rm Cov}} 61 | \newcommand{\x}{\times} 62 | 63 | 64 | \usepackage{tcolorbox} 65 | \tcbuselibrary{breakable} 66 | \tcbuselibrary{most} 67 | 68 | 69 | 70 | \newtcolorbox{ex}[1][] 71 | {colback = white, colframe = cyan!75!black, fonttitle = \bfseries, 72 | colbacktitle = cyan!85!black, enhanced, 73 | attach boxed title to top center={yshift=-2mm},breakable, 74 | title=练习, #1} 75 | 76 | \newtcolorbox{blk}[2][] 77 | {colback = white, colframe = magenta!75!black, fonttitle = \bfseries, 78 | colbacktitle = magenta!85!black, enhanced, 79 | attach boxed title to top left={xshift=5mm, yshift=-2mm},breakable, 80 | title=#2, #1} 81 | 82 | 83 | \setcounter{tocdepth}{2} 84 | 85 | \setcounter{secnumdepth}{3} 86 | 87 | 88 | 89 | \ctexset { 90 | section = { 91 | name = {第,节}, 92 | number = \chinese{section}}, 93 | subsection = { 94 | name = {,、\hspace{-1em}}, 95 | number = \chinese{subsection} 96 | }, 97 | subsubsection = { 98 | name = {(,)\hspace{-1em}}, 99 | number = \chinese{subsubsection}, 100 | } 101 | } 102 | 103 | 104 | 105 | \renewcommand{\contentsname}{目~~录} 106 | 107 | \newcommand{\poly}{\polynomial[reciprocal]} 108 | 109 | 110 | 111 | \usepackage{mathtools} 112 | 113 | \setlength{\abovecaptionskip}{0.cm} 114 | \setlength{\belowcaptionskip}{-0.cm} 115 | 116 | \usetikzlibrary{decorations.pathmorphing, patterns} 117 | \usetikzlibrary{calc, patterns, decorations.markings} 118 | \usetikzlibrary{positioning, snakes} 119 | 120 | 121 | \usepackage{yhmath} 122 | \usepackage{longdivision} 123 | \usepackage{polynom} 124 | \usepackage{polynomial} 125 | \usepackage{cancel} 126 | 127 | \renewcommand{\frac}{\dfrac} 128 | \newcommand{\oc}{$^{\circ}{\rm C}$} 129 | 130 | \usepackage{tkz-base} 131 | \usepackage{tkz-euclide} 132 | 133 | \usepackage{multicol} 134 | \usepackage{cases} 135 | 136 | \newcommand\parallelogram{% 137 | \mathord{\text{% 138 | \tikz[baseline] 139 | \draw(0em,.1ex)--++(.9em,0ex)--++(.2em,1.2ex)--++(-.9em,0ex)--cycle; 140 | }} 141 | } 142 | 143 | \renewcommand\parallel{\mathrel{/\mskip-2.5mu/}} 144 | 145 | 146 | \begin{document} 147 | 148 | 149 | 150 | \title{\Huge\bfseries 中学数学实验教材\\第二册} 151 | 152 | 153 | 154 | \author{\Large 中学数学实验教材编写组编} 155 | \date{\Large 1982年7月} 156 | 157 | \maketitle 158 | 159 | 160 | 161 | 162 | \frontmatter 163 | 164 | \input{preface.tex} 165 | \tableofcontents 166 | 167 | 168 | \mainmatter 169 | 170 | \input{1.tex} 171 | 172 | \input{2.tex} 173 | \input{3.tex} 174 | \input{4.tex} 175 | \input{5.tex} 176 | \input{6.tex} 177 | 178 | 179 | 180 | \end{document} 181 | 182 | -------------------------------------------------------------------------------- /main - 2A.tex: -------------------------------------------------------------------------------- 1 | \documentclass[b5paper, openany]{ctexbook} 2 | 3 | 4 | \usepackage[margin=2.5cm]{geometry} 5 | 6 | 7 | \usepackage{pifont} 8 | \usepackage[perpage,symbol*]{footmisc} 9 | \DefineFNsymbols{circled}{{\ding{192}}{\ding{193}}{\ding{194}} 10 | {\ding{195}}{\ding{196}}{\ding{197}}{\ding{198}}{\ding{199}}{\ding{200}}{\ding{201}}} 11 | \setfnsymbol{circled} 12 | 13 | \usepackage{ulem} 14 | 15 | \usepackage{amsmath,amsfonts,mathrsfs,amssymb} 16 | \usepackage{graphicx} 17 | 18 | \usepackage[font=bf,labelfont=bf,labelsep=quad]{caption} 19 | 20 | \usepackage{tikz} 21 | 22 | 23 | \usepackage{ntheorem} 24 | \theoremseparator{\;} 25 | 26 | 27 | 28 | \usepackage{blkarray} 29 | \usepackage{bm} 30 | \usepackage[colorlinks=true, linkcolor=black]{hyperref} 31 | 32 | \usepackage{enumerate} 33 | 34 | 35 | \theoremstyle{plain} 36 | \theoremheaderfont{\normalfont\bfseries} 37 | \theorembodyfont{\normalfont} 38 | 39 | 40 | \usepackage[framemethod=tikz]{mdframed} 41 | 42 | 43 | \newtheorem{example}{\bf 例}[chapter] 44 | \newenvironment{solution}{\noindent {\bf 解:}}{} 45 | \newenvironment{analyze}{\noindent {\bf 分析:}}{} 46 | \newenvironment{rmk}{\noindent {\bf 注意:}}{} 47 | \newenvironment{note}{\noindent {\bf 说明:}}{} 48 | 49 | 50 | 51 | \renewcommand{\proofname}{\bf 证明:} 52 | \newenvironment{proof}{{\noindent \bf 证明:}}{}%{\hfill $\square$\par} 53 | 54 | \newcommand{\E}{\mathbb{E}} 55 | \renewcommand{\Pr}{\mathbb{P}} 56 | \newcommand{\EP}{\mathbb{E}^{\mathbb{P}}} 57 | \newcommand{\EQ}{\mathbb{E}^{\mathbb{Q}}} 58 | \newcommand{\dif}{\,{\rm d}} 59 | \newcommand{\Var}{{\rm Var}} 60 | \newcommand{\Cov}{{\rm Cov}} 61 | \newcommand{\x}{\times} 62 | 63 | 64 | \usepackage{tcolorbox} 65 | \tcbuselibrary{breakable} 66 | \tcbuselibrary{most} 67 | 68 | 69 | 70 | \newtcolorbox{ex}[1][] 71 | {colback = white, colframe = cyan!75!black, fonttitle = \bfseries, 72 | colbacktitle = cyan!85!black, enhanced, 73 | attach boxed title to top center={yshift=-2mm},breakable, 74 | title=练习, #1} 75 | 76 | \newtcolorbox{blk}[2][] 77 | {colback = white, colframe = magenta!75!black, fonttitle = \bfseries, 78 | colbacktitle = magenta!85!black, enhanced, 79 | attach boxed title to top left={xshift=5mm, yshift=-2mm},breakable, 80 | title=#2, #1} 81 | 82 | 83 | \setcounter{tocdepth}{2} 84 | 85 | \setcounter{secnumdepth}{3} 86 | 87 | 88 | 89 | \ctexset { 90 | section = { 91 | name = {第,节}, 92 | number = \chinese{section}}, 93 | subsection = { 94 | name = {,、\hspace{-1em}}, 95 | number = \chinese{subsection} 96 | }, 97 | subsubsection = { 98 | name = {(,)\hspace{-1em}}, 99 | number = \chinese{subsubsection}, 100 | } 101 | } 102 | 103 | 104 | 105 | \renewcommand{\contentsname}{目~~录} 106 | 107 | \newcommand{\poly}{\polynomial[reciprocal]} 108 | 109 | 110 | 111 | \usepackage{mathtools} 112 | 113 | \setlength{\abovecaptionskip}{0.cm} 114 | \setlength{\belowcaptionskip}{-0.cm} 115 | 116 | \usetikzlibrary{decorations.pathmorphing, patterns} 117 | \usetikzlibrary{calc, patterns, decorations.markings} 118 | \usetikzlibrary{positioning, snakes} 119 | 120 | 121 | \usepackage{yhmath} 122 | \usepackage{longdivision} 123 | \usepackage{polynom} 124 | \usepackage{polynomial} 125 | \usepackage{cancel} 126 | 127 | \renewcommand{\frac}{\dfrac} 128 | \newcommand{\oc}{$^{\circ}{\rm C}$} 129 | 130 | \usepackage{tkz-base} 131 | \usepackage{tkz-euclide} 132 | 133 | \usepackage{multicol} 134 | \usepackage{cases} 135 | 136 | \newcommand\parallelogram{% 137 | \mathord{\text{% 138 | \tikz[baseline] 139 | \draw(0em,.1ex)--++(.9em,0ex)--++(.2em,1.2ex)--++(-.9em,0ex)--cycle; 140 | }} 141 | } 142 | 143 | \renewcommand\parallel{\mathrel{/\mskip-2.5mu/}} 144 | 145 | 146 | \begin{document} 147 | 148 | 149 | 150 | \title{\Huge\bfseries 中学数学实验教材\\第二册(上)} 151 | 152 | 153 | 154 | \author{\Large 中学数学实验教材编写组编} 155 | \date{\Large 1982年7月} 156 | 157 | \maketitle 158 | 159 | 160 | 161 | 162 | \frontmatter 163 | 164 | \input{preface.tex} 165 | \tableofcontents 166 | 167 | 168 | \mainmatter 169 | 170 | \input{1.tex} 171 | 172 | \input{2.tex} 173 | \input{3.tex} 174 | % \input{4.tex} 175 | % \input{5.tex} 176 | % \input{6.tex} 177 | 178 | 179 | 180 | \end{document} 181 | 182 | -------------------------------------------------------------------------------- /main - 2B.tex: -------------------------------------------------------------------------------- 1 | \documentclass[b5paper, openany]{ctexbook} 2 | 3 | 4 | \usepackage[margin=2.5cm]{geometry} 5 | 6 | 7 | \usepackage{pifont} 8 | \usepackage[perpage,symbol*]{footmisc} 9 | \DefineFNsymbols{circled}{{\ding{192}}{\ding{193}}{\ding{194}} 10 | {\ding{195}}{\ding{196}}{\ding{197}}{\ding{198}}{\ding{199}}{\ding{200}}{\ding{201}}} 11 | \setfnsymbol{circled} 12 | 13 | \usepackage{ulem} 14 | 15 | \usepackage{amsmath,amsfonts,mathrsfs,amssymb} 16 | \usepackage{graphicx} 17 | 18 | \usepackage[font=bf,labelfont=bf,labelsep=quad]{caption} 19 | 20 | \usepackage{tikz} 21 | 22 | 23 | \usepackage{ntheorem} 24 | \theoremseparator{\;} 25 | 26 | 27 | 28 | \usepackage{blkarray} 29 | \usepackage{bm} 30 | \usepackage[colorlinks=true, linkcolor=black]{hyperref} 31 | 32 | \usepackage{enumerate} 33 | 34 | 35 | \theoremstyle{plain} 36 | \theoremheaderfont{\normalfont\bfseries} 37 | \theorembodyfont{\normalfont} 38 | 39 | 40 | \usepackage[framemethod=tikz]{mdframed} 41 | 42 | 43 | \newtheorem{example}{\bf 例}[chapter] 44 | \newenvironment{solution}{\noindent {\bf 解:}}{} 45 | \newenvironment{analyze}{\noindent {\bf 分析:}}{} 46 | \newenvironment{rmk}{\noindent {\bf 注意:}}{} 47 | \newenvironment{note}{\noindent {\bf 说明:}}{} 48 | 49 | 50 | 51 | \renewcommand{\proofname}{\bf 证明:} 52 | \newenvironment{proof}{{\noindent \bf 证明:}}{}%{\hfill $\square$\par} 53 | 54 | \newcommand{\E}{\mathbb{E}} 55 | \renewcommand{\Pr}{\mathbb{P}} 56 | \newcommand{\EP}{\mathbb{E}^{\mathbb{P}}} 57 | \newcommand{\EQ}{\mathbb{E}^{\mathbb{Q}}} 58 | \newcommand{\dif}{\,{\rm d}} 59 | \newcommand{\Var}{{\rm Var}} 60 | \newcommand{\Cov}{{\rm Cov}} 61 | \newcommand{\x}{\times} 62 | 63 | 64 | \usepackage{tcolorbox} 65 | \tcbuselibrary{breakable} 66 | \tcbuselibrary{most} 67 | 68 | 69 | 70 | \newtcolorbox{ex}[1][] 71 | {colback = white, colframe = cyan!75!black, fonttitle = \bfseries, 72 | colbacktitle = cyan!85!black, enhanced, 73 | attach boxed title to top center={yshift=-2mm},breakable, 74 | title=练习, #1} 75 | 76 | \newtcolorbox{blk}[2][] 77 | {colback = white, colframe = magenta!75!black, fonttitle = \bfseries, 78 | colbacktitle = magenta!85!black, enhanced, 79 | attach boxed title to top left={xshift=5mm, yshift=-2mm},breakable, 80 | title=#2, #1} 81 | 82 | 83 | \setcounter{tocdepth}{2} 84 | 85 | \setcounter{secnumdepth}{3} 86 | 87 | 88 | 89 | \ctexset { 90 | section = { 91 | name = {第,节}, 92 | number = \chinese{section}}, 93 | subsection = { 94 | name = {,、\hspace{-1em}}, 95 | number = \chinese{subsection} 96 | }, 97 | subsubsection = { 98 | name = {(,)\hspace{-1em}}, 99 | number = \chinese{subsubsection}, 100 | } 101 | } 102 | 103 | 104 | 105 | \renewcommand{\contentsname}{目~~录} 106 | 107 | \newcommand{\poly}{\polynomial[reciprocal]} 108 | 109 | 110 | 111 | \usepackage{mathtools} 112 | 113 | \setlength{\abovecaptionskip}{0.cm} 114 | \setlength{\belowcaptionskip}{-0.cm} 115 | 116 | \usetikzlibrary{decorations.pathmorphing, patterns} 117 | \usetikzlibrary{calc, patterns, decorations.markings} 118 | \usetikzlibrary{positioning, snakes} 119 | 120 | 121 | \usepackage{yhmath} 122 | \usepackage{longdivision} 123 | \usepackage{polynom} 124 | \usepackage{polynomial} 125 | \usepackage{cancel} 126 | 127 | \renewcommand{\frac}{\dfrac} 128 | \newcommand{\oc}{$^{\circ}{\rm C}$} 129 | 130 | \usepackage{tkz-base} 131 | \usepackage{tkz-euclide} 132 | 133 | \usepackage{multicol} 134 | \usepackage{cases} 135 | 136 | \newcommand\parallelogram{% 137 | \mathord{\text{% 138 | \tikz[baseline] 139 | \draw(0em,.1ex)--++(.9em,0ex)--++(.2em,1.2ex)--++(-.9em,0ex)--cycle; 140 | }} 141 | } 142 | 143 | \renewcommand\parallel{\mathrel{/\mskip-2.5mu/}} 144 | 145 | 146 | \begin{document} 147 | 148 | 149 | 150 | \title{\Huge\bfseries 中学数学实验教材\\第二册(下)} 151 | 152 | 153 | 154 | \author{\Large 中学数学实验教材编写组编} 155 | \date{\Large 1982年11月} 156 | 157 | \maketitle 158 | 159 | 160 | 161 | 162 | \frontmatter 163 | 164 | \input{preface.tex} 165 | \tableofcontents 166 | 167 | 168 | \mainmatter 169 | 170 | % \input{1.tex} 171 | % 172 | % \input{2.tex} 173 | % \input{3.tex} 174 | \setcounter{chapter}{3} 175 | \input{4.tex} 176 | \input{5.tex} 177 | \input{6.tex} 178 | 179 | 180 | 181 | \end{document} 182 | 183 | -------------------------------------------------------------------------------- /5.tex: -------------------------------------------------------------------------------- 1 | \chapter{轨迹与作图} 2 | 在前面的几章中,我们学习了直线形和圆的有关性质. 3 | 学习的途径主要是根据图形的定义和已知性质去推演图形的 4 | 其它性质.这一章,我们将把图形看成点的集合(点集), 5 | 研究如何根据点所具有的某种性质来求出点集在平面上的形 6 | 状和位置. 7 | 8 | \section{轨迹} 9 | \subsection{轨迹的概念} 10 | 我们知道,物体在运动中都要经过一定的路线.例如, 11 | 人在雪地里行走会留下明显的足迹,飞机飞行有一定的航 12 | 线,地球运行也有它的轨道等等.一般,我们常把物体按某 13 | 种规律运动的路线叫做物体运动的\textbf{轨迹}.在几何中,我们用 14 | 点表示物体在空间的位置.这样,一个点在空间按某种规律 15 | 运动的路线,我们就把它叫做这个点运动的轨迹,这个点就 16 | 叫做\textbf{动点}.例如,我们用圆规画圆时,圆规的一个脚尖固定 17 | 不动,而另一个脚上装上的铅笔尖端就可看作一个动点.它 18 | 和固定的脚尖保持一定的距离运动,所画出的图形就是这个 19 | 动点的轨迹.我们知道,圆是“同一平面上和某定点的距离 20 | 等于定长的点的集合”.由此可见,按某种规律运动的点的 21 | 轨迹,也就是具有某种性质的点的集合. 22 | 23 | \begin{blk}{定义} 24 | 具有性质$\alpha$的所有点构成的集合,叫做具有性 25 | 质$\alpha$的点的轨迹. 26 | \end{blk} 27 | 28 | 设$X=\{\text{具有性质$\alpha$的点}\}$.由上述定义,当我们要证明 29 | 某图形$A$是具有某种性质$\alpha$的点的轨迹时,也就是要证明 30 | 集合$A=X$. 要证明$A=X$, 就必须从以下两方面进行证 31 | 明: 32 | \begin{enumerate} 33 | \item $P$点$\in A\Rightarrow P$点具有性质$\alpha$ $(P\in X)$. 34 | \item $P$点具有性质$\alpha$ $(P\in X)\Rightarrow P$点$\in A$. 35 | \end{enumerate} 36 | 37 | 按上述两个方面证明,这是缺一不可的.如果我们只证 38 | 了第一条,实际上只是说明$A$是$X$的一个子集,并不能断 39 | 定$A=X$; 如果只证了第二条,也只是说$X$是$A$的一个子 40 | 集,同样不能断定$A=X$, 只有当我们证明了第一条:$A\subseteq 41 | X$, 又证明了第二条:$X\subseteq A$, 我们才能断定$A=X$. 42 | 43 | 第一条证明了$A\subseteq X$, 这就是说在图形$A$上的点,都具 44 | 有性质$\alpha$. 没有一点是鱼目混珠的,通常把证这一条叫做证 45 | \textbf{轨迹的纯粹性}.第二条证明了$X\subseteq A$, 这就是说,具有性质 46 | $\alpha$的点都在图形$A$上,没有一点被遗漏掉.通常又把证这一 47 | 条叫做证\textbf{轨迹的完备性}. 48 | 49 | 由于原命题与逆否命题等价,所以也可以分别去证上述 50 | 两条的逆否命题,即要证轨迹的纯粹性也可证: 51 | \[P\text{点不具有性质}\alpha\Rightarrow P\notin A\] 52 | 要证轨迹的完备性时,也可证: 53 | \[P\text{点}\notin A\Rightarrow P\text{点不具有性质}\alpha\] 54 | 55 | \begin{ex} 56 | \begin{enumerate} 57 | \item 叙 58 | 述两个集合相等的定义. 59 | \item 在证轨迹命题时,为什么即要证轨迹的纯粹性,又要证 60 | 轨迹的完备性? 61 | \item 如果我们证明了 62 | $\overline{AB}$的垂直平分线上的任一点到$A$、 63 | $B$两 64 | 点的距离相等,能否就说与$A$、$B$两点距离相等的点 65 | 的轨迹是$\overline{AB}$的垂直平分线? 66 | \end{enumerate} 67 | \end{ex} 68 | 69 | 70 | \subsection{基本轨迹} 71 | 这一小节,我们来学习六个平面上的点的基本轨迹,我 72 | 们只证了1和4, 其它四个由同学们自证. 73 | 74 | \begin{blk}{基本轨迹1} 75 | 与两个已知点距离相等的点的轨迹是连结 76 | 这两点的线段的垂直平分线. 77 | \end{blk} 78 | 79 | 已知:两定点$A$、$B$, 直线$MN$是$\overline{AB}$的垂直平分线(图5.1). 80 | 81 | 求证:与$A$、$B$两点距离相等的点的轨迹是直线$MN$. 82 | 83 | \begin{figure}[htp] 84 | \centering 85 | \begin{tikzpicture}[scale=.7] 86 | \draw(-2,0)node[left]{$A$}--(2,0)node[right]{$B$}; 87 | \draw (0,3)node[above]{$M$}--(0,-2.5)node[below]{$N$}; 88 | \draw[dashed](-2,0)--(0,2.5)node[left]{$P$}--(2,0)--(0,-1.8)node[left]{$Q$}--(-2,0); 89 | \node at (.25,.25){$O$}; 90 | 91 | \end{tikzpicture} 92 | \caption{} 93 | \end{figure} 94 | 95 | 96 | \begin{proof} 97 | \begin{enumerate} 98 | \item 99 | 设$P$是直线$MN$上的任一点,作$\overline{PA}$、$\overline{PB}$, 100 | 在$\triangle AOP$与$\triangle BOP$中, 101 | 102 | $\because\quad \overline{AO}=\overline{BO},\quad \angle AOP=\angle BOP,\quad \overline{OP}=\overline{OP}$ 103 | 104 | $\therefore\quad \triangle AOP\cong \triangle BOP$ (SAS), 105 | $\overline{PA}=\overline{PB}$. 106 | 107 | 这就说明了直线$MN$上的点,都与两点的距离相等. 108 | 109 | \item 设$Q$为与$A$、$B$等距的点,即$\overline{QA}=\overline{QB}$. 过$AB$的 110 | 中点$O$与$Q$作直线$OQ$, 根据等腰三角形的性质,直线$OQ$ 111 | 垂直平分$\overline{AB}$, 但$\overline{AB}$的垂直平分线只有一条, 112 | 113 | $\therefore\quad MN$与$OQ$重合,$Q\in MN$. 114 | 115 | \end{enumerate} 116 | 117 | 于是由1、2可知,与$A$、$B$两点距离相等的点的 118 | 轨迹是直线$MN$. 119 | \end{proof} 120 | 121 | \begin{blk} 122 | {基本轨迹2} 与已知角的两边距离相等的点的轨迹是这 123 | 个已知角的平分线. 124 | \end{blk} 125 | 126 | \begin{blk} 127 | {基本轨迹3} 与两条平行线等距离的点的轨迹是和这两 128 | 条平行线平行且平分它们的公垂线段的直线. 129 | \end{blk} 130 | 131 | \begin{blk} 132 | {基本轨迹4}与一条直线的距离等于定长的点的轨迹,是 133 | 平行于这条直线,并和这条直线的距离等于定长的两条直线. 134 | \end{blk} 135 | 136 | 已知:直线$CD\parallel$直线$AB$; 直线$EF\parallel$直线$AB$; $CD$、$EF$和$AB$之间的距离都是$d$(图5.2). 137 | 138 | 求证:与$AB$的距离等于 139 | $d$的点的轨迹是$CD$和$EF$. 140 | 141 | \begin{figure}[htp] 142 | \centering 143 | \begin{tikzpicture} 144 | \draw (0,0)node[left]{$E$}--(4,0)node[right]{$F$}; 145 | \draw (0,1)node[left]{$A$}--(4,1)node[right]{$B$}; 146 | \draw (0,2)node[left]{$C$}--(4,2)node[right]{$D$}; 147 | \draw[dashed, thick] (1.5,-1)--(1.5,3); 148 | \draw[thick] (3,1)node[below]{$L$}--(3,2)node[above]{$P$}; 149 | \node at (1.8,0)[below]{$N'$}; 150 | \node at (1.8,1)[above]{$L'$}; 151 | \node at (1.8,2)[above]{$M'$}; 152 | \draw (1.5,.6) [fill=black] circle (1.5pt) node[right]{$P'$}; 153 | \end{tikzpicture} 154 | 155 | \caption{} 156 | \end{figure} 157 | 158 | 159 | \begin{proof} 160 | \begin{enumerate} 161 | \item 设$P$是$CD$或$EF$ 上的任一点.作$PL\bot AB$于$L$点. 162 | 163 | $\because\quad CD\parallel AB$且和$AB$的距离等于$d$ 164 | 165 | $\therefore\quad \overline{PL}$是$AB$和$CD$的公垂线段,且$\overline{PL}=d$. 166 | 167 | 这就是说$CD$上的任一点和$AB$的距离都等于$d$, 同理 168 | 可证$EF$上的任一点和$AB$的距离也都等于$d$. 169 | \item 设$P'$点是不在$CD$或$EF$上的任一点. 170 | 经过$P'$点作垂直于$AB$的直线,分别交$AB$、$CD$、$EF$于 171 | $L'$、$M'$、$N'$, 则$\overline{M'L'}=\overline{N'L'}=d$. 172 | 173 | $\because\quad P'$不在$CD$或$EF$上 174 | 175 | $\therefore\quad P'$不和$M'$、$N'$重合 176 | 177 | $\because\quad $在直线$M'N'$上和$L'$距离等于$d$的点只有$M'$、$N'$ 178 | 179 | $\therefore\quad \overline{P'L'}\ne d$ 180 | 181 | 这就是说,不在$CD$和$EF$上的任何一点和$AB$的距离都 182 | 不等于$d$. 183 | \end{enumerate} 184 | 185 | 于是由1、2可知,和$AB$的距离等于$d$的点的轨 186 | 迹是$CD$和$EF$. 187 | \end{proof} 188 | 189 | \begin{blk} 190 | {基本轨迹5} 与一个定点的距离等于定长的点的轨迹, 191 | 是以定点为圆心,定长为半径的一个圆. 192 | \end{blk} 193 | 194 | \begin{blk} 195 | {基本轨迹6}与一条定线段的两端连线所夹的角等于定角的点的轨迹,是以这条定线段为弦,所含的圆周角等于定 196 | 角的两条弧. 197 | \end{blk} 198 | 199 | 以上六个基本轨迹是研究其它轨迹问题的基础,同学们 200 | 一定要熟记. 201 | 202 | \begin{example} 203 | 求已知圆内等于定长的弦的中点的轨迹. 204 | 205 | 已知$\odot (O,r)$和定长$a$, 且$a<2r$(图5.3). 206 | 207 | 求$\odot (O,r)$内等于定长$a$的弦的中点的轨迹. 208 | \end{example} 209 | 210 | \begin{figure}[htp] 211 | \centering 212 | \begin{tikzpicture}[scale=1.6] 213 | \draw[very thick] (0,0) circle (1.414) ; 214 | \draw[very thick, dotted] (0,0) circle (1) ; 215 | \draw (-45:1.414)node[right]{$B$}--(-45-90:1.414)node[left]{$A$}; 216 | \draw[dashed] (80:1.414)--(0,0)--node[left]{$r$}(-45-90:1.414); 217 | \draw [thick] (0,0)--node[right]{$r'$}(0,-1)node[below]{$M$}; 218 | \draw (80:1.414)node[above]{$C$}--(-10:1.414)node[below]{$D$}; 219 | \draw [thick] (0,0)--(35:1)node[right]{$P$}; 220 | 221 | \draw[|-|](-4,1)--node[above]{$a$}(-2,1); 222 | \end{tikzpicture} 223 | \caption{} 224 | \end{figure} 225 | 226 | \begin{solution} 227 | 如图5.3, 设$\overline{AB}$是$\odot (O,r)$内等于定长$a$的弦,$M$ 228 | 是它的中点,作$\overline{OM}$, 那么,$\overline{OM}\bot \overline{AB}$. 229 | \[\overline{AM}=\frac{1}{2}\overline{AB}=\frac{a}{2}\] 230 | 所以 231 | \[\overline{OM}=\sqrt{\overline{OA}^2-\overline{AM}^2}=\sqrt{r^2-\left(\frac{a}{2}\right)^2}\] 232 | 设$r'=\sqrt{r^2-\left(\frac{a}{2}\right)^2}$,则$r'$ 233 | 为定长,以$O$为圆心,$r$为 234 | 半径画$\odot (O,r')$, 那么,$\odot (O,r)$内等于定长$a$的弦的中 235 | 点都在$\odot (O,r')$上.另外,在$\odot (O,r')$上任取一点$P$, 作 236 | $\overline{OP}$, 再作弦$\overline{CD}\bot\overline{OP}$于$P$点,则$P$点是$\overline{CD}$弦的中点,且 237 | \[\overline{CD}=2\overline{CP}=2\sqrt{r^2-{r'}^2}=2\sqrt{r^2-\left[r^2-\left(\frac{a}{2}\right)^2\right]}=2\cdot\frac{a}{2}=a\] 238 | 这就是说$\odot (O,r')$上的任一点都是$\odot (O,r')$内等于定长$a$ 239 | 的一条弦的中点,所以我们所求的轨迹就是$\odot (O,r')$. 240 | \end{solution} 241 | 242 | \begin{example} 243 | 过定圆外一定点引圆的割线,求割线被圆截下的弦的中点的轨迹. 244 | 245 | 已知:定$\odot (O,r)$和$\odot O$外定点$P$(图5.4). 246 | 247 | 求:过$P$点引$\odot O$的割线,被$\odot O$截下的弦的中点 248 | 的轨迹. 249 | \end{example} 250 | 251 | 252 | \begin{figure}[htp] 253 | \centering 254 | \begin{tikzpicture}[scale=1.2] 255 | \tkzDefPoints{0/0/O, 2.5/0/O', 5/0/P} 256 | \draw[dashed] (P)--(-2,0)node[left]{$E'$}; 257 | \draw (0,0) circle (2); 258 | \draw[dashed] (O') circle (2.5); 259 | \tkzDefTangent[from with R=P](O, 2cm) 260 | \tkzGetPoints{A}{B} 261 | \tkzDrawSegments[add=0 and .2](P,A P,B) 262 | \node at (-.25,-.25){$O$};\node at (.25+2.5,.25){$O'$}; 263 | \node at (2.2,0)[above]{$E$}; 264 | \tkzDrawPoints(O',O) 265 | \tkzDefShiftPoint[O'](150:2.5){M} 266 | \tkzDefShiftPoint[O'](-142:2.5){N} 267 | \tkzDefShiftPoint[O'](-155:2.5){X} 268 | \tkzAutoLabelPoints[center=O'](M,N,X,A,B,P) 269 | \tkzInterLC(P,M)(O,A) \tkzGetPoints{C'}{C} 270 | \tkzInterLC(P,X)(O,A)\tkzGetPoints{D}{D'} 271 | \tkzInterLC(P,N)(O,A)\tkzGetPoints{F}{F'} 272 | \tkzAutoLabelPoints[center=O](D,F,C,C',D',F') 273 | \draw(C')--(P)--(F'); 274 | \draw(D')--(P); 275 | \draw[dashed](M)--(O)--(X); 276 | \draw[dashed](A)--(O); 277 | \end{tikzpicture} 278 | \caption{} 279 | \end{figure} 280 | 281 | \begin{solution} 282 | 由于轨迹是具有某种性质$\alpha$的点的集合,求轨迹时,可先按照“性质”画出一些点,看看这些点可 283 | 能构成什么样的图形,如图 284 | 5.4, 过$P$点作$\odot O$的割线,与$\odot O$相交于$C$、$C'$, 作弦 285 | $\overline{CC'}$的中点$M$, 我们证割线$PCC'$绕$P$点旋转,看这条变动 286 | 的割线被$\odot O$截下的弦的中点经过什么路线.大概可以看 287 | 出,可能是一段圆弧,究竟是不是圆弧,如果是圆弧,又如 288 | 何把它作出来,还要进一步分析. 289 | 290 | $\because\quad M$是弦$\overline{CC'}$的中点,作$\overline{OM}$ 291 | 292 | 则$\overline{OM}\bot\overline{CC'}$,即:$\angle OMP$是直角. 293 | 294 | 这就是说,过$P$点作$\odot O$的任一条 割线 295 | 被$\odot O$截下的弦的中点与$O$、$P$的连线的夹角等于直角,因 296 | 此,符合题中条件的弦的中点都在以$\overline{OP}$ 297 | 为直径的圆上,以$\overline{OP}$ 298 | 为直径作$\odot O'$, 我们所作的$\odot O'$是不是就是所求的轨迹呢? 299 | 这还要看$\odot O'$上有没有不符合条件的点.设$\odot O'$与$\odot O$相 300 | 交于$A$、$B$两点.显然,在$\odot O'$上$\wideparen{AOB}$外的点都是不合条 301 | 件的(包括$A$、$B$),我们再来看$\wideparen{AOB}$上的点是不是都是合 302 | 条件的点.在 303 | $\wideparen{AOB}$上任取一点$X$, 设$PX$与$\odot O$相交于$D$、 304 | $D'$, 作$\overline{OX}$, 则$\overline{OX}\bot \overline{DD'}$,所以$X$是$\overline{DD'}$的中点,这就 305 | 是说$\wideparen{AOB}$上的点都是过$P$点的某条割线被$\odot O$截下的弦的中 306 | 点. 307 | \end{solution} 308 | 309 | 综合以上分析我们可得:过定圆外一定点引圆的割线, 310 | 割线被圆截下的弦的中点的轨迹是以定点与圆心间的线段为 311 | 直径的圆被夹在定圆内的一段弧. 312 | 313 | \begin{ex} 314 | 说出下列的点的轨迹是什么图形?并把它们分别画出 315 | 来. 316 | \begin{enumerate} 317 | \item 到一条5cm长的线段的两端距离相等的点的轨迹. 318 | \item 通过两定点的圆的圆心的轨迹. 319 | \item 到一个等于60$^{\circ}$的已知角的两边距离相等的点的轨述: 320 | \item 与两条相交直线等距离的点的轨迹. 321 | \item 与$\angle AOB$的两边都相切的圆的圆心的轨迹. 322 | \item 与距离是3cm的两条平行直线$AB$、$CD$的距离相等的点 323 | 的轨迹. 324 | \item 与距离是3cm的两条平行直线都相切的圆的圆心的轨 325 | 迹. 326 | \item 和已知直线$AB$的距离等于2cm的点的轨迹. 327 | \item 和已知直线$AB$相切,并且半径等于1.5cm的圆的圆心 328 | 的轨迹. 329 | \item 和一条已知直线切于已知点的圆的圆心的轨迹. 330 | \item 与定点$A$的距离等于2cm的点的轨迹. 331 | \item 和一条长是3cm的$AB$的两端连线所夹的角是直角的点 332 | 的轨迹. 333 | \item 和一条长是4cm的已知线段$AB$的两端连线所夹的角等 334 | 于60$^{\circ}$的点的轨迹. 335 | 336 | \end{enumerate} 337 | \end{ex} 338 | 339 | 340 | \section*{习题5.1} 341 | \addcontentsline{toc}{subsection}{习题5.1} 342 | \begin{enumerate} 343 | \item 求下列轨迹 344 | \begin{enumerate} 345 | \item 以已知$\overline{AB}$为一边的三角形的外心的轨迹. 346 | \item 以已知$\overline{AB}$为一边,并且这边上的中线的长等于定长$m$的三角形的重心的轨迹. 347 | \item 以3cm长的已知$\overline{AB}$为一边,并且面积等于6平方厘 348 | 米的三角形的顶点$C$的轨迹. 349 | \item 和一个半径等于定长$r$的$\odot O$外切,并且半径等于 350 | $r'$的圆的圆心的轨迹. 351 | \item 以已知$\overline{BC}$为斜边的直角$\triangle ABC$的顶点$A$的轨迹. 352 | \end{enumerate} 353 | 354 | \item 叙述符合下列条件的点的轨迹. 355 | \begin{enumerate} 356 | \item 平行于三角形的一边而夹在其余两边之间的线段的 357 | 中点的轨迹. 358 | \item 平行于已知直线而在已知圆内的弦的中点的轨迹. 359 | \end{enumerate} 360 | \item 作出下列各题中给出的两个点集,向它们的交集各含有 361 | 几个元素. 362 | 363 | \begin{enumerate} 364 | \item 和距离等于3cm的两条已知平行线$AB$、$CD$的距离 365 | 相等的点集;和直线$AB$上的一定点$E$的距离是2cm的点 366 | 集. 367 | \item 和已知$\angle AOB$的两边的距离相等的点集;和边$OA$ 368 | 的距离等于$d$的点集. 369 | \item 和一条长3cm的已知$\overline{AB}$的两端连线所夹的角是直角 370 | 的点集;和$\overline{AB}$所在直线距离等于2cm的点集. 371 | \end{enumerate} 372 | 373 | 374 | \item 求通过$\odot (O,r)$内一定点$P$的弦的中点的轨迹. 375 | \item 求到$\odot(O,3{\rm cm})$的圆面等于4cm的点的轨迹. 376 | \end{enumerate} 377 | 378 | \section{作图} 379 | \subsection{基本作图} 380 | 在前几章中,我们曾用直尺、圆规解过不少作图题.在 381 | 这一节里,我们将进一步学习解作图题的一些重要方法,下 382 | 面列出我们已经学过的一些作图题(具体作法不再写出,由 383 | 同学自己复习、研究),这些作图题一般叫做\textbf{基本作图题}, 384 | 它们是进一步解较复杂的作图题的基础. 385 | \begin{enumerate} 386 | \item 作一条线段等于已知线段,作一条线段等于$n$条线 387 | 段的和($n\ge 2$ 且 $n\in\mathbb{N}^+$). 388 | \item 作一条线段等于两条已知线段的差. 389 | \item 作一个角等于已知角. 390 | \item 平分一个已知角. 391 | \item 过已知直线上或已知直线外一点,作已知直线的垂 392 | 线. 393 | \item 作已知线段的垂直平分线. 394 | \item 等分已知线段. 395 | \item 按已知条件作三角形. 396 | \begin{enumerate} 397 | \item 已知三边. 398 | \item 已知两边及其夹角. 399 | \item 已知两角及其中一角的对边. 400 | \item 已知两角及其夹边. 401 | \end{enumerate} 402 | \item 作三角形的外接圆和内切圆. 403 | \item 已知斜边和一直角边,作直角三角形. 404 | \item 已知线段$a$, 作一线段$x=\frac{m}{n}a$. (其中$\frac{m}{n}$ 405 | 是正有理数). 406 | \item 作已知三条线段$a$、$b$、$c$的比例第四项. 407 | \item 已知线段$a$、$b$, 作$a$、$b$的比例中项. 408 | \item 已知线段$a$、$b$ $(a>b)$, 作线段$x=\sqrt{a^2+b^2}$ 409 | 或$x=\sqrt{a^2-b^2}$. 410 | \item 已知线段$a$, 作线段$x=\sqrt{\frac{m}{n}}a$ ($\frac{m}{n}$为正有理数). 411 | \item 过已知圆上一点作圆的切线. 412 | \item 过已知圆外一点作圆的切线. 413 | \item 作两个已知圆的公切线. 414 | \item 过已知直线外的一个已知点,作这条直线的平行 415 | 线. 416 | \end{enumerate} 417 | 418 | \begin{ex} 419 | \begin{enumerate} 420 | \item 作一直角三角形$ABC$, 使$\angle C=90^{\circ}$, $AB=3$cm, $BC= 421 | 2$cm. 你能想出几种作法? 422 | \item 已知线段$a$、$b$,你能用几种方法作线段$x=\sqrt{ab}$. 423 | \item 已知线段$a$, 作线段$x=\sqrt{\frac{2}{3}a}$ 424 | \item 过圆外一点,你能用几种方法作这个圆的切线. 425 | \end{enumerate} 426 | \end{ex} 427 | 428 | \subsection{轨迹法作图} 429 | 我们在解作图题时,常常归结为要确定某些点的位置, 430 | 而这些点所要满足的条件又往往不是一个,我们只要根据点 431 | 所满足的各个条件,分别作出相应的轨迹,那么,这些轨迹 432 | 交集中的点,就是我们所要求作的点. 433 | 434 | 例如,已知两个定点$B$、$C$, 且 435 | $\overline{BC}=3$cm (图5.5),以 436 | $B$、$C$为两个顶点,求作一三角形,使第三个顶点与$B$的距 437 | 离是2cm, 与$C$的距离是4cm, 这个作图题,实际上就是确定 438 | 三角形的第三个顶点的位置.第三个顶点要满足两个条件: 439 | \begin{enumerate} 440 | \item 和$B$点的距离是2cm 441 | \item 和$C$点的距离是4cm 442 | \end{enumerate} 443 | 满足第一个条件的点的轨迹是$\odot(B,2{\rm cm})$,满足第二个条件的 444 | 点的轨迹是$\odot (C,4{\rm cm})$,所以第三个顶点就应该是$\odot(B,2{\rm cm})$与$\odot (C,4{\rm cm})$的交集中的点.由作图可知: 445 | \[\odot(B,2{\rm cm})\cap \odot (C,4{\rm cm})=\{A, A'\}\] 446 | 于是,$A$点和$A'$点都是我们所 447 | 要求作的点,$\triangle ABC$与$\triangle A'BC$都是我们所要求作的三角 448 | 形.像这样应用轨迹的交集来确定点的位置,从而来解作图 449 | 题的方法,就叫做\textbf{轨迹法}. 450 | 451 | \begin{figure}[htp]\centering 452 | \begin{minipage}[t]{0.48\textwidth} 453 | \centering 454 | \begin{tikzpicture}[>=latex, scale=.8, rotate=-15] 455 | \tkzDefPoints{0/0/B, 3/0/C, -.5/1.94/A,-.5/-1.94/A'} 456 | \draw(A)--(B)--(C)--(A); 457 | \draw(B)--(A')--(C); 458 | \tkzDrawArc[delta=10](C,A)(A') 459 | \tkzCompass(B,A) 460 | \tkzCompass(B,A') 461 | \tkzLabelPoints[above](A,C) 462 | \tkzLabelPoints[left](A',B) 463 | 464 | \end{tikzpicture} 465 | \caption{} 466 | \end{minipage} 467 | \begin{minipage}[t]{0.48\textwidth} 468 | \centering 469 | \begin{tikzpicture}[>=latex, scale=1.7] 470 | \draw (0,0) circle(1); 471 | \tkzDefPoints{0/0/O} 472 | \tkzDrawPoints(O) 473 | \tkzLabelPoints[below](O) 474 | \tkzDefPoint(-5:1){C} 475 | \tkzDefPoint(45:1){D} 476 | \tkzDrawLines[add=.5 and .5](O,C O,D) 477 | \draw[dashed](15:1)node[right]{$B$}--(0,0)--(75:1)node[above]{$A$}; 478 | \draw (75:1)--(15:1); 479 | \node at (-5:1.5)[right]{$\ell$}; 480 | \node at (45:1.5)[right]{$m$}; 481 | 482 | \end{tikzpicture} 483 | \caption{} 484 | \end{minipage} 485 | \end{figure} 486 | 487 | 488 | 489 | \begin{example} 490 | 已知一条定直线和直线外两个定点,求作一个 491 | 圆,使圆心在这条直线上,并且经过这两个定点. 492 | \end{example} 493 | 494 | 已知:定直线$\ell$和两定点$AB$, 且$A\notin \ell$, $B\notin \ell$(图5.6). 495 | 496 | 求作:一圆使圆心在$\ell$上,且经过$A$、$B$两点. 497 | 498 | 分析:假定$\odot O$为所求作的圆;那么,圆心$O$应满足两 499 | 个条件: 500 | \begin{enumerate} 501 | \item $O\in \ell$; 502 | \item $O$点到$A$、$B$两点等距离. 503 | \end{enumerate} 504 | 因为$\ell$ 505 | 是已知的直线,而到$A$、$B$两点等距离的点的轨迹是$\overline{AB}$的 506 | 垂直平分线,所以,圆心$O$应是$\ell\cap \overline{AB}$的垂直平分线中的点,于是得作法如下: 507 | 508 | 作法: 509 | \begin{enumerate} 510 | \item 作$\overline{AB}$; 511 | \item 作$\overline{AB}$的垂直平分线$m$与$\ell$相交于$O$点, 512 | \item 以$O$为圆心,$OA$为半径作$\odot O$, 则$\odot O$即为所求 513 | 作的圆. 514 | \end{enumerate} 515 | 516 | \begin{proof} 517 | 作$\overline{OA}$、$\overline{OB}$, 518 | 519 | $\because\quad m$是$\overline{AB}$的垂直平分线,且$O\in m$. 520 | 521 | $\therefore\quad \overline{OA}=\overline{OB}$ 522 | 523 | \begin{multicols}{2} 524 | $\because\quad A$点$\in \odot O$, 525 | 526 | $\therefore\quad B$点$\in \odot O$, 527 | 528 | 又知$O$点$\in\ell$, 529 | 530 | $\therefore \odot O$为所求作的圆. 531 | \end{multicols} 532 | 533 | \end{proof} 534 | 535 | 讨论: 536 | \begin{enumerate} 537 | \item 当直线$\ell$与AB所在的直线不垂直时,$\overline{AB}$的垂直平分线$m$与$\ell$一定有一个交点,且只有一个交点.这时,问 538 | 题有一解. 539 | \item 当$\ell$与直线$\overline{AB}$垂直时,如果$\ell$与$m$重合时,问题 540 | 有无穷多解;如果$\ell\parallel m$时问题无解. 541 | \end{enumerate} 542 | 543 | \begin{example} 544 | 已知三角形的一边和这条边上的中线及高,求作 545 | 三角形. 546 | \end{example} 547 | 548 | 已知:线段$a$、$m$、$h$(图5.7). 549 | 550 | \begin{figure}[htp] 551 | \centering 552 | \begin{tikzpicture}[scale=1.7] 553 | \begin{scope} 554 | \draw(0,0)--node[above]{$h$}(1.4,0); 555 | \draw(0,.5)--node[above]{$m$}(1.6,.5); 556 | \draw(0,1)--node[above]{$a$}(2,1); 557 | \end{scope} 558 | \begin{scope}[xshift=3cm] 559 | 560 | 561 | \draw(0,0)node[left]{$B$}--node[below]{$a$}(2,0)node[right]{$C$}; 562 | \draw(-.5,1.4)--(1.8,1.4); 563 | \tkzDefPoints{1/0/M, 0.23/1.4/A} 564 | \tkzCompass(M,A) 565 | \draw(M)--node[right]{$m$}(A)--(2,0); 566 | \draw(0,0)--(A)--node[right]{$h$}(0.23,0)node[below]{$D$}; 567 | \tkzLabelPoints[above](M,A) 568 | \node (D) at (0.23,0){}; 569 | \tkzMarkRightAngle[size=.1](C,D,A) 570 | \end{scope} 571 | \end{tikzpicture} 572 | \caption{} 573 | \end{figure} 574 | 575 | 求作:三角形使它的一边等于$a$, 且这边上的中线等于 576 | $m$, 高等于$h$. 577 | 578 | 分析:画一草图(图5.7),假设$\triangle ABC$为所求作的三 579 | 角形,且$\overline{BC}=a$, $\overline{BC}$上的中线$\overline{AM}=m$, 高$\overline{AD}=h$, $B$、 580 | $C$两个顶点由条件$\overline{BC}=a$, 很容易确定;$A$点的位置应该 581 | 满足条件: 582 | \begin{enumerate} 583 | \item 与$\overline{BC}$的中点$M$的距离等于$a$的长; 584 | \item 与$\overline{BC}$所在直线的距离等于$h$. 585 | \end{enumerate} 586 | 分别满足条件1和2的轨迹 587 | 都可作,故$A$的位置可作出,于是所要求作的三角形可作. 588 | 589 | 作法:(图5.8) 590 | \begin{figure}[htp] 591 | \centering 592 | \begin{tikzpicture}[scale=1.7] 593 | \draw(0,0)node[left]{$B$}--(2,0)node[right]{$C$}; 594 | \draw(-.5,1.4)--(2,1.4)node[right]{$\ell$}; 595 | \tkzDefPoints{1/0/M, 0.23/1.4/A} 596 | \tkzCompass(M,A) 597 | \draw(M)node[below]{$M$}--node[right]{$m$}(A)--(2,0); 598 | \draw(0,0)--(A)--(0.23,0)node[below]{$D$}; 599 | \draw[dashed](1.5,0)--node[right]{$h$}(1.5,1.4); 600 | \tkzLabelPoints[above](A) 601 | \node (D) at (0.23,0){}; 602 | \tkzMarkRightAngle[size=.1](C,D,A) 603 | \end{tikzpicture} 604 | \caption{} 605 | \end{figure} 606 | 607 | \begin{enumerate} 608 | \item 任取一点$B$, 作$\overline{BC}=a$, 609 | \item 作直线$\ell\parallel$直线$BC$, 610 | 且与直线$BC$的距离等于$h$. 611 | \item 作$\overline{BC}$的中点$M$, 作$\odot(M,m)$交$\ell$于$A$点 612 | \item 作$\overline{AB}$、$\overline{AC}$, 则$\triangle 613 | ABC$为所求作的三角形. 614 | \end{enumerate} 615 | 616 | \begin{proof} 617 | (略) 618 | \end{proof} 619 | 620 | 讨论 621 | \begin{enumerate} 622 | \item 当$m\ge h$时,由于$\odot(M,m)$与直线$\ell$能够相 623 | 交,所以问题有一解. 624 | \item 当$m=latex, scale=1] 669 | \foreach \x in {0,20,40} 670 | { 671 | \draw(0,0)--(\x:5); 672 | } 673 | \draw(2,1)--(5,1)node[right]{$\ell$}; 674 | \draw[dashed](4.5,0)node[below]{$C$}--node[right]{$r$}(4.5,1); 675 | \draw(2.75,1) circle(1); 676 | \draw(2.75,0)node[below]{$E$}--(2.75,1)node[above]{$O$}--(40:2.75)node[above]{$D$}; 677 | \node at (0,0) [left]{$B$}; 678 | \node at (20:5) [right]{$F$}; 679 | \node at (40:5) [right]{$A$}; 680 | \draw(0,2)--node[above]{$r$}(1,2); 681 | \end{tikzpicture} 682 | \caption{} 683 | \end{minipage} 684 | \begin{minipage}[t]{0.48\textwidth} 685 | \centering 686 | \begin{tikzpicture}[>=latex, scale=1] 687 | \foreach \x/\xtext in {0/C,20/F,40/A} 688 | { 689 | \draw(0,0)--(\x:5)node[right]{$\xtext$}; 690 | } 691 | \draw(2,1)--(5,1); 692 | \draw(2.75,1) circle(1); 693 | \draw(2.75,0)--node[right]{$r$}(2.75,1)node[above]{$O$}--(40:2.75); 694 | \node at (0,0) [left]{$B$}; 695 | \tkzDefPoint(40:2.75){D} 696 | \tkzDefPoint(0:2.75){E} 697 | \tkzDefPoint(2.75,1){O}\tkzDefPoint(0,0){B} 698 | \tkzMarkRightAngles[size=.1](B,D,O B,E,O) 699 | 700 | 701 | \end{tikzpicture} 702 | \caption{} 703 | \end{minipage} 704 | \end{figure} 705 | 706 | \begin{proof} 707 | 作$\overline{OD} \bot BA$于$D$点,$\overline{OE}\bot BC$于$E$点. 708 | 709 | $\because\quad EF$平分$\angle ABC$, 且$O\in BF$, 710 | 711 | $\therefore\quad \overline{OD}=\overline{OE}$ 712 | 713 | 又$\because\quad O\in\ell$, 而$\ell$与$BC$的距离是$r$的长. 714 | 715 | $\therefore\quad \overline{OD}=\overline{OE}=r,\quad D\in \odot O,\quad E\in \odot O$ 716 | 717 | $\odot O$与$BA$和$BC$都相切,$\odot O$为所求作的圆. 718 | \end{proof} 719 | 720 | 讨论:由于$\angle ABC$的平分线$BF$与直线$\ell$不会平行,所 721 | 以它们总有一个唯一的交点,所以此题只有一解. 722 | 723 | 从例5.5中可以看到,$\wideparen{DE}$把$\angle ABC$的两边在切点$D$、$E$ 724 | 处连接起来了,这种连结叫做\textbf{直线与圆弧的平滑连接}. 725 | 726 | \begin{example} 727 | 已知:$\odot (O_1,r_1)$和$\odot (O_2,r_2)$外离,且$r_1= 728 | 10$mm, $r_2=8$mm(图5.11). 729 | 730 | 求作:一圆与$\odot (O_1,r_1)$外切,与$\odot (O_2,r_2)$内切,并 731 | 且半径是20mm. 732 | \end{example} 733 | 734 | \begin{figure}[htp]\centering 735 | \begin{minipage}[t]{0.48\textwidth} 736 | \centering 737 | \begin{tikzpicture}[>=latex, scale=.7] 738 | \draw (0,0) circle (2); 739 | \draw (4,-1.2) circle (1.2); 740 | \draw[thick](4,-3) circle (3); 741 | \tkzDefPoints{0/0/O_1, 4/-1.2/O_2, 4/0/C1, 4/-3/O, 1.6/-1.2/B, 0/2/A, 1/-3/C} 742 | \tkzLabelPoints[right](O_1,O_2,O) 743 | \tkzLabelPoints(B,A,C) 744 | \tkzDrawPoints(O_1,O_2,O, B,A,C) 745 | \draw[dashed](O_1)--(O)--(4,0); 746 | \tkzCompass(O_1,O) 747 | \tkzCompass(C1,O) 748 | \end{tikzpicture} 749 | \caption{} 750 | \end{minipage} 751 | \begin{minipage}[t]{0.48\textwidth} 752 | \centering 753 | \begin{tikzpicture}[>=latex, scale=.5] 754 | \draw (0,0) circle (2); 755 | \draw (4,-1.2) circle (1.2); 756 | \draw[thick](4,-3) circle (3); 757 | \tkzDefPoints{0/0/O_1, 4/-1.2/O_2, 4/-3/O} 758 | \tkzLabelPoints[right](O_1,O_2,O) 759 | \tkzDrawPoints(O_1,O_2,O) 760 | \draw[dashed](O_1)--(O)--(4,0); 761 | \end{tikzpicture} 762 | \caption{} 763 | \end{minipage} 764 | \end{figure} 765 | 766 | \begin{analyze} 767 | 画一草图(图5.12),设$\odot (O,20{\rm mm})$为要求作的 768 | 圆.由于$\odot (O,20{\rm mm})$与$\odot (O_1,r_1)$外切,并与$\odot (O_2,r_2)$内切, 769 | 所以 770 | \[\begin{split} 771 | \overline{OO_1}&=r_1+20{\rm mm}=30{\rm mm}\\ 772 | \overline{OO_2}&=20{\rm mm}-r_2=12{\rm mm} 773 | \end{split}\] 774 | 775 | 因此,$O$点是$\odot (O_1,30{\rm mm})$和$\odot (O_2,12{\rm mm})$的交点, 776 | 故$O$点可作出. 777 | \end{analyze} 778 | 779 | 作法:(图5.11). 780 | \begin{enumerate} 781 | \item 作$\odot (O_1,30{\rm mm})$和$\odot (O_2,12{\rm mm})$, 设两圆有 782 | 交点$O$; 783 | \item 以$O$为圆心,以20mm为半径作$\odot O$, 即$\odot O$为 784 | 所求作的圆. 785 | \end{enumerate} 786 | 787 | \begin{proof} 788 | (略) 789 | \end{proof} 790 | 791 | 讨论: 792 | \begin{enumerate} 793 | \item 当$(r_1+20{\rm mm})+(20{\rm mm}-r_2)\ge \overline{O_1O_2}$时, 794 | 即当$ \overline{O_1O_2}\le 42{\rm mm}$时,问题有一解. 795 | \item 当$ \overline{O_1O_2}> 42{\rm mm}$时,问题无解. 796 | \end{enumerate} 797 | 798 | 从例5.6中的图5.11, 我们可看出$\odot O_1$的 799 | $\wideparen{AB}$与$\odot O_2$的$\wideparen{BC}$ 800 | 在切点$B$处连接起来了,这种连结叫做\textbf{圆弧与圆弧的平 801 | 滑连接}. 802 | 803 | \begin{ex} 804 | \begin{enumerate} 805 | \item 已知两定点$A$、$B$, 且直线$AB$与定直线$\ell$平行,在$\ell$上求作与$A$、$B$两点直线$\ell$距离相等的点. 806 | \item 求作和两条相交直线$\ell$和$m$的距离相等,且和它们的交点的距离等于定长$d$的点. 807 | \item 求作和已知直线$\ell$的距离等于定长$d$,并且和$\ell$上的一个定点的距离等于$2d$的点. 808 | \item 求作和$\overline{AB}$的两端的距离相等,且和$\overline{AB}$的两端的连线所夹的角等于定角$\alpha$的点. 809 | \item 求作等腰三 810 | 角形,使它的底边等于定长$a$, 顶角等于定 811 | 角$\alpha$. 812 | \item 求作一圆,使它的半径等于定长$a$, 且经过一定点并和 813 | 一条定直线相切. 814 | \item 已知三角形的二边和其中一边上的高,求作三角形. 815 | \item 已知$\odot O(0,15{\rm mm})$和直线$\ell$, 并且$\ell$与$\odot O$相离,画半 816 | 径为10mm的圆,使其与$\odot O$外切并和$\ell$相切. 817 | \end{enumerate} 818 | \end{ex} 819 | 820 | \subsection{代数法作图} 821 | 解作图题时,有时也常常归结为要求作一条线段的长度 822 | 问题.这时我们可根据给出的条件,求出这条线段的代数表 823 | 达式,根据线段的代数表达式,把线段作出,使问题得到解 824 | 决.这种解作图题的方法,就叫做\textbf{代数分析法}. 825 | 826 | \begin{example} 827 | 已知:正方形$ABCD$(图5.13). 828 | 829 | 求作:一点$P$使$P\in \overline{CD}$, 且$\overline{AP}=\overline{BC}+\overline{CP}$. 830 | \end{example} 831 | 832 | \begin{figure}[htp]\centering 833 | \begin{minipage}[t]{0.48\textwidth} 834 | \centering 835 | \begin{tikzpicture}[>=latex, scale=1] 836 | \tkzDefPoints{0/0/A, 0/2/D, 2/2/C, 2/0/B, 1/2/E, 1.5/2/P} 837 | \tkzDrawPolygon(A,B,C,D) 838 | \tkzLabelPoints[left](A,D) 839 | \tkzLabelPoints[right](B,C) 840 | \tkzLabelPoints[above](E,P) 841 | \draw(A)--(P); 842 | \tkzDrawPoints(E) 843 | \end{tikzpicture} 844 | \caption{} 845 | \end{minipage} 846 | \begin{minipage}[t]{0.48\textwidth} 847 | \centering 848 | \begin{tikzpicture}[>=latex, scale=1] 849 | \tkzDefPoints{0/0/A, 0/2/D, 2/2/C, 2/0/B, 1.5/2/P} 850 | \tkzDrawPolygon(A,B,C,D) 851 | \tkzLabelPoints[left](A,D) 852 | \tkzLabelPoints[right](B,C) 853 | \tkzLabelPoints[above](P) 854 | \draw(A)--(P); 855 | \tkzDrawPoints(E) 856 | \end{tikzpicture} 857 | \caption{} 858 | \end{minipage} 859 | \end{figure} 860 | 861 | \begin{analyze} 862 | 画一草图(图5.14),已知正方形$ABCD$, 假设$P$点是所求作的点,即$P\in \overline{CD}$, 且$\overline{AP}=\overline{BC}+ 863 | \overline{CP}$, 设 864 | $\overline{CP}=x$, $\overline{BC}=a$, 故$\overline{AP}=8+x$, $\overline{DP}=\overline{DC}-\overline{CP}=\overline{BC}-\overline{CP}=a-x$, 又因$\triangle ADP$是直角三角形,所以根据 865 | 勾股定理有:$\overline{AP}^2=\overline{AD}^2+\overline{DP}^2$, 即:$(a+x)^2=a^2+(a-x)^2$. 866 | 解此方程得 867 | $x=\frac{a}{4}$;$a$为已知,$\frac{a}{4}$ 868 | 可作出,故$P$点也可作 869 | 出. 870 | \end{analyze} 871 | 872 | 作法:(图5.13). 873 | 874 | \begin{enumerate} 875 | \item 作$\overline{CD}$的中点$E$. 876 | \item 作$\overline{CE}$的中点$P$, $P$点即为所求作的点. 877 | \end{enumerate} 878 | 879 | \begin{proof} 880 | 由作图知$\overline{CP}=\frac{a}{4}$, 881 | 882 | $\therefore\quad \overline{DP}=\overline{CD}-\overline{CP}=\frac{3}{4}a$ 883 | \[\overline{AP}=\sqrt{\overline{AD}^2+\overline{DP}^2}=\sqrt{a^2+\left(\frac{3}{4}a\right)^2}=\frac{5}{4}a=a+\frac{a}{4}\] 884 | \[\overline{AP}=\overline{BC}+\overline{CP}\] 885 | \end{proof} 886 | 887 | 888 | 讨论:由分析知,$\overline{DP}:\overline{PC}=3:1$, 因为分点$P$是唯一的,所以此题只有一解. 889 | 890 | 891 | \begin{example} 892 | 在已知线段上求作一点,分已知线段为两部分, 893 | 使其中一部分是全线段和另一部分的比例中项. 894 | 895 | 已知:$\overline{AB}$(图5.15). 896 | 897 | 求作:$\overline{AB}$的内分点$G$, 使$\overline{AG}^2=\overline{AB}\cdot \overline{GB}$. 898 | \end{example} 899 | 900 | \begin{figure}[htp]\centering 901 | \begin{minipage}[t]{0.48\textwidth} 902 | \centering 903 | \begin{tikzpicture}[>=latex, scale=.8] 904 | \draw(0,0)node[left]{$A$}--(0.618*4,0)node[below]{$G$}--(4,0)node[right]{$B$}; 905 | \tkzDrawPoint(0,0) 906 | \tkzDrawPoint(0.618*4,0) 907 | \tkzDrawPoint(4,0) 908 | \tkzDefPoints{4/2/D, 4/0/B, 0/0/A, 2.472/0/G} 909 | \tkzDefPoint(26.57: 2.472){E} 910 | \tkzDrawArc[delta=10](D,E)(B) 911 | \tkzDrawArc[delta=10](A,G)(E) 912 | \tkzDrawSegments(A,D D,B) 913 | \tkzLabelPoints[above](D,E) 914 | 915 | \end{tikzpicture} 916 | \caption{} 917 | \end{minipage} 918 | \begin{minipage}[t]{0.48\textwidth} 919 | \centering 920 | \begin{tikzpicture}[>=latex, scale=1] 921 | \draw(0,0)node[left]{$A$}--node[above]{$x$}(0.618*4,0)node[below]{$G$}--(4,0)node[right]{$B$}; 922 | \tkzDrawPoint(0,0) 923 | \tkzDrawPoint(0.618*4,0) 924 | \tkzDrawPoint(4,0) 925 | \end{tikzpicture} 926 | \caption{} 927 | \end{minipage} 928 | \end{figure} 929 | 930 | 931 | 932 | \begin{analyze} 933 | 画一草图(图5.16),已知$\overline{AB}$, 假定$G$为所求 934 | 之点,设$\overline{AG}=x$, 则 935 | $\overline{BG}=a-x$, 936 | $x$满足方程 937 | \[ x^2=a(a-x)\] 938 | 即:$x^2+ax-a^2=0$. 解此方程得: 939 | \[x_1=\frac{-a+\sqrt{4a^2+a^2}}{2},\qquad x_2=\frac{-a-\sqrt{4a^2+a^2}}{2}\quad \text{舍去}\] 940 | $\therefore\quad x=\frac{-a+\sqrt{4a^2+a^2}}{2}=\sqrt{a^2+\left(\frac{a}{2}\right)^2}-\frac{a}{2}$ 941 | 942 | 此线段$x$可作,故$G$点也可作出来. 943 | \end{analyze} 944 | 945 | 作法:(图5.15). 946 | \begin{enumerate} 947 | \item 作$\overline{ED}\bot\overline{AB}$, 使$\overline{BD}=\frac{1}{2}\overline{AB}$ 948 | \item 在$\overline{DA}$上截取$\overline{DE}=\overline{DB}$,\item 在$\overline{AB}$上截取$\overline{AG}=\overline{AE}$, $G$点即为所求作的点. 949 | \end{enumerate} 950 | 951 | \begin{proof} 952 | 设$\overline{AB}=a$, 由作法有, 953 | \[\begin{split} 954 | \overline{AG}&=\overline{AE}=\overline{AD}-\overline{ED}\\ 955 | &=\sqrt{\overline{AB}^2+\overline{BD}^2 }-\overline{BD}=\sqrt{a^2+\left(\frac{a}{2}\right)^2}-\frac{a}{2}\\ 956 | \overline{GB}&=\overline{AB}-\overline{AG}=a-\left(\sqrt{a^2+\left(\frac{a}{2}\right)^2}-\frac{a}{2}\right)\\ 957 | &=\frac{3}{2}a-\sqrt{a^2+\left(\frac{a}{2}\right)^2} 958 | \end{split}\] 959 | $\therefore\quad \overline{AB}\cdot \overline{GB}=a\left(\frac{3}{2}a-\sqrt{a^2+\left(\frac{a}{2}\right)^2}\right)$ 960 | 961 | 由于: 962 | \[\begin{split} 963 | \overline{AG}^2&=\left(\sqrt{a^2+\left(\frac{a}{2}\right)^2}-\frac{a}{2}\right)^2\\ 964 | &=a^2+\left(\frac{a}{2}\right)^2-2\x\frac{a}{2}\x \sqrt{a^2+\left(\frac{a}{2}\right)^2}+\left(\frac{a}{2}\right)^2\\ 965 | &=\frac{3}{2}a^2-a\sqrt{a^2+\left(\frac{a}{2}\right)^2}\\ 966 | &=a\left(\frac{3}{2}a-\sqrt{a^2+\left(\frac{a}{2}\right)^2}\right) 967 | \end{split} 968 | \] 969 | $\therefore\quad \overline{AG}^2=\overline{AB}\cdot \overline{BG}$ 970 | 971 | 故$G$点为所求之点. 972 | \end{proof} 973 | 974 | 讨论:由分析可知,满足条件的$\overline{AG}$总有一个,所以$G$ 975 | 点总能作出一个,故此题有一解也只有一解. 976 | 977 | 由于 978 | \[\overline{AG}=\sqrt{a^2+\left(\frac{a}{2}\right)^2}-\frac{a}{2}=\frac{\sqrt{5}}{2}a-\frac{1}{2}a=\frac{\sqrt{5}-1}{2}a\approx 0.618 a\] 979 | 所以$\overline{AG}$是$\overline{AB}$被$G$点分成的两段中,较长的一段,这种作图通常叫做分已知线段成“\textbf{中 980 | 外比}”,又叫做\textbf{黄金分割}. 981 | 982 | \begin{example} 983 | 在已知圆中,作内接正十边形. 984 | 985 | 已知:$\odot(O,r)$(图5.17). 986 | 987 | 求作:$\odot O$的内接正十边形. 988 | \end{example} 989 | 990 | \begin{figure}[htp]\centering 991 | \begin{minipage}[t]{0.48\textwidth} 992 | \centering 993 | \begin{tikzpicture}[>=latex, scale=1] 994 | \draw (0,0) circle (2.5); 995 | \tkzDefPoint(0,0){O} 996 | \foreach \x/\xtext in {0/D,1/E,2/F,3/G,4/H,5/I,6/J,7/A,8/B,9/C} 997 | { 998 | \tkzDefPoint(\x*36:2.5){\xtext} 999 | \tkzDrawPoint(\xtext) 1000 | \tkzAutoLabelPoints[center=O](\xtext) 1001 | } 1002 | \tkzDrawPolygon(A,B,C,D,E,F,G,H,I,J) 1003 | \draw(O)--(A); 1004 | \tkzDefPoint(-108:1.545){C'} 1005 | \tkzLabelPoints[left](O, C') \tkzDrawPoints(O, C') 1006 | \tkzCompass[delta=10](A,B) 1007 | \tkzCompass[delta=10](B,C) 1008 | \tkzCompass[delta=10](C,D) 1009 | \tkzCompass[delta=10](D,E) 1010 | \tkzCompass[delta=10](E,F) 1011 | \tkzCompass[delta=10](F,G) 1012 | \tkzCompass[delta=10](G,H) 1013 | \tkzCompass[delta=10](H,I) 1014 | \tkzCompass[delta=10](I,J) 1015 | \tkzCompass[delta=10](J,A) 1016 | \end{tikzpicture} 1017 | \caption{} 1018 | \end{minipage} 1019 | \begin{minipage}[t]{0.48\textwidth} 1020 | \centering 1021 | \begin{tikzpicture}[>=latex, scale=.8] 1022 | \draw (0,0) circle (2.5); 1023 | \tkzDefPoint(0,0){O} 1024 | \tkzDefPoint(7*36:2.5){A} 1025 | \tkzDrawPoint(A) 1026 | \tkzAutoLabelPoints[center=O](A) 1027 | \tkzDefPoint(8*36:2.5){B} 1028 | \tkzDrawPoint(B) 1029 | \tkzAutoLabelPoints[center=O](B) 1030 | \draw(O)--(A)--(B); 1031 | \tkzDefPoint(-108:1.545){C'} 1032 | \tkzLabelPoints[left](O, C') \tkzDrawPoints(O, C') 1033 | \draw(C')--(B)--(O); 1034 | \end{tikzpicture} 1035 | \caption{} 1036 | \end{minipage} 1037 | \end{figure} 1038 | 1039 | 1040 | 1041 | 1042 | 1043 | 1044 | 1045 | \begin{analyze} 1046 | 画一草图(图5.18),设$\overline{AB}$是$\odot O$的内接正十 1047 | 边形的一边,则 1048 | \[\angle AOB=36^{\circ},\qquad \angle OAB=\angle OBA=72^{\circ}\] 1049 | 作$\angle OBA$的平分线交$\overline{OA}$于$C'$点,则 1050 | \[\angle OBC'=\angle ABC'=\frac{1}{2}\angle OBA=36^{\circ}\] 1051 | 又知$\angle BC'A=180^{\circ}-\angle ABC'-\angle OAB$, 1052 | 1053 | $\therefore\quad \angle BC'A=72^{\circ},\quad 1054 | \overline{OC'}=\overline{BC'}=\overline{AB}$, 且 1055 | $$\triangle OAE\backsim \triangle BAC',\quad \overline{OA}:\overline{AB}=\overline{AB}:\overline{AC'}$$ 1056 | 1057 | 即:$\overline{AB}^2=\overline{OA}\cdot \overline{AC'}$ 1058 | 1059 | $\therefore\quad \overline{OC'}^2=\overline{OA}\cdot \overline{AC'}$ 1060 | \end{analyze} 1061 | 1062 | 这个结果告诉我们,$C'$点恰好把半径$\overline{OA}$分成中外比且 1063 | $\overline{OC'}$是较长的一段.故应用黄金分割法由已知圆的半径作出 1064 | 正十边形的边长,从而圆内接正十边形可以作出来. 1065 | 1066 | 作法 (图5.17). 1067 | \begin{enumerate} 1068 | \item 作半径$\overline{OA}$. 1069 | \item 作$C'$点分$\overline{OA}$成中外比,使$\overline{OC'}$为较长的一段, 1070 | \item 以$A$为起点,顺次作$\odot O$的弦$\overline{AB}$、$\overline{BC}$、$\overline{CD}$、 1071 | $\overline{DE}$、$\overline{EF}$、$\overline{FG}$、$\overline{GH}$、$\overline{HI}$、$\overline{IJ}$、$\overline{JA}$, 且使它们都 1072 | 等于$\overline{OC'}$, 则$ABCDEFGHIJ$为所求作的圆内接正十边形. 1073 | \end{enumerate} 1074 | 1075 | 1076 | 1077 | \begin{proof} 1078 | (略) 1079 | \end{proof} 1080 | 1081 | 讨论:由分析可知一个圆内接正十边形的边长等于一个 1082 | 定值,所以此问题有一解. 1083 | 1084 | 我们把圆十等分后,把相 1085 | 间的五个分点用弦顺次连结, 1086 | 就可作出圆内接正五边形,把 1087 | 正五边形的五条对角线都作出 1088 | 来,然后去掉各边剩下的图形 1089 | 就是正五角星了(图5.19). 1090 | 1091 | \begin{figure}[htp] 1092 | \centering 1093 | \begin{tikzpicture}[scale=.8] 1094 | \draw (0,0) circle (2.5); 1095 | \tkzDefPoint(0,0){O} 1096 | \foreach \x/\xtext in {0/A,1/B,2/C,3/D,4/E} 1097 | { 1098 | \tkzDefPoint(18+\x*72:2.5){\xtext} 1099 | \tkzDrawPoint(\xtext) 1100 | % \tkzAutoLabelPoints[center=O](\xtext) 1101 | } 1102 | \tkzDrawPolygon[dashed](A,B,C,D,E) 1103 | \tkzDrawSegments(A,C C,E B,D B,E A,D) 1104 | 1105 | \foreach \x/\xtext in {0/A1,1/B1,2/C1,3/D1,4/E1} 1106 | { 1107 | \tkzDefPoint(54+\x*72:2.5){\xtext} 1108 | % \tkzAutoLabelPoints[center=O](\xtext) 1109 | } 1110 | \tkzCompass[delta=10](A,A1) 1111 | \tkzCompass[delta=10](A1,B) 1112 | \tkzCompass[delta=10](B,B1) 1113 | \tkzCompass[delta=10](B1,C) 1114 | \tkzCompass[delta=10](C,C1) 1115 | \tkzCompass[delta=10](C1,D) 1116 | \tkzCompass[delta=10](D,D1) 1117 | \tkzCompass[delta=10](D1,E) 1118 | \tkzCompass[delta=10](E,E1) 1119 | \tkzCompass[delta=10](E1,A) 1120 | 1121 | \end{tikzpicture} 1122 | \caption{} 1123 | \end{figure} 1124 | 1125 | 1126 | 通过以上三例,我们看出,用代数法解作图题的一般步 1127 | 骤: 1128 | \begin{itemize} 1129 | \item 首先要分析解决这个问题需要作出哪条线段,并用$x$ 1130 | 表示; 1131 | \item 其次依照题中所给的条件和图形的性质,列出关于$x$ 1132 | 的方程; 1133 | \item 第三步,解这个方程(不合题意的根舍去); 1134 | \item 第四 1135 | 步,依照方程的根的表示式,作出未知线段$x$; 1136 | \item 第五步,完 1137 | 成作图.即得所求作的图形. 1138 | \end{itemize} 1139 | 1140 | 1141 | 1142 | \begin{ex} 1143 | \begin{enumerate} 1144 | \item 已知线段$a$、$b$ ($a>b$), 求作下列线段$x$ 1145 | \begin{multicols}{2} 1146 | \begin{enumerate} 1147 | \item $x=\frac{ab}{a+b}$ 1148 | \item $x=\sqrt{4a^2+b^2}$ 1149 | \item $x=\sqrt{a^2-\frac{b^2}{4}}$ 1150 | \item $x=\sqrt{a^2+3b^2}-\frac{b}{2}$ 1151 | \end{enumerate} 1152 | \end{multicols} 1153 | \item 求作已知三角形的相似形,使它的面积等于已知三角形 1154 | 面积的三分之二. 1155 | \item 从圆外一定点求作圆的一条割线,使它的圆外部分同圆 1156 | 内部分相等. 1157 | \item 求作一正方形,使它同已知长方形等积. 1158 | \item 在已知梯形中,求作底的平行线,平分已知梯形的面积. 1159 | \item 作一圆的内接正五边形(只用尺、规). 1160 | \item 经过圆内一点作一条弦,使这个点是这条弦的一个三等 1161 | 分点. 1162 | \end{enumerate} 1163 | \end{ex} 1164 | 1165 | 1166 | \section*{习题5.2} 1167 | \addcontentsline{toc}{subsection}{习题5.2} 1168 | \begin{enumerate} 1169 | \item 已知线段$a$、$b$、$c$且$a>b$, 求作线段$x=\sqrt{a^2-b^2}+c$. 1170 | \item 已知五条线段$a$、$b$、$c$、$d$、$e$, 求作线段$x=\frac{abc}{de}$. 1171 | \item 已知线段$a$, 求作一条线段$x=\frac{\sqrt{5}-1}{2}a$ 1172 | \item 已知线段$a$、$b$, 且$a>b$, 求作$a+b$和$a-b$的比例中 1173 | 项. 1174 | \item 求作与一个已知圆相切于已知点,且经过已知圆外的一 1175 | 个定点的圆. 1176 | \item 已知三角形的一边长是3cm, 这边上的中线长4cm, 这边 1177 | 的对角是$65^{\circ}$, 求作这个三角形. 1178 | \item 已知一边和这边上的中线及另一边上的高线,求作这个 1179 | 三角形. 1180 | \item 求作一个三角形,使它同已知三角形等积,又同另一个 1181 | 已知的三角形相似. 1182 | \item 求作直径是$d$的圆内接矩形,使它的面积等于每边是$a$ 1183 | 的已知正方形面积(提示:设矩形的长、宽各为$x$、$y$, 1184 | 列出含有$x$、$y$的方程组解之). 1185 | \item 从已知圆外的一个已知点,作圆的割线,使它在圆外的 1186 | 部分与圆内的部分的比是$1:2$. 1187 | \item 求作过两定点,且在一条定直线上截取定长弦的圆 1188 | (提示:利用切割线定理). 1189 | \end{enumerate} 1190 | 1191 | \section*{复习题五}\addcontentsline{toc}{section}{复习题五} 1192 | 1193 | \begin{enumerate} 1194 | \item 连结一定直线上的点和线外一定点的线段,求这线段中 1195 | 点的轨迹. 1196 | \item 求具有公共底边,且这边上的高相等的三角形顶点的轨 1197 | 迹. 1198 | \item 三定点$A$、$B$、$C$在一直线上,且$\overline{AB}=\overline{BC}$, 求与$A$、$B$ 1199 | 两点和与$B$、$C$两点连线夹角相等点的轨迹. 1200 | \item 从一定点$A$向通过另一定点$B$的动直线引垂线,求垂足 1201 | $P$的轨迹. 1202 | \item 求到两定点$A$、$B$的距离的平方和等于$\overline{AB}^2$ 1203 | 的点的轨迹. 1204 | \item 求到一定圆引切线,切线长等于定长的点的轨迹. 1205 | \item 求对相交的两定圆有等幂的点的轨迹. 1206 | \item 求到两定点$A$、$B$平方差等于$\overline{AB}^2$的点的轨迹. 1207 | \item 已知三点$A$、$B$、$C$在一条直线上,求与$A$、$B$两点和$B$、 1208 | $ C$两点连线夹角相等的点的轨迹. 1209 | \item 求作一个圆使和两条已知平行线都相切,并且经过两平 1210 | 行线间一个已知点. 1211 | \item 已知$\odot(O_1,1.5{\rm cm})$, $\odot(O_2,1{\rm cm})$, 圆心间的距离 1212 | $\overline{O_1O_2}=4{\rm cm}$, 求作一圆,使它的半径等于1.3cm, 并且 1213 | 和$\odot O_1$与$\odot O_2$都外切. 1214 | \item 已知$\odot(O_1,1.4{\rm cm})$, $\odot(O_2,1{\rm cm})$, $\overline{O_1O_2}=3{\rm cm}$, 求 1215 | 作一个圆,使它的半径为3.2cm, 并且和$\odot O_1$与$\odot O_2$都相内切. 1216 | \item 已知线段$a$, 求作线$x=\sqrt{3a}$, $y=\sqrt{12a}$. 1217 | \item 已知线段$x$与$y$满足方程组: 1218 | \[\begin{cases} 1219 | \frac{x+y}{2}=a\\ 1220 | \sqrt{xy}=b 1221 | \end{cases}\qquad (a>b)\] 1222 | 求作线段$x$和$y$. 1223 | \item 作一个三角形具有已知的周长,并和一个已知三角形相 1224 | 似. 1225 | \item 求作$\triangle ABC$的内接正方形,使正方形的一边位于$\overline{AB}$边 1226 | 上,另外两个顶点各在$\overline{AC}$边和$\overline{BC}$边上. 1227 | \item 在已知正方形$ABCD$内作内接正方形,使内接正方形的 1228 | 四个顶点分别在正方形的各边上,且使它的边长等于定 1229 | 长$b$. 1230 | \item 把任一个三角形改为等边三角形,使它的面积不变. 1231 | \item 求作三个两两外切的圆,使它们的半径分别等于1cm, 1232 | 2cm, 3cm. 1233 | \item 作一个正五角星(只画出图形). 1234 | \item 已知$A$、$B$、$C$为定直线$\ell$上的三个定点,求作$\odot A$、 1235 | $\odot B$、$\odot C$使它们两两均相切,但不切于同一点. 1236 | \item 在已知矩形内作两个互相外切的等圆,使各切于这矩形 1237 | 一组对角的两边. 1238 | \end{enumerate} 1239 | -------------------------------------------------------------------------------- /2.tex: -------------------------------------------------------------------------------- 1 | \chapter{集合与简易逻辑} 2 | 3 | \section{集合} 4 | \subsection{集合的概念} 5 | 我们在第一册中曾用到过集合的概念,如自然数所构成 6 | 的集合$\{1,2,3,\ldots\}$, 方程$x^2=4$的解所构成的集合$\{-2,2\}$ 7 | 等等.那么,什么是集合呢?集合是数学中一个很根本的也 8 | 是很原始的概念,通常我们把一些确定的、彼此不同的“事 9 | 物”作为一个整体来考虑时,这个整体便说是一个\textbf{集合}.这 10 | 些事物叫做该集合的\textbf{元素}.例如: 11 | 某中学初二(一)班全体学生;小于100的全体质数; 12 | 一个生产队的全体社员;一个工厂的全部机器等等,都可分别构成一个集合.可见集合的概念是很简单 13 | 的. 14 | 15 | 对于集合这个概念,我们要注意以下几点: 16 | 17 | 第一,一个集合完全被它所含的元素所确定.至于集合 18 | 的元素之间是否具有某种相互关系,怎样排列,以及这些元 19 | 素所构成的集合是具有某种功效,单从集合的观点来看, 20 | 是一样的.例如,“一堆还没有组装的手表另件”和用“这 21 | 些另件组装好了的手表”是同一个集合.因为两者包含同样 22 | 的元素.因此,集合这个概念的要素是:\textbf{一个集合完全被它 23 | 所含的元素所确定}. 24 | 25 | 第二,集合是指构成集合的全体元素,而不是个别元 26 | 素.作为整体的集合和集合中的每个元素都是不同的. 27 | 28 | 例如,集合$A=\{a,b,c,d\}$, $A$代表的是字母$a$、$b$、 29 | $c$、$d$的全体,而不是代表其中的个别字母,因此,作为字母 30 | $a$、$b$、 31 | $c$、$d$的整体的集合$A$与$A$中的个别元素如$a,b,c,d$不 32 | 能混为一谈. 33 | 34 | 第三,集合中所含的元素必须是“确定”的,是可以判 35 | 断的.例如,由“比较小的实数”的全体就不能构成一个集 36 | 合,因为到底什么叫做比较小的实数,没有判断的标准.但 37 | 是“比80小的实数”是完全可以确定的,这就有了检验一个 38 | 实数是否是这个集合的元素的标准. 39 | 40 | 任一几何图形,我们可以看作由点构成的,也就是可看 41 | 作点的集合.例如: 42 | 43 | \textbf{圆}是同一平面上与一定点的距离等于定长的所有点的集 44 | 合(图2.1(1)). 45 | 46 | \textbf{圆面}是同一平面上与一定点距离小于或等于定长的所有 47 | 点的集合.(图2.1(2)). 48 | 49 | \begin{figure}[htp] 50 | \centering 51 | \begin{tikzpicture}[scale=.7] 52 | \begin{scope} 53 | \draw (0,0) circle (2); 54 | \draw[ultra thick] (0,0)node[below]{$O$}--node[left]{$r$}(45:2); 55 | \node at (0,-3){(1)圆}; 56 | \end{scope} 57 | \begin{scope}[xshift=6cm] 58 | \draw[pattern=north west lines] (0,0) circle (2); 59 | \draw[ultra thick] (0,0)node[below, fill=white]{$O$}--(45:2); 60 | \node at (60:1.5) [left, rotate=45, fill=white]{$r$}; 61 | 62 | \node at (0,-3){(2)圆面}; 63 | \end{scope} 64 | \end{tikzpicture} 65 | \caption{} 66 | \end{figure} 67 | 68 | 我们通常用大写字母$A,B,C,\ldots$等表示某一个集合, 69 | 用小写字母$a,b,c,\ldots$表示集合的元素.如果$a$是集合$A$ 70 | 的一个元素,我们就记为$a\in A$, 71 | 读作$a$属于$A$, 或说$a$是$A$中的一个元素.例如,$2\in\{2,3\}$, 72 | 表示$2$是集合$\{2,3\}$中的一个元素. 73 | 74 | 如果$a$不是集合$A$的元素,记作 75 | $a\notin A$ 76 | 读作$a$不属于$A$. 77 | 78 | 应该注意的是:几何图形中的元素“点”我们仍用大写 79 | 字母$A,B,C,\ldots$表示,这一点 80 | 请同学们务必注意,不要混 81 | 淆.如$X$点在直线$AB$上,也 82 | 可以说$X$点属于直线$AB$, 可 83 | 写成$X\in\text{直线}AB$. $Y$点不在直 84 | 线$AB$上,也可以说$Y$点不属于直线$AB$, 可写成$Y\notin\text{直线} 85 | AB$(图2.2). 86 | 87 | \begin{figure}[htp] 88 | \centering 89 | \begin{tikzpicture}[scale=1] 90 | \draw(0,0)--(6,0); 91 | \draw (1,0)[fill=black]circle (1.5pt) node[below]{$A$}; 92 | \draw (4.5,0)[fill=black]circle (1.5pt) node[below]{$B$}; 93 | \draw (4,-.5)[fill=black]circle (1.5pt) node[below]{$Y$}; 94 | \draw (3,0)[fill=black]circle (1.5pt) node[above]{$X$}; 95 | \end{tikzpicture} 96 | \caption{} 97 | \end{figure} 98 | 99 | \begin{ex} 100 | \begin{enumerate} 101 | \item 若$S$是 102 | 所有平方数的集合,试判定100至200之间哪些数 103 | 属于$S$. 104 | \item 若$B$是所有英语元音字母所构成的集合,$A$是所有英语 105 | 辅音字母所构成的集合,试判定$a,b,c,d,e$这五个字 106 | 母分别属于哪一集合,又不属于哪一集合. 107 | \end{enumerate} 108 | \end{ex} 109 | 110 | \subsection{集合的描述法} 111 | 决定一个集合的要素,就是它所含的元素,所以要描述 112 | 一个集合,也就是要描述它所含的是哪些元素.下面介绍两 113 | 种常用的集合描述法. 114 | 115 | \subsubsection{列举法} 116 | 如果一个集合$A$只含有很少几个元素,那么可以直截了 117 | 当地把这个集合含有的所有元素逐一列举出来,并用大括号 118 | $\{\quad \}$把它们括起来,这种描述法叫做\textbf{列举法}. 119 | 120 | 例如$\{0,1\}$是由0,1这两个元素所构成的集合;$\{+,-,\x,\div\}$表示由$+$、$-$、$\x$、$\div$四个运算符号所构成的 121 | 集合.用列举法描述集合时,描述方法与元素在括号内的排 122 | 列顺序无关,即$\{3,7,10\}$、$\{10,3,7\}$与$\{7,3,10\}$ 123 | 都表示同一个集合. 124 | 125 | \subsubsection{特征性质描述法} 126 | 当集合的元素稍多一些时,如小于100的质数所构成的 127 | 集合:$$\{2,3,5,7,11,13,17,19,23,29,31,37, 128 | 41,43,47,53,59,61,67,71,73,79,83,89, 129 | 97\}$$ 130 | 逐一列举已是很麻烦的了,而对于含有无穷多个元素的 131 | 集合,例如全体整数所构成的集合,逐一列举它的元素更是 132 | 不可能的,这时我们可用某集合所含的元素的“特征性质” 133 | 去描述这个集合,这种方法叫做\textbf{特征性质描述法}.如: 134 | 135 | \begin{enumerate} 136 | \item 集合元素为 137 | $\pm 2,\pm 4,\pm 6,\pm 8,\ldots,\pm 2n\ldots$的集 138 | 合,可描述为\{偶数}或\{能被2整除的数}. 139 | \item 集合元素为$\pm 1,\pm 3,\pm 5,\pm 7,\ldots,\pm (2n+1)\ldots$的集合,可描述为\{奇数\}或\{被2除余1的数\}. 140 | \item $\{-\sqrt{2},\sqrt{2}\}$,可描述为$\{\text{平方为2的数}\}$. 141 | \item 圆面上不在圆上的点叫做圆内的点.在平面$P$上以$O$为 142 | 圆心,5厘米长为半径的圆内的点所成的集合,可描述为\{在 143 | 平面$P$上和点$O$的距离小于5厘米的点\}. 144 | \end{enumerate} 145 | 146 | 集合的特征性质描述法,常常采用下面更一般的形式: 147 | \[A=\{x|\alpha\}\] 148 | 其中$x$表示集合$A$的任一元素,$x|\alpha$表示元素$x$具有特征性 149 | 质$\alpha$, 而$A=\{x|\alpha\}$则表示由所有具有性质$\alpha$的元素所构 150 | 成的集合$A$. 这样一来,上述各例又可表示如下: 151 | \begin{enumerate} 152 | \item $A=\{x|x\text{能被2整除}\}$ 153 | \item $B=\{x|x\text{被2除余1}\}$ 154 | \item $C= \{x|x^2=2\}$ 155 | \item $D=\{X|\overline{OX}<5{\rm cm},\; \text{且$O$是平面$P$上定点,$X\in$平面$P$} \}$ 156 | \end{enumerate} 157 | 158 | 有时候“任一元素$x$”也可用某种形式写出来,例如上面的集合$A$、$B$可写为: 159 | \[\begin{split} 160 | A&=\{2n|n\text{为任意整数}\}=\{2n|n\in\mathbb{Z}\}\\ 161 | B&=\{2n+1|n\text{为任意整数}\}=\{2n+1|n\in\mathbb{Z}\} 162 | \end{split}\] 163 | 164 | \begin{ex} 165 | \begin{enumerate} 166 | \item 用列举法表示下列集合: 167 | \begin{enumerate} 168 | \item 头五个质数的全体构成的集合. 169 | \item 12的所有因数构成的集合. 170 | \item 自然数里头五个平方数的全体构成的集合. 171 | \item 20与30间的奇数的全体构成的集合. 172 | \item 小于20的全体偶数构成的集合$P$. 173 | \end{enumerate} 174 | 175 | \item 用符号“$\in$”,“$\notin$”表示$b$, $c$, $d$与集合的关系. 176 | \begin{enumerate} 177 | \item $A=\{x|x\text{是15的因数}\}$,$b=5$, $c=15$, $d=12$; 178 | \item $O=\{x|x\text{是小于16的质数}\}$,$b=2$, $c=3$, $d=7$. 179 | \end{enumerate} 180 | 181 | \item $C$是平面$P$上以$O$为圆心,半径为3cm的圆周上的所有点组 182 | 成的集合,$X$、$Y$、$Z$是平面$P$上的三个点,且$\overline{OX}=5{\rm cm}$, $\overline{OY}=2{\rm cm}$, $\overline{OZ}=3{\rm cm}$, 试用符号“$\in$”和“$\notin$”表示$X$, 183 | $Y$, $Z$与集合$C$的关系. 184 | 185 | \item 试用特征性质描述法描述下列集合. 186 | \begin{enumerate} 187 | \item 一元二次方程$x^2+2x-3=0$的两个根所构成的集合. 188 | \item 所有加7就大于15的实数所构成的集合. 189 | \item 所有大于或等于3而小于5的实数的集合. 190 | \end{enumerate} 191 | 192 | \item 试用特征性质描述第1题中的五个集合. 193 | 194 | \item 把下列集合用列举法描述出来: 195 | \begin{enumerate} 196 | \item $A=\{x|x\text{是整数且}|x|<5\}$ 197 | \item $B=\{x|x\text{是英语中的元音字母}\}$ 198 | \item $C=\{x|x\text{是整数且}1=latex, scale=.9] 218 | \draw (0,0) circle (.6); 219 | \draw (0,-.5) circle (1.3); 220 | \node at (0,0){$A$}; 221 | \node at (-.7,-1){$B$}; 222 | \end{tikzpicture} 223 | \caption{} 224 | \end{minipage} 225 | \begin{minipage}[t]{0.48\textwidth} 226 | \centering 227 | \begin{tikzpicture}[>=latex, scale=.9] 228 | \draw[<->](-2,1)--(2,1); 229 | \draw (-.8,1) [fill=black]circle (1.5pt)node[below]{$A$}; 230 | \draw (.8,1) [fill=black]circle (1.5pt)node[below]{$B$}; 231 | \end{tikzpicture} 232 | \caption{} 233 | \end{minipage} 234 | \end{figure} 235 | 236 | 在图2.4中,$\overline{AB}$上的点都是直线$AB$上的点,但是直线$AB$上的点却还有很多不属于$\overline{AB}$, 所以$\overline{AB}$是直线$AB$的真子集,记作: 237 | \[\overline{AB}\subset \text{直线}AB\] 238 | 239 | 这里我们要提醒同学们注意区分“属于”关系和“含于”关系.“属于”关系是集合的元素与集合本身的关系,但“含于”关系却是集合与集合之间的关系. 240 | 241 | 例如集合$A=\{3, 4, 5\},\; B=\{3, 4, 5, 6, 7\}$, 对于元素4来说,它和$A$或$B$的关系是“属于”关系,即$4\in A$, $4\in B$; 对于集合$A$与$B$来说,它们的关系却是“含于”关系,即$A\subset B$. 242 | 243 | 如果两个集合$A$、$B$是由共同的元素所构成的,我们称它们为相等的集合,记作$A=B$. 例如,$\{x,y,z\}=\{y,x, z\}$, $\{+, -, \x, \div \}=\{ \x, -,+,\div\}$等. 244 | 245 | 如果两个集合$A$、$B$, $A\subseteq B$且$B\subseteq A$, 这就是说$A$的每个元素都是$B$的元素,而$B$的每个元素也都是$A$的元素,显然这两个集合含有相同的元素,则$A=B$. 例如,$A=\{\text{偶数}\}$,$B=\{2n|n\in\mathbb{Z}\}$, 则$A=B$. 事实上$A$, $B$两个集合 246 | 就是同一个集合的两种不同描述法. 247 | 248 | 如果有三个集合$A$、$B$、$C$, $A\subseteq B$且$B\subseteq C$, 那么显然有$A\subseteq C$. 这就是说集合的含于关系具有传递性. 249 | 250 | \subsubsection{集合的运算} 251 | 252 | \paragraph{交集} 253 | 由集合$A$与集合$B$的公共元素所成的集合叫做集合$A$与集合$B$的交集(或交).记为: 254 | $A\cap B$,读为$A$交$B$. 255 | \begin{figure}[htp]\centering 256 | \begin{minipage}[t]{0.48\textwidth} 257 | \centering 258 | \begin{tikzpicture}[>=latex, scale=1] 259 | \draw (0,0)node{$A$} circle(.75); 260 | \draw (1.3,0)node{$B$} circle (1); 261 | \draw[->](.5,-1.3)node[below]{$A\cap B$}--(.5,-.7); 262 | \clip {(0,0) circle (.75)}; 263 | \fill[pattern=north east lines] {(1.3,0) circle (1)}; 264 | 265 | \end{tikzpicture} 266 | \caption{} 267 | \end{minipage} 268 | \begin{minipage}[t]{0.48\textwidth} 269 | \centering 270 | \begin{tikzpicture}[>=latex, scale=.9] 271 | \draw (-2,.8)--(2,-.8)node[right]{$m$}; 272 | \draw (-1.5,-1)--(1.5,1)node[right]{$\ell$}; 273 | \node at (0,0)[above]{$A$}; 274 | \end{tikzpicture} 275 | \caption{} 276 | \end{minipage} 277 | \end{figure} 278 | 279 | 280 | 两个集合$A$、$B$的交集用维恩图来示意,如图2.5中的阴影部分就表示$A\cap B$. 281 | 282 | 若$A=\{ a, b, c, d \}$, $B=\{ c, d, e\}$, 则$A\cap B=\{c,d\}$. 283 | 284 | 在图2.6中,直线$\ell$与直线$m$的交集是$A$点,即$\ell \cap m=\{A\text{点}\}$. 285 | \begin{figure}[htp]\centering 286 | \begin{minipage}[t]{0.48\textwidth} 287 | \centering 288 | \begin{tikzpicture}[>=latex, scale=.9] 289 | \draw[<->](-2,1)--(2,1); 290 | \draw (-.8,1) [fill=black]circle (1.5pt)node[below]{$A$}; 291 | \draw (.8,1) [fill=black]circle (1.5pt)node[below]{$B$}; 292 | \end{tikzpicture} 293 | \caption{} 294 | \end{minipage} 295 | \begin{minipage}[t]{0.48\textwidth} 296 | \centering 297 | \begin{tikzpicture}[>=latex, scale=.9] 298 | \draw[dashed](-1,.3)--(0,0)--(-1,-.3); 299 | \draw(4,-1.2)node[right]{$B$}--(0,0)node[below]{$O$}--(4,1.2)node[right]{$A$}; 300 | \fill[pattern=north east lines](-1,.3)--(-1,1.5)--(4,1.5)--(4,-1.2)--(-1,.3); 301 | \fill[pattern=north west lines](-1,-.3)--(-1,-1.5)--(4,-1.5)--(4,1.2)--(-1,-.3); 302 | \end{tikzpicture} 303 | \caption{} 304 | \end{minipage} 305 | \end{figure} 306 | 307 | 在图2.7中,$\overline{AB}$是射线$AB$和射线$BA$的交集,即: 308 | \[\overline{AB}=\text{射线}AB\cap \text{射线}BA\] 309 | 310 | 一条直线把一个平面分成两部分,其中每一部分都叫做半平面,这条直线叫作半平面的界.图2.8中,$\angle AOB$的内部是以直线$OA$为界含有射线$OB$的半平面与以直线$OB$为界含有射线$OA$的半平面的交集(即图中的阴影部分). 311 | 312 | \begin{figure}[htp]\centering 313 | \begin{minipage}[t]{0.48\textwidth} 314 | \centering 315 | \begin{tikzpicture}[>=latex, scale=1] 316 | \fill[draw, pattern=north east lines] (0,0)node[fill=white]{$A$} circle(.75); 317 | \fill[draw, pattern=north east lines] (1.3,0)node[fill=white]{$B$} circle (1); 318 | \end{tikzpicture} 319 | \caption{} 320 | \end{minipage} 321 | \begin{minipage}[t]{0.48\textwidth} 322 | \centering 323 | \begin{tikzpicture}[>=latex, scale=.9] 324 | \draw[<->](-2,1)--(2,1); 325 | \draw (-.8,1) [fill=black]circle (1.5pt)node[below]{$A$}; 326 | \draw (.8,1) [fill=black]circle (1.5pt)node[below]{$B$}; 327 | \end{tikzpicture} 328 | \caption{} 329 | \end{minipage} 330 | \end{figure} 331 | 332 | 显然,若$A\subseteq B$, 则$A\cap B=A$, 反之,若$A\cap B=A$, 则$A\subseteq B$. 333 | 334 | \paragraph{并集} 335 | 由集合$A$的元素或集合$B$的元素合并而成的 336 | 集合叫做集合$A$与集合$B$的并集(或并)记为: 337 | $A\cup B$,读为A并B. 338 | 339 | 集合$A$与集合$B$的并集用维恩图示意,如图2.9所示,图中的阴影部分就表示$A\cup B$. 340 | 341 | 如果$A=\{1, 2, 3\}$, $B=\{3, 4, 5\}$, 那么$A\cup B= \{1, 2, 3, 4, 5\}$. 342 | 343 | \begin{rmk} 344 | 在求上述$A$、$B$的并集时,虽然$A$与$B$含有共同的元素3, 但在$A\cup B$中3只取一次. 345 | \end{rmk} 346 | 347 | 如果$A_+=\{x|x\text{是实数,且}x\ge 5\}$,$A_-=\{x|x\text{是实数,且}x\le -5\}$,$A=\{x|x^2\ge 25\}$,那么$A_+\cup A_-=A$. 348 | 349 | 在图2.10中,直线$AB$是射线$AB$和射线$BA$的并集,即 350 | \[\text{直线}AB=\text{射线}AB \cup \text{射线}BA\] 351 | 352 | \paragraph{空集} 353 | 为了使集合$A$和集合$B$的交集$A\cap B$在集合$A$与集合$B$不含有任何公共元素时仍有意义,我们自然想到:这时的$A\cap B$应是一个不含有任何元素的集合.因此,我们把这种不含任何元素的集合叫做\textbf{空集},并用符号$\emptyset$表示. 354 | 355 | 由空集的意义可知,对任何一个集合$P$, 都有$P\cap \emptyset=\emptyset$, $P\cup\emptyset=P$成立,并且空集是任何一个集合$P$的子集.即:$P\supseteq \emptyset$ 356 | 357 | 如果$A=\{\text{奇数}\}$,$B=\{\text{偶数}\}$,那么$A\cap B=\emptyset$. 358 | 359 | 在图2.11中,若$\odot O_1$和$\odot O_2$相离,则$\odot O_1\cap \odot O_2=\emptyset$ 360 | \begin{figure}[htp] 361 | \centering 362 | \begin{tikzpicture} 363 | \draw (0,0)node[below]{$O_1$} circle (1); 364 | \draw (2.5,0)node[below]{$O_2$} circle (.8); 365 | \draw (0,0) [fill=black] circle(1pt); 366 | \draw (2.5,0) [fill=black] circle(1pt); 367 | \end{tikzpicture} 368 | \caption{} 369 | \end{figure} 370 | 371 | 如果直线$a\parallel$直线$b$, 那么$a\cap b=\emptyset$. 372 | 373 | \paragraph{基集与补集} 374 | 375 | 如果我们所讨论的集合都是一个给定集合的子集,我们就称这个给定集合为\textbf{基集},通常用符号$I$表示基集. 376 | 377 | 例如,我们所讨论的集合是$\emptyset$、$\{1\}$、$\{2\}$、$\{3\}$、$\{1, 2\}$、$\{2, 3\}$、$\{1, 3\}$和$\{1, 2, 3\}$时,因为上述集合都是集合$\{1, 2, 3\}$的子集,所以我们称$\{1, 2, 3\}$为基集. 378 | 379 | 又因为平面和平面上的几何图形都是点的集合,那么平面几何所讨论的图形都可以看作是某平面上所有点的集合的 380 | 子集,这时,我们把平面叫做基集. 381 | 382 | 在代数中,当我们讨论有理数运算时,全体有理数就是基集. 383 | 384 | 如果$A\subseteq I$, 在基集$I$中所有不属于$A$的元素所构成的集 385 | 合叫做$A$的\text{补集},以符号“$A^c$”表示,读为$A$补.因此, 386 | \[A\cap A^c=\emptyset,\qquad A\cup A^c=I\] 387 | 388 | 基集在图示时,常常用一矩形所包围的点表示.图2.12中的阴影部分示意$A^c$. 389 | 390 | \begin{figure}[htp]\centering 391 | \begin{minipage}[t]{0.48\textwidth} 392 | \centering 393 | \begin{tikzpicture}[>=latex, scale=1.3] 394 | \fill[pattern=north east lines, draw](0,0) rectangle (3.5,2.5); 395 | \draw [fill=white](1.5,1)node{$A$} circle (.6); 396 | \node at (3,2)[fill=white]{$I$}; 397 | \node at (3,1)[fill=white]{$A^c$}; 398 | \end{tikzpicture} 399 | \caption{} 400 | \end{minipage} 401 | \begin{minipage}[t]{0.48\textwidth} 402 | \centering 403 | \begin{tikzpicture}[>=latex, scale=1.3] 404 | \fill[pattern=north east lines, draw](0,0) rectangle (3.5,2.5); 405 | \draw [fill=white](1.5,1)node[left]{$O$} circle (.6); 406 | \node at (1.5,.75){$B^c=A$}; 407 | \draw [->](1.5,1)--node[above]{3}+(30:.6); 408 | \node at (1.2,1.35){$A$}; 409 | \node at (3,2)[fill=white]{$P$}; 410 | \node at (2.8,1)[fill=white]{$A^c=B$}; 411 | \node at (.5,.5)[fill=white]{$B$}; 412 | \end{tikzpicture} 413 | \caption{} 414 | \end{minipage} 415 | \end{figure} 416 | 417 | 如果以实数全体为基集$I$,$A=\{\text{有理数}\}$,$B=\{\text{无理数}\}$,那么$A^c=B$, $B^c=A$, $A\cup B=I$, $A$和$B$互为补集. 418 | 419 | 如果以实数的全体为基集$I$, $A=\{x|x\ge 5\}$, $B=\{x|x<5\}$, $那么A^c=B$, $B^c=A$, $A$和$B$互为补集. 420 | 421 | 如果以平面$P$上的所有点的集合为基集$I$, $O$是$P$上一个定点,$A=\{X|OX<3{\rm cm},\; X\in P\}$, $B=\{X|OX>3{\rm cm},\; X\in P\}$, 那么$A^c=B$, $B^c=A$, $A$和$B$互为补集(图2.13). 422 | 423 | 对于基集$I$的任何子集合$A$, 都有$I\supseteq A\supseteq \emptyset$. 424 | 425 | $A\cap B=A$和$A\cap B^c=\emptyset$是同一件事的两种说法(图2.14).$A\subseteq B$和$B^c\subseteq A^c$是同一件事的两种说法(图2.15). 426 | 427 | 428 | 429 | \begin{figure}[htp]\centering 430 | \begin{minipage}[t]{0.48\textwidth} 431 | \centering 432 | \begin{tikzpicture}[>=latex, scale=1.3] 433 | \fill[pattern=north east lines, draw](0,0) rectangle (3.5,2.5); 434 | \draw [fill=white](1.8,1.2)node{$A$} circle (1); 435 | \draw [fill=white](2,1)node{$A$} circle (.5); 436 | \node at (1.2,1.4){$B$}; 437 | \node at (3.1,.25)[fill=white]{$B^c$}; 438 | \node at (.5,2.2)[fill=white]{$I$}; 439 | 440 | \end{tikzpicture} 441 | \caption{} 442 | \end{minipage} 443 | \begin{minipage}[t]{0.48\textwidth} 444 | \centering 445 | \begin{tikzpicture}[>=latex, scale=1.3] 446 | \fill[pattern=crosshatch, draw](0,0) rectangle (3.5,2.5); 447 | \fill [white](1.8,1.2) circle (1); 448 | \draw [pattern=north east lines](1.8,1.2) circle (1); 449 | \draw [fill=white](2,1)node{$A$} circle (.5); 450 | \node at (3.1,.25)[fill=white]{$B^c$}; 451 | \node at (.5,2.2)[fill=white]{$I$}; 452 | \node at (1.2,1.4)[fill=white]{$B$}; 453 | \node at (2,2.2)[fill=white]{$A^c$}; 454 | \end{tikzpicture} 455 | \caption{} 456 | \end{minipage} 457 | \end{figure} 458 | 459 | \section*{习题2.1} 460 | \addcontentsline{toc}{subsection}{习题2.1} 461 | \begin{enumerate} 462 | \item 设$A=\{1, 2, 3\}$ 463 | \begin{enumerate} 464 | \item 写出一个对象,它是$A$的元素; 465 | \item 写出一个对象,它不是$A$的元素; 466 | \item 写出一个对象,它是$A$的子集; 467 | \item 写出一个对象,它不是$A$的子集; 468 | \item 存在一个既是$A$的元素又是$A$的子集的对象吗? 469 | \end{enumerate} 470 | 471 | \item 设$A=\{x|x\text{是一位的奇数}\}$,指出在下列集合中,哪个 472 | 是空集?哪个是只有一个元素的集合? 473 | \[B=\{x|x\in A,\; x>2\};\qquad C=\{x|x\in A,\;x>9\}\] 474 | \[D=\{x|x\in A,\;x\text{除以3余2}\}\] 475 | \item 指出下列各题中集合$A$与集合$B$的关系. 476 | \begin{enumerate} 477 | \item $A= \{1, 3, 5,7,9\},\qquad B=\{3,5,7\}$ 478 | \item $A= \{2, 4, 6, 8\} ,\qquad B=\{x|x\text{是偶数且}12$, $\beta :x>3$, 因为一个大于3的数也一定大于2, 所以$\beta \Rightarrow\alpha$, 若$A=\{ x|x>2\}$, $B=\{ x|x>3\}$, 则有$B\subseteq A$. 536 | 537 | 如果$\alpha :|x|>2$, $\beta :x>1$. 设$A=\{x|\; |x|>2\}$, $B=\{ x| x>1\}$, 因为$A=\{ x|\; |x|>2\}=\{x|x>2\}\cup\{x|<-2\}$, 由此,我们可以看到一个绝对值大于2 538 | 的数,并不一定大于1, 所以$A$不含于$B$,即$\alpha \Rightarrow\beta$ 不成立,反之,$\beta \Rightarrow \alpha$也不成立. 539 | 540 | 如果$\alpha \Rightarrow\beta$, 且$\beta \Rightarrow\alpha$, 我们便表示为:$\alpha\longleftrightarrow \beta$. 541 | 542 | 设$\alpha \longleftrightarrow A$, $\beta \longleftrightarrow B$, 若$\alpha \longleftrightarrow\beta$, 从集合的观点看就是$A\subseteq B$且$B\subseteq A$, 即$A=B$. 543 | 544 | 设$\alpha \longleftrightarrow A$, $\beta \longleftrightarrow B$, $\gamma\longleftrightarrow C$, 如果$\alpha \Rightarrow\beta$, 且$\beta \Rightarrow\gamma$, 由集合与性质之间的对应关系可知$A\subseteq B$, $B\subseteq C$. 再由包含关系的传递性我们就得到$A\subseteq C$. 所以$\alpha \Rightarrow\gamma$. 这就说明了推出关系的一个基本特性: 545 | 546 | 当$\alpha \Rightarrow\beta$, 且$\beta \Rightarrow\gamma$成立时,$\alpha \Rightarrow\gamma$也一定成立.这个特性叫做推出关系的\textbf{传递性}. 547 | 548 | 现在我们从性质的观点看一下集合的交集,并集,补集的意义. 549 | 550 | 设 $\alpha \longleftrightarrow A$、$\beta\longleftrightarrow B$, 不难看出: 551 | \begin{itemize} 552 | \item $A\cap B$就是具有性质$\alpha$ 且具有性质$\beta$ 的元素所构成的集 553 | 合.“具有性质$\alpha$ 且具有性质$\beta$ ”,记作$\alpha \wedge\beta$, 读作$\alpha$ 且$\beta$; 554 | \item $A\cup B$就是具有性质$\alpha$或具有性质$\beta$ 的元素所构成的集 555 | 合.“具有性质$\alpha$ 或具有性质$\beta$ ”,记作$\alpha \vee\beta$. 读作$\alpha$ 或$\beta$. 556 | \item $A^c$就是哪些不具有性质$\alpha$ 的元素所构成的集合.我 557 | 们用$\bar{\alpha}$表示“不具有性质$\alpha$”,读作非$\alpha$, 也叫$\alpha$的反性质. 558 | \end{itemize} 559 | 560 | \begin{figure}[htp] 561 | \centering 562 | \begin{tikzpicture} 563 | \draw (0,0) circle (1); 564 | \draw (1.4,0) circle (.8); 565 | \node at (0,0)[below]{$O_1$}; 566 | \node at (1.4,0)[below]{$O_2$}; 567 | \draw (0,0)[fill=black] circle (1.5pt); 568 | \draw (1.4,0)[fill=black] circle (1.5pt); 569 | \end{tikzpicture} 570 | \caption{} 571 | \end{figure} 572 | 573 | 设$A=\{X|X\text{为$\odot O_1$内的点}\}$,$B=\{X|X\text{为$\odot O_2$内的点}\}$,$\alpha$表示$\odot O_1$内的点,$\beta$表示$\odot O_2$内的点.则$A,B$与$\alpha,\beta$的关系如下(图2.16): 574 | \begin{itemize} 575 | \item $A\cap B \longleftrightarrow \alpha\wedge \beta$\quad 在$\odot O_1$且在$\odot O_2$内的点的集合. 576 | \item $A\cup B\longleftrightarrow \alpha\vee \beta$\quad 在$\odot O_1$或在$\odot O_2$内的点的集合. 577 | \item $A^c\longleftrightarrow \bar{\alpha}$\quad 不在$\odot O_1$内的点的集合. 578 | \item $B^c\longleftrightarrow \bar{\beta}$\quad 不在$\odot O_2$内的点的集合. 579 | \item $A^c\cap B^c\longleftrightarrow \bar\alpha\wedge\bar\beta $\quad 不在$\odot O_1$内且不在$\odot O_2$内的点的集合. 580 | \item $A^c\cup B^c\longleftrightarrow \bar\alpha\vee\bar\beta$\quad 不在$\odot O_1$内或不在$\odot O_2$内的点的集合. 581 | \end{itemize} 582 | 583 | 总结以上讨论,我们把性质与集合之间的关系列成下表: 584 | \begin{center} 585 | \begin{tabular}{cc|cc} 586 | \hline 587 | 集合 & 性质 &集合 &性质\\ 588 | \hline 589 | $A$ & $\alpha$ & $B$ & $\beta$\\ 590 | $A^c$ & $\bar\alpha$ & $B^c$ & $\bar\beta$\\ 591 | $A\cap B$ & $\alpha\wedge \beta$ & $A\cup B$ & $\alpha\vee\beta$\\ 592 | $A\subseteq B$ & $\alpha\Rightarrow\beta $& $A=B$&$\alpha\Leftrightarrow \beta$\\ 593 | \multicolumn{2}{c|}{$A\subseteq B$与$B\subseteq A^c$同义}& \multicolumn{2}{|c}{$\alpha\Rightarrow\beta$与$\bar\beta\Rightarrow\bar\alpha$同义}\\ 594 | \hline 595 | \end{tabular} 596 | \end{center} 597 | 598 | \subsection{基本逻辑语句} 599 | \subsubsection{命题} 600 | 凡可决定其真假的语句就叫做\textbf{命题}.例如: 601 | \begin{enumerate} 602 | \item 雪是白的.(真) 603 | \item 对顶角相等.(真) 604 | \item $2+2=5$. (假) 605 | \item 若$x>y$, 则$\frac{x+2y}{3}>\frac{x+y}{2}$.(假) 606 | \end{enumerate} 607 | 608 | 这些语句都是命题.从上面这些命题中我们可以看到,命题不一定为数学所独有,如1就不是数学命题,另外我们还看到一个命题是可以判断真假的,有的容易判断如1、2、3.有的就较难判断一些,如4是个假命题,但一眼 609 | 不容易看出来,实际上如果设$x=1$, $y=0,\;(x>y)$那么 610 | $\frac{1+2\x 0}{3}\not>\frac{1+0}{2}$.这就说明4是个假命题了.所谓“判断真假”是对事物的本质说的,并不要求现在就要,“决定”,也不要求最近的将来便可决定.例如: 611 | \begin{itemize} 612 | \item 别的星球上有生物. 613 | \item 凡大于4的偶数都是两个奇质数之和(这就是著名的哥德巴赫猜想). 614 | \end{itemize} 615 | 616 | 这些语句的真假不但现在不能决定,在最近的将来也未必“可决定”,但是我们认为从事物的本质来说,它们本身是有真假可言的,所以应该承认它们都是命题. 617 | 618 | 数学命题,经常使用“如果……,那么……”,“若……,则…….”的叙述形式,如前面提到的4便是这一类形式的 619 | 命题.这类命题写成一般形式就是: 620 | \begin{blk}{} 621 | 若$\alpha$, 则$\beta$\qquad 或\qquad 如果$\alpha$, 那么$\beta$. 622 | \end{blk} 623 | 624 | 下面我们仔细的分析它的结构和逻辑关系. 625 | 626 | 命题“若$\alpha$, 则$\beta$”是否正确,就要看$\alpha$、$\beta$之间是否具有推出关系$\alpha\Rightarrow\beta$了.也就是说,如果$\alpha$、$\beta$之间具有$\alpha\Rightarrow\beta$的关系,则“若$\alpha$, 则$\beta$”是真命题(即正确的命题),由此可见“$\alpha\Rightarrow\beta$”与“若$\alpha$, 则$\beta$”是真命题是同一关系的两种不同说法. 627 | 628 | 在命题“若$\alpha$, 则$\beta$”中,我们把$\alpha$叫做命题的\textbf{条件},$\beta$叫做命题的\textbf{结论}. 629 | 630 | 把“若$\alpha$, 则$\beta$”中的$\beta$作为条件,$\alpha$作为结论,就得到另一个命题:若$\beta$, 则$\alpha$. 这个命题叫做“若$\alpha$, 则$\beta$”的\textbf{逆命题}. 631 | 632 | 把“若$\alpha$, 则$\beta$”中的$\alpha$的反性质:非$\alpha$, $\beta$的反性质:非$\beta$, 分别作为条件和结论,又得到一个新命题:若$\bar\alpha$, 则$\bar\beta$, 这个命题叫做“若$\alpha$, 则$\beta$”的\textbf{否命题}. 633 | 634 | 把“若$\alpha$, 则$\beta$”中的$\beta$的反性质:非$\beta$, $\alpha$的反性质:非$\alpha$, 分别作为条件和结论,又可得到一个新命题:若$\bar\beta$, 则$\bar\alpha$. 这个命题叫做“若$\alpha$, 则$\beta$”的\textbf{逆否命题}. 635 | 636 | 若“$\alpha$, 则$\beta$”则叫做上述逆命题,否命题、逆否命题的\textbf{原命题}.这就是说,逆命题,否命题,逆否命题都是对原命题而说的,在四种命题中任何一个命题都可作为原命题.如: 637 | \begin{itemize} 638 | \item 原命题\quad 若$\bar\alpha$,则$\bar\beta$ 639 | \item 逆命题\quad 若$\bar\beta$,则$\alpha$ 640 | \item 否命题\quad 若$\alpha$,则$\beta$($\bar\alpha$的非是$\alpha$,$\bar\beta$的非是$\beta$) 641 | \item 逆否命题\quad 若$\beta$,则$\alpha$ 642 | \end{itemize} 643 | 644 | 我们把上述四种命题之间的关系,列成下图: 645 | \begin{center} 646 | \begin{tikzpicture}[>=latex] 647 | \node (A)[draw, rectangle]at (-3,2) {原命题:若$\alpha$,则$\beta$}; 648 | \node (B)[draw, rectangle]at (3,2) {逆命题:若$\beta$,则$\alpha$}; 649 | \node (C)[draw, rectangle]at (-3,-2){否命题:若$\bar\alpha$,则$\bar\beta$}; 650 | \node (D)[draw, rectangle]at (3,-2) {逆否命题:若$\bar\beta$,则$\bar\alpha$}; 651 | \node (E)[draw, rectangle, rounded corners=3mm]at (0,0) {互为逆否}; 652 | \draw[<->, ultra thick](A)--node[above]{互逆}(B); 653 | \draw[<->, ultra thick](C)--node[below]{互逆}(D); 654 | \draw[<->, ultra thick](A)--node[left]{互否}(C); 655 | \draw[<->, ultra thick](B)--node[right]{互否}(D); 656 | \foreach \x in {A,B,C,D} 657 | { 658 | \draw[->, ultra thick](E)--(\x); 659 | } 660 | \end{tikzpicture} 661 | \end{center} 662 | 663 | \begin{example} 664 | 写出“对顶角相等”这个命题的逆命题,否命题和逆否命题. 665 | \end{example} 666 | 667 | \begin{solution} 668 | “对顶角相等”这个命题是由两部分组成的:(1)两个角是对顶角,(2)这两个角相等.(1)是命题的条件,(2)是命题的结论.这个命题若用“若—则”连结(1)、(2)两部分,就可写成. 669 | 670 | 若两个角是对顶角,则这两个角相等.所以, 671 | \begin{itemize} 672 | \item 逆命题\quad 若两个角相等,则这两个角是对顶角. 673 | \item 否命题\quad 若两个角不是对顶角,则这两个角就不相等. 674 | \item 逆否命题\quad 若两个角不相等,则这两个角就不是对顶角. 675 | \end{itemize} 676 | \end{solution} 677 | 678 | 从这个例子,我们会发现,在这四个命题中有的正确,有的不正确,那么,这四种命题的真假之间是否存在某种关系呢?下面我们再举两个例子. 679 | 680 | \begin{example} 681 | 写出“如果一个数末位数字是0,那么它一定能被5整除.”这个命题的逆命题,否命题、逆否命题. 682 | \begin{itemize} 683 | \item 原命题\quad 如果一个数末位数字是0, 那么它一定能被5整除(真). 684 | \item 逆命题\quad 如果一个数能被5整除,那么它的末位数字一定是0(假). 685 | \item 否命题\quad 如果一个数末位数字不是0, 那么它一定 不能被5整除(假). 686 | \item 逆否命题\quad 如果一个数不能被5整除,那么它的末位数字一定不是0(真). 687 | \end{itemize} 688 | \end{example} 689 | 690 | \begin{example} 691 | 写出“若某人是中国人,则他是北京人.”这个命题的逆命题、否命题、逆否命题. 692 | 693 | \begin{itemize} 694 | \item 原命题\quad 若某人是中国人,则他是北京人(假). 695 | \item 逆命题\quad 若某人是北京人,则他是中国人(真). 696 | \item 否命题\quad 若某人不是中国人,则他不是北京人(真). 697 | \item 逆否命题\quad 若某人不是北京人,则他不是中国人(假). 698 | \end{itemize} 699 | \end{example} 700 | 701 | 从以上的例子,我们看到原命题和逆否命题同真假,逆命题和否命题同真假. 702 | 703 | 前面我们已经知道,$\alpha\Rightarrow \beta$ ($A\subseteq B$) 与$\beta\Rightarrow\alpha$ ($B^c\subseteq A^c$) 704 | 同义.这就是说,两个互为逆否的命题是同真假的,而$\alpha\Rightarrow\beta$ ($A\subseteq B$)成立并不能保证$\beta\Rightarrow \alpha$ ($B\subseteq A$) 705 | 也成立.这就是说,两个互逆命题不一定同真假. 706 | 707 | \begin{ex} 708 | \begin{enumerate} 709 | \item 试从日常生活中举出两个命题的例子,并判断它们是否正确. 710 | \item 举出两个代数内容的命题,并分别指出它们的条件和结论. 711 | \item 举出三个几何内容的命题(写成若一则形式). 712 | \item 把下列命题写成“若一则”形式,并标出条件和结论. 713 | \begin{enumerate} 714 | \item 等角的补角相等. 715 | \item 同圆或等圆的半径相等. 716 | \item 同位角相等则两直线平行. 717 | \item 三角形内角和等于180$^{\circ}$. 718 | \item 自然数必为有理数. 719 | \end{enumerate} 720 | \item 在以下各题中,用符号“$\Rightarrow$”把$\alpha$、$\beta$联系起来: 721 | \begin{enumerate} 722 | \item $\alpha$表示自然数$a$被4整除,$\beta$表示自然数$a$被2整除. 723 | \item $\alpha$表示两角相等,$\beta$表示两个角是对顶角. 724 | \end{enumerate} 725 | \item 举出反例说明下列命题是假命题. 726 | \begin{enumerate} 727 | \item 若$|a|=|b|$, 则$a=b$ 728 | \item 若$a^2=b^2$, 则$a=b$ 729 | \item 若在$\triangle ABC$与$\triangle A'B'C'$中,$\angle A=\angle A'$, $\angle B=\angle B'$, $\angle C=\angle C'$,则$\triangle ABC\cong \triangle A'B'C'$ 730 | \item 如果$(x-a)(x-b)=0$,则$x-a=0$ 731 | \item 如果$a+c=b+d$,则$a=b$, $c=d$ 732 | \end{enumerate} 733 | \item 分别写出下列命题的逆命题、否命题,逆否命题,并判断它们的真假. 734 | \begin{enumerate} 735 | \item 若$a=0$, 则$ab=0$. 736 | \item 如果$a=b$, 则$a^2=b^2$. 737 | \item 如果$|a|=|b|$, 则$a=b$. 738 | \item 如果$\triangle ABC\cong \triangle A'B'C'$, 则$\overline{AB}=\overline{A'B'}$. 739 | \item 等角的余角相等. 740 | \item 若$a\ne b$, 则$a^2\ne c^2$. 741 | \item 若$(x-a)(x-b)=0$, $x=a$. 742 | \item 如果$x_1$、$x_2$是方程$ax^2+bx+c=0$的两个根,那么$x_1+x_2=-\frac{b}{a}$, $x_1x_2=\frac{c}{a}$. 743 | \item 多项式$f(x)$除以$(x-a)$所得的余式等于$f(a)$. 744 | \end{enumerate} 745 | \end{enumerate} 746 | \end{ex} 747 | 748 | \subsubsection{充分、必要条件} 749 | 750 | 如果“若$\alpha$, 则$\beta$.”是正确的命题,这个命题与$\alpha\Rightarrow\beta$的意义就完全一致,也就是说这两种表达形式是同一逻辑关系的两种说法. 751 | 752 | \begin{blk}{} 753 | 如果$\alpha\Rightarrow\beta$, 我们就说$\alpha$是$\beta$的充分条件,$\beta$是$\alpha$的必要条件. 754 | \end{blk} 755 | 756 | 例如,如果李华是北京人,那么他是中国人.这个命题显然是个真命题.这时,我们也说,“李华是北京人”是“李华是中国人”的充分条件,而“李华是中国人”是“李华是北京人”的必要条件,这就是说由“李华是北京人”就能充分说明“李华是中国人”,但“李华是中国人”只是“李华是北京人”必定有的条件.如果李华不是中国人,更谈不上他是北京人了. 757 | 758 | 又如若$\triangle {\rm I}\cong \triangle {\rm II}$, 则$\triangle {\rm I}$与$\triangle {\rm II}$的三个角分别相等.我们知道这也是个真命题,即可由$\triangle {\rm I}\cong \triangle {\rm II}\Rightarrow \triangle {\rm I}$与$\triangle {\rm II}$的三个角分别相等.这时我们说$\triangle {\rm I}\cong \triangle {\rm II}$是$\triangle {\rm I}$与$\triangle {\rm II}$的三个角分别相等的充分条件.这就是说,若有$\triangle {\rm I}\cong\triangle {\rm II}$, II就能充分保证$\triangle {\rm I}$与$\triangle {\rm II}$的三个角分别相等.但$\triangle {\rm I}$与$\triangle {\rm II}$的三个角分别相等是$\triangle {\rm I}\cong \triangle {\rm II}$的必要条件,如果$\triangle {\rm I}$与$\triangle {\rm II}$的三个角不分别相等,则$\triangle {\rm I}$和$\triangle {\rm II}$绝不会全等. 759 | 760 | “若$\alpha$, 则$\beta$”与它的逆命题“若$\beta$, 则$\alpha$”都是 761 | 真命题,即$\alpha\Rightarrow\beta$, $\beta\Rightarrow\alpha$或$\alpha\Leftrightarrow\beta$. 这时我们说$\alpha$是$\beta$的充分、必要条件,同时$\beta$也是$\alpha$的充分、必要条件,充分、必要条件简称\textbf{充要条件}. 762 | 763 | 例如,$\triangle ABC\cong \triangle A'B'C'$且$\overline{BC}$与$\overline{B'C'}$是对应边,则$\triangle ABC\cong \triangle A'B'C'$是 764 | $\angle B=\angle B'$,$\angle C=\angle C'$, 765 | $\overline{BC}=\overline{B'C'}$的充分必要条件:$x-2=0$是$x=0$的充要条件;两圆能够重合是两个圆半径相等的充要条件. 766 | 767 | \begin{ex} 768 | \begin{enumerate} 769 | \item 下面四句话是否都是表达的同一逻辑关系? 770 | \begin{enumerate} 771 | \item $\alpha\Rightarrow\beta$; 772 | \item 若$\alpha$,则$\beta$是真命题; 773 | \item $\alpha$是$\beta$的充分条件; 774 | \item $\beta$是$\alpha$的必要条件. 775 | \end{enumerate} 776 | 777 | \item 说明$\alpha$是$\beta$的什么条件: 778 | 779 | \begin{tabular}{cll} 780 | & $\alpha$ &$\beta$\\ 781 | (a)& $a=-b$ &$a^2=b^2$\\ 782 | (b)& $a+b=c+d$ & $a=c,\; b=d$\\ 783 | (c)& $a=b$ & $a^2-2ab+b^2=0$\\ 784 | (d)& $\triangle ABC\cong \triangle A'B'C'$ & $\angle B=\angle B'$, 785 | $\overline{AB}=\overline{A'B'}$\\ 786 | (e)& $\overline{AB}$能够重合$\overline{CD}$ 787 | & $\overline{AB}$与$\overline{CD}$等长 788 | \end{tabular} 789 | \end{enumerate} 790 | \end{ex} 791 | 792 | \subsubsection{定义和基本性质} 793 | \paragraph{定义} 794 | 在数学中,我们使用的“概念”和“符号”,如果它们的意义不明确,人们可以自由理解,那就无法研究问题了.以必须对“概念”和“符号”的意义作确切的规定.这种又“概念”和“符号”的意义作规定的工作就叫做给“概念” 795 | 和“符号”下定义. 796 | 797 | 例如,“两个形状和大小完全相同的图形叫做全等形.”这就是全等形的定义. 798 | 799 | 学习每一个概念必须清楚地理解它的含义,那么“全等形”这个概念的含义是什么呢? 800 | 801 | 第一,如果两个图形的形状和大小\textbf{完全相同},那么这两个图形就是\textbf{全等形}. 802 | 803 | 第二,如果两个图形是\textbf{全等形},那么它们的形状和大小\textbf{完全相同}. 804 | 805 | 从第一、第二两个含义可以看出:“两个图形的形状和大小完全相同”这个条件是全等形的充分、必要条件.因此,一个概念的定义是这个概念的充要条件.通常我们给一个概念下定义,也就是寻求这个概念的充要条件.这个充要条件,也就是这个概念的特征性质. 806 | 807 | 在数学中,有些概念是很原始也是很根本的,很难再用别的词来说明它,也就是说,这些概念很难再给它们下定义.例如;点、直线、平面、集合等,我们仅对这些概念加以描述,以邦助大家理解. 808 | 809 | 在前面我们已经给很多概念和使用的符号下了定义,请复习和记忆下列各概念和符号的意义,以作为今后学习的依据. 810 | 811 | \paragraph{概念} 点、线段、直线、射线、平面、两点间距离、线段中点、角、角的平分线、锐角、直角、钝角、平角、周角、对顶角、两条直线互相垂直、线段的垂直平分线、点到直线的距离、两角的和与差、两角互余、两角互补、同位角、两条直线互相平行、全等形、相似形、相似比、圆. 812 | 813 | \paragraph{集合} 子集、交集、并集、补集、基集、空集、命题、原命题、逆命题、否命题、逆否命题、充分条件、必要条件、充要条件. 814 | 815 | \paragraph{符号} $\angle$, $\triangle$, $\odot$, $\bot$, $\parallel$, $\cong$, $\backsim$, $\in$, $\notin$, $\subseteq$, $=$, $\cup$, $\cap$, ${\rm I}$, $\emptyset$, $\wedge$, $\vee$, $\Rightarrow$, $\Leftrightarrow$. 816 | 817 | \paragraph{基本性质} 818 | 819 | 我们要论证一个命题的正确性,必须先由实践确立一系列命题的正确性,再以它们为根据作为出发点,才能设法论证其他命题的正确性,作为论证的原始依据的正确命题,我们称它为\textbf{基本性质(或公理)}.基本性质的正确性不是通过推理论证而得出来的,而是通过实践检验确定无疑地归纳出来的. 820 | 821 | 以下几个命题今后作为\textbf{基本性质}使用. 822 | 823 | \begin{enumerate} 824 | \item 两点确定一条直线. 825 | \item 过已知直线外一点有且只有一条直线和已知直线平行 826 | (平行公理). 827 | \item 一个图形可以移动到任何其它位置而不改变它的形状 828 | 和大小(移形公理). 829 | \item 任何一个图形都可以翻转而不改变它的形状和大小 830 | (翻转公理). 831 | \item 能够重合的两个图形具有相等的量. 832 | \item 三条边对应相等的两个三角形全等(SSS). 833 | \end{enumerate} 834 | 835 | 在代数中学过的等式和不等式的基本性质也可以作为今 836 | 后的推理依据. 837 | \begin{blk}{等量的基本性质} 838 | \begin{enumerate} 839 | \item 等量加等量它们的和相等. 840 | \item 等量减等量它们的差相等. 841 | \item 等量的同倍量相等. 842 | \item 等量的同分量相等. 843 | \item 在一个等式或不等式中,一个量也可用它的等量代 844 | 换(简称等量代换). 845 | \end{enumerate} 846 | \end{blk} 847 | 848 | \begin{blk}{不等量的基本性质} 849 | 设$a$、$b$、$c$、$d$是任意实数. 850 | \begin{enumerate} 851 | \item 如果$a>b$, 那么$a+c>b+c$. 852 | \item 如果$a>b, c>0$, 那么$ac>bc$, $\frac{a}{c}>\frac{b}{c}$. 853 | \item 如果$a>b$, 并且$c>d$, 那么$a+c>b+d$. 854 | \item 如果$a>b$, 并且$cb-d$. 855 | \item 如果$a>b$, 并且$b>c$, 那么$a>c$. 856 | \item 全量大于它的任意一部分. 857 | \end{enumerate} 858 | \end{blk} 859 | 860 | \subsubsection{定理和证明} 861 | 我们要说明.“若$\alpha$ 则$\beta$”是真命题时,以什么方式来 862 | 推证呢?最常用的基本推理格式就是推出关系的传递性. 863 | 即: 864 | \begin{align*} 865 | \text{如果}& \quad \alpha \Rightarrow \beta \tag{1}\\ 866 | & \quad \beta \Rightarrow \gamma \text{成立} \tag{2}\\ 867 | \text{那么}& \quad \alpha \Rightarrow \gamma \text{成立}\tag{3} 868 | \end{align*} 869 | 870 | 例如, 871 | \begin{align*} 872 | \text{若}& \quad \text{$\angle 1$和$\angle 2$是对顶角} \tag{1}\\ 873 | & \quad \text{对顶角相等} \tag{2}\\ 874 | \text{则}& \quad \angle 1=\angle 2 \tag{3} 875 | \end{align*} 876 | 877 | 在几何论证中,使用上述推理格式,在书写上常常由(1) 878 | 直接得到(3), 而把(2)作为由(1)推出(3)的理由写在(3)后边的括 879 | 号里.因此,上例又可以写为: 880 | \[\begin{split} 881 | \because&\quad \text{$\angle 1$和$\angle 2$是对顶角}\\ 882 | \therefore&\quad \angle 1=\angle 2 \qquad \text{(对顶角相等)} 883 | \end{split}\] 884 | 885 | 在数学中要证明一个命题的正确性,往往要连续使用若 886 | 干次上述基本推理格式.为此,我们重新回忆一下“对顶角 887 | 相等”的证明过程. 888 | 889 | 已知:$\angle 1$是$\angle 2$的对顶角(如图2.17). 890 | 891 | 求证:$\angle 1=\angle 2$ 892 | 893 | \begin{figure}[htp] 894 | \centering 895 | \begin{tikzpicture} 896 | \draw (-3,1)--(3,-1); 897 | \draw(-3,-1)--(3,1); 898 | \draw (-18.4:.5) arc (-18.4:18.4:.5); 899 | \node at (1,0){2};\node at (-1,0){1};\node at (0,.5){3}; 900 | \draw (180-18.4:.5) arc (180-18.4:180+18.4:.5); 901 | \draw (18.4:.25) arc (18.4:180-18.4:.25); 902 | \end{tikzpicture} 903 | \caption{} 904 | \end{figure} 905 | 906 | 907 | \begin{proof} 908 | \begin{center} 909 | \begin{tikzpicture}[yscale=1.5, >=latex] 910 | \node (A)[draw, rectangle] at (0,0) {$\angle 1$是$\angle 2$的对顶角}; 911 | \node (B)[draw, rectangle] at (0,-1) {$\angle 1$的两边分别是$\angle 2$的反向延长线}; 912 | \node (C)[draw, rectangle] at (0,-2) {$\angle 1$与$\angle 3$; $\angle 2$与$\angle 3$互为补角}; 913 | \node (D)[draw, rectangle] at (0,-3) {$\angle 1+\angle 3=180^{\circ}\qquad \angle 2+\angle 3=180^{\circ}$}; 914 | \node (E)[draw, rectangle] at (0,-4) {$\angle 1+\angle 3=\angle 2+\angle 3$}; 915 | \node (F)[draw, rectangle] at (0,-5) {$\angle 1=\angle 2$}; 916 | 917 | \draw[very thick, ->](A)--(B);\draw[very thick, ->](B)--(C); 918 | \draw[very thick, ->](C)--(D);\draw[very thick, ->](D)--(E); 919 | \draw[very thick, ->](E)--(F); 920 | 921 | \foreach \x/\xtext in {0/\alpha,-1/\alpha_1,-2/\alpha_2,-3/\alpha_3,-4/\alpha_4,-5/\beta} 922 | { 923 | \node at (4,\x){$\xtext$}; 924 | } 925 | \foreach \x in {0,-1,...,-4} 926 | { 927 | \node at (4,\x-.5){$\Downarrow$}; 928 | } 929 | \end{tikzpicture} 930 | \end{center} 931 | 932 | \end{proof} 933 | 934 | 从上述推演过程可知,要证明$\alpha \Rightarrow \beta$, 我们先设法找出 935 | 了一连串适当的并且已知是正确的命题,如 936 | $\alpha \Rightarrow \alpha_1$, $\alpha_1\Rightarrow 937 | \alpha_2$, $\alpha_2\Rightarrow \alpha_3$, $\alpha_3\Rightarrow \alpha_4$, $\alpha_4\Rightarrow \beta$, 再由推出关系的传递 938 | 性,就可断定$\alpha \Rightarrow\beta$了.这种推演过程就叫做\textbf{证明}.经过证 939 | 明正确的命题叫做\textbf{定理}.如果一个定理的逆命题经过证明也 940 | 正确时,这个逆命题也叫做这个定理的\textbf{逆定理}. 941 | 942 | 应用已经被认为正确的命题为依据,经过推理,导出某 943 | 一命题成立,这种方法叫做\textbf{演绎推理法}(简称\textbf{演绎法}).从 944 | 下一章开始,我们就要用这种方法来研究几何学了.所以学 945 | 习几何学,不但可对图形性质有精确的了解,也可以初步体 946 | 会演绎法这个基本有力的科学方法的实际应用. 947 | 948 | 以下几个命题今后作为定理来使用. 949 | \begin{enumerate} 950 | \item 对顶角相等, 951 | \item 两个三角形如果有两边一夹角对应相等,它们就全等 952 | (SAS). 953 | \item 两个三角形如果有两角一夹边对应相等,它们就全等 954 | (ASA). 955 | \end{enumerate} 956 | 957 | \subsubsection{证明举例} 958 | 在几何学中,要证明一个命题的正确性,一定要经过以 959 | 下过程.首先要分清待证命题的条件和结论,并根据条件画 960 | 出图形,然后按图上标记的符号写出已知,求证.接着分析 961 | 思考证明的途径,最后有根据的写出证明的全过程. 962 | 963 | 但是证明一个定理,从逻辑上说只要求写出三项: 964 | \begin{multicols}{3} 965 | \begin{enumerate} 966 | \item 已知(带图); 967 | \item 求证; 968 | \item 证明. 969 | \end{enumerate} 970 | \end{multicols} 971 | 972 | 在下例中为了使同学们体会 973 | 一下证明一个定理的全过程,我们把分析命题的条件和结论 974 | 以及思考证明的途径等也都写出来了,以供同学们参考. 975 | 976 | \begin{example} 977 | 证明:同角的余角相等. 978 | 979 | 待证命题的条件是“如果两 980 | 个角是同一个角的余角”,结论 981 | 是“这两个角相等.”根据条件 982 | 画出图形,标上必要的字母,如 983 | 图2.18, 再根据图形写出已知、求证. 984 | \end{example} 985 | 986 | \begin{figure}[htp] 987 | \centering 988 | \begin{tikzpicture} 989 | \foreach \x/\xtext in {0/A, 45/B, 90/C, 135/D} 990 | { 991 | \draw(0,0)--(\x:2.5)node[right]{$\xtext$}; 992 | } 993 | \node at (0,0)[below]{$O$}; 994 | \end{tikzpicture} 995 | \caption{} 996 | \end{figure} 997 | 998 | 已知:$\angle AOB$和$\angle COD$都是$\angle BOC$的余角. 999 | 1000 | 求证:$\angle AOB=\angle COD$. 1001 | 1002 | 分析思考证明途径:要证明$\angle AOB=\angle COD$, 由于已知$\angle AOB$和$\angle COD$ 1003 | 都是$\angle BOC$的余角,回忆互为余角的定义可知:$\angle AOB+ 1004 | \angle BOC=90^{\circ}$, $\angle COD+\angle BOC=90^{\circ}$, 比较这两个等式, 1005 | 自然有$\angle AOB=\angle COD$了. 1006 | 1007 | \begin{proof} 1008 | $\because\quad \angle AOB$和$\angle COD$都是$\angle BOC$的余角(已知) 1009 | 1010 | $\therefore\quad \angle AOB+\angle BOC=90^{\circ}$, 1011 | $\angle COD+\angle BOC=90^{\circ}$ (余角定义) 1012 | 1013 | $\therefore\quad \angle AOB+\angle BOC=\angle COD+\angle BOC$ (等量代换) 1014 | 1015 | $\because\quad \angle BOC=\angle BOC$ 1016 | 1017 | $\therefore\quad \angle AOB=\angle COD$(等量减等量差相等) 1018 | \end{proof} 1019 | 1020 | \begin{example} 1021 | 证明定理:线段垂直平分线上的点到线段两端点 1022 | 的距离相等. 1023 | 1024 | 已知:直线$CD\bot\overline{AB}$且过$\overline{AB}$ 1025 | 的中点$O$, $P$是$CD$上任一点(图2.19). 1026 | 1027 | 求证:$\overline{PA}=\overline{PB}$. 1028 | \end{example} 1029 | 1030 | \begin{figure}[htp] 1031 | \centering 1032 | \begin{tikzpicture}[scale=1.5] 1033 | \draw(-1,0)node[left]{$A$}--(1,0)node[right]{$B$}--(0,1.5)node[left]{$P$}--(-1,0); 1034 | \draw(0,-.5)node[below]{$D$}--(0,2)node[right]{$C$}; 1035 | \node at (.2,0)[below]{$O$}; 1036 | \end{tikzpicture} 1037 | \caption{} 1038 | \end{figure} 1039 | 1040 | \begin{proof} 1041 | 在$\triangle PAO$与$\triangle PBO$中 1042 | 1043 | $\because\quad \overline{PO}=\overline{PO}$(公共边), 1044 | 1045 | 又$\because\quad $直线$CD\bot\overline{AB}$于$O$, $O$是$AB$的中点(已知) 1046 | 1047 | $\therefore\quad \overline{AO}=\overline{BO}$(线段中点定义) $\angle POA=\angle POB$(垂直定义). 1048 | 1049 | $\therefore\quad \triangle POA\cong \triangle POB$ (SAS) 1050 | 1051 | $\therefore\quad \overline{PA}=\overline{PB}$(全等三角形的对应边相等). 1052 | \end{proof} 1053 | 1054 | \begin{example} 1055 | 已知:直线$EF$与直线$AB$、$CD$分别相交于$P$、$Q$ 1056 | 两点,并且$\angle 1=\angle 2$(图2.20). 1057 | 1058 | 求证:$AB\parallel CD$ 1059 | \end{example} 1060 | 1061 | \begin{figure}[htp] 1062 | \centering 1063 | \begin{tikzpicture} 1064 | \tkzDefPoints{0/0/A, 4/0/B, 0/-1/C, 4/-1/D} 1065 | \tkzDrawLines (A,B C,D) 1066 | \tkzLabelPoints[above left](A,C)\tkzLabelPoints[above right](B,D) 1067 | \tkzDefPoints{2/0/P, 1.5/-1/Q} 1068 | \tkzDrawLines[add =1 and 1](P,Q) 1069 | \tkzLabelPoints[below](P,Q) 1070 | \tkzMarkAngles[mark=none, size=.4](A,P,Q D,Q,P) 1071 | \tkzLabelAngles[pos=.6](A,P,Q){1} 1072 | \tkzLabelAngles[pos=.6](D,Q,P){2} 1073 | \node at (.8,-2){$F$}; 1074 | \node at (2.8,1){$E$}; 1075 | \end{tikzpicture} 1076 | \caption{} 1077 | \end{figure} 1078 | 1079 | 1080 | \begin{proof} 1081 | $\because\quad $已知$AB$与$EF$是两条相交 1082 | 于$P$点的直线 1083 | 1084 | $\therefore\quad \angle 1$与$\angle EPB$是对顶角 1085 | 1086 | 又$\because\quad $对顶角相等 1087 | 1088 | $\therefore\quad \angle 1=\angle EPB$ 1089 | 1090 | 已知$\angle 1=\angle 2$, 由等量代换,就有 1091 | $\angle 2=\angle EPB$. 1092 | 由平行线定义,可知$AB\parallel CD$. 1093 | \end{proof} 1094 | 1095 | 通过以上三例,我们初步体会了一下证明定理的步骤.值 1096 | 得注意的是在推理时,每推一步都要有可靠的根据,都要有 1097 | 充分的理由.定义、基本性质、已经证明过的定理和已知条件 1098 | 都可作为推理的根据.推理的根据可以象例2.4、例2.5那样写在 1099 | 结论后面的(\qquad)之中,也可以象例2.6那样写在结论的前面. 1100 | 总之,不管怎样叙述,都要“出言有本”,“推理有据”把 1101 | 论证的理由说清楚. 1102 | 1103 | \section*{习题2.2} 1104 | \addcontentsline{toc}{subsection}{习题2.2} 1105 | \begin{enumerate} 1106 | \item 写出下列各命题的已知和求证. 1107 | \begin{enumerate} 1108 | \item 如果两个角是对顶角,则它们相等. 1109 | \item 如果$x_1$、$x_2$是方程$ax^2+bx+c=0$的两个根,则 1110 | $$x_1+x_2=-\frac{b}{a},\quad x_1\cdot x_2=\frac{c}{a}$$ 1111 | \item 若在$\triangle ABC$中,$\overline{AB}=\overline{AC}$, 则$\angle B=\angle C$. 1112 | \item 多项式$f(x)$除以$(x-a)$所得的余式等于$f(a)$. 1113 | \item 如果$(x-a)(x-b)=0$, 则$x=a$或$x=b$. 1114 | \end{enumerate} 1115 | 1116 | \item 把下列各题抄在练习本上,并在括号内填写理由: 1117 | \begin{enumerate} 1118 | \item 已知:如图$\overline{AB}=\overline{CD}$, 求证:$\overline{AC}=\overline{BD}$. 1119 | 1120 | 证明:$\because\quad \overline{AB}=\overline{CD}$ (\qquad) 1121 | 1122 | 又$\because\quad \overline{BC}=\overline{BC}$ 1123 | 1124 | $\therefore\quad \overline{AB}+\overline{BC}=\overline{CD}+\overline{BC}$ (\qquad) 1125 | 1126 | 即 $\overline{AC}=\overline{BD}$. 1127 | 1128 | \begin{figure}[htp] 1129 | \centering 1130 | \begin{tikzpicture} 1131 | \draw[|-|](0,0)node[below]{$A$}--(5,0)node[below]{$D$}; 1132 | \draw[|-|](2,0)node[below]{$B$}--(3,0)node[below]{$C$}; 1133 | \end{tikzpicture} 1134 | \caption{第2(a)题} 1135 | \end{figure} 1136 | 1137 | \item 已知:如图$\angle AOC=\angle BOD$, 求证:$\angle AOB=\angle COD$ 1138 | 1139 | 证明:$\because\quad \angle AOC=\angle BOD$ (\qquad) 1140 | 1141 | 又$\because\quad \angle BOC=\angle BOC$ 1142 | 1143 | $\therefore\quad \angle AOC-\angle BOC=\angle BOD-\angle BOC$ (\qquad ) 1144 | 1145 | 即 $\angle AOB=\angle COD$. 1146 | 1147 | \item 已知:如图,$C$是$\overline{AB}$的中点,$F$点是$\overline{DE}$的中点,$\overline{AC}=\overline{DF}$, 1148 | 1149 | 求证:$\overline{AB}=\overline{DE}$. 1150 | 1151 | 证明:$\because\quad \overline{AC}=\overline{DF}$ (\qquad) 1152 | 1153 | $\therefore\quad 2\overline{AC}=2\overline{DF}$ (\qquad) 1154 | 1155 | 又$\because\quad C$点,$F$点分别是$\overline{AB}$, $\overline{DE}$的中点(\qquad) 1156 | 1157 | $\therefore\quad 2\overline{AC}=2\overline{AB}$ (\qquad), 1158 | $2\overline{DF}=\overline{DE}$(\qquad). 1159 | 1160 | $\therefore\quad \overline{AB}=\overline{DE}$(\qquad). 1161 | 1162 | \begin{figure}[htp]\centering 1163 | \begin{minipage}[t]{0.48\textwidth} 1164 | \centering 1165 | \begin{tikzpicture}[>=latex, scale=1] 1166 | \foreach \x/\xtext in {15/C,-15/B,50/D,-50/A} 1167 | { 1168 | \draw(0,0)--(\x:2)node[right]{$\xtext$}; 1169 | } 1170 | \node at (0,0)[left]{$O$}; 1171 | \end{tikzpicture} 1172 | \caption*{第2(b)题} 1173 | \end{minipage} 1174 | \begin{minipage}[t]{0.48\textwidth} 1175 | \centering 1176 | \begin{tikzpicture}[>=latex, scale=1] 1177 | \draw[|-|](0,0)node[below]{$A$}--(4,0)node[below]{$B$}; 1178 | \draw[|-|](0,-1)node[below]{$D$}--(4,-1)node[below]{$E$}; 1179 | \node at (2,0)[below]{$C$}; 1180 | \node at (2,-1)[below]{$F$}; 1181 | \draw(2,0)--(2,.1);\draw(2,-1)--(2,-1+.1); 1182 | \end{tikzpicture} 1183 | \caption*{第2(c)题} 1184 | \end{minipage} 1185 | \end{figure} 1186 | 1187 | 1188 | \item 已知:如图,$\angle ABC=\angle EFG$, $BD$, $FH$分别是 1189 | $\angle ABC$,$\angle FFG$的平分线. 1190 | 1191 | 求证:$\angle ABD=\angle HFG$. 1192 | 1193 | 证明:$\because\quad BD, FH$分别是$\angle ABC$和$\angle EFG$的平分线(\qquad ) 1194 | 1195 | $\therefore\quad \angle ABD=\angle DBC,\quad \angle EFH=\angle HFG$(\qquad) 1196 | 1197 | 又$\because\quad \angle ABC=\angle EFG$(\qquad) 1198 | 1199 | $\therefore\quad \angle ABD=\angle HFG$(\qquad) 1200 | 1201 | \begin{figure}[htp]\centering 1202 | \begin{minipage}[t]{0.48\textwidth} 1203 | \centering 1204 | \begin{tikzpicture}[>=latex, scale=1] 1205 | \foreach \x/\xtext in {30/D,60/A,0/C} 1206 | { 1207 | \draw(0,0)--(\x:2)node[right]{$\xtext$}; 1208 | } 1209 | \node at (0,0)[left]{$B$}; 1210 | 1211 | \draw (0,-2)node[left]{$F$}--(1.5,-2)node[right]{$G$}; 1212 | \draw (0,-2)--(.75*1.732,-2+.75)node[right]{$H$}; 1213 | \draw (0,-2)--(.75,-2+.75*1.732)node[right]{$E$}; 1214 | \end{tikzpicture} 1215 | \caption*{第2(d)题} 1216 | \end{minipage} 1217 | \begin{minipage}[t]{0.48\textwidth} 1218 | \centering 1219 | \begin{tikzpicture}[>=latex, scale=1] 1220 | \tkzDefPoints{-1/-2/A, 3/-2/B, 1/1/C, -.5/-2.5/D, 2.5/-2.5/E} 1221 | \tkzDrawLines[add =0 and 0](C,D C,E A,B) 1222 | \tkzInterLL(A,B)(C,D) \tkzGetPoint{F} 1223 | \tkzInterLL(A,B)(C,E) \tkzGetPoint{G} 1224 | \tkzMarkAngles[mark=none, size=.4](A,F,D E,G,B B,F,C C,G,A) 1225 | \tkzLabelAngle[pos=.5](A,F,D){3} 1226 | \tkzLabelAngle[pos=.5](E,G,B){4} 1227 | \tkzLabelAngle[pos=.5](B,F,C){1} 1228 | \tkzLabelAngle[pos=.5](C,G,A){2} 1229 | 1230 | \end{tikzpicture} 1231 | \caption*{第2(e)题} 1232 | \end{minipage} 1233 | \end{figure} 1234 | 1235 | \item 已知:如图,$\angle 1$和$\angle 3$, $\angle 2$和$\angle 4$是对顶角,且$\angle 1=\angle 2$. 1236 | 1237 | 求证:$\angle 3=\angle 4$. 1238 | 1239 | 证明:$\because\quad \angle 1$和$\angle 3$, $\angle 2$和$\angle 4$是对顶角(\qquad) 1240 | 1241 | $\therefore\quad \angle 1=\angle 3,\; \angle 2=\angle 4$(\qquad) 1242 | 1243 | 又$\because\quad \angle 1=\angle 2$(\qquad ) 1244 | 1245 | $\therefore\quad \angle 3=\angle 4$(\qquad ) 1246 | 1247 | \item 已知:如图,直线$EF$与直线$AB$、$CD$分别相交于 1248 | $P$、$Q$两点,且$\angle 1=\angle 2$. 1249 | 1250 | 求证:$\angle 2+\angle 3=180^{\circ}$. 1251 | 1252 | 证明:$\because\quad AB$是过$P$的直线(\qquad) 1253 | 1254 | $\therefore\quad \angle 1$与$\angle 3$互为补角(\qquad) 1255 | 1256 | 即$\angle 1+\angle 3=180^{\circ}$. 1257 | 1258 | 又$\because\quad \angle 1=\angle 2$(\qquad) 1259 | 1260 | $\therefore\quad \angle 2+\angle 3=180^{\circ}$(\qquad ). 1261 | 1262 | \item 已知:如图,直线$AO\bot OC$, 直线$BO\bot OD$ 1263 | 1264 | 求证:$\angle 1=\angle 2$. 1265 | 1266 | 证明:$\because\quad AO\bot OC$(\qquad ) 1267 | 1268 | $\therefore\quad \angle AOC=90^{\circ}$ (\qquad ) 1269 | 1270 | 即$\angle 1+\angle BOC=90^{\circ}$ 1271 | 1272 | $\therefore\quad \angle 1=90^{\circ}-\angle BOC$. 1273 | 1274 | 同理:$\angle 2=90^{\circ}-\angle BOC$ 1275 | 1276 | $\therefore\quad \angle 1=\angle 2$ (\qquad ) 1277 | 1278 | \begin{figure}[htp]\centering 1279 | \begin{minipage}[t]{0.48\textwidth} 1280 | \centering 1281 | \begin{tikzpicture}[>=latex, scale=1.2] 1282 | \tkzDefPoints{0/0/A, 4/0/B, 0/-1/C, 4/-1/D} 1283 | \tkzDrawSegments (A,B C,D) 1284 | \tkzLabelPoints[left](A,C) 1285 | \tkzLabelPoints[right](B,D) 1286 | \tkzDefPoints{2/0/P, 1.5/-1/Q} 1287 | \tkzDrawLines[add =1 and 1](P,Q) 1288 | \tkzLabelPoints[below](Q)\tkzLabelPoints[above](P) 1289 | \tkzMarkAngles[mark=none, size=.4](A,P,Q D,Q,P) 1290 | \tkzLabelAngles[pos=.6](A,P,Q){1} 1291 | \tkzLabelAngles[pos=.6](D,Q,P){2} 1292 | \tkzMarkAngle[mark=none, size=.5](Q,P,B) 1293 | \tkzLabelAngles[pos=.6](Q,P,B){3} 1294 | \node at (.8,-2){$F$}; 1295 | \node at (2.8,1){$E$}; 1296 | \end{tikzpicture} 1297 | \caption*{第2(f)题} 1298 | \end{minipage} 1299 | \begin{minipage}[t]{0.48\textwidth} 1300 | \centering 1301 | \begin{tikzpicture}[>=latex, scale=1] 1302 | \tkzDefPoint(0:0){O} 1303 | \tkzDefPoint(0:2.5){D} 1304 | \tkzDefPoint(30:2.5){C} 1305 | \tkzDefPoint(90:2.5){B} 1306 | \tkzDefPoint(120:2.5){A} 1307 | \tkzAutoLabelPoints[center=O](A,B,C,D) 1308 | \tkzDrawSegments(O,A O,B O,C O,D) 1309 | \tkzMarkRightAngles(B,O,D A,O,C) 1310 | \tkzMarkAngles[mark=none, size=.7](D,O,C B,O,A) 1311 | \tkzLabelAngles[pos=.8](D,O,C){2} 1312 | \tkzLabelAngles[pos=.8](B,O,A){1} 1313 | \node at (-.2,-.2){$O$}; 1314 | \end{tikzpicture} 1315 | \caption*{第2(g)题} 1316 | \end{minipage} 1317 | \end{figure} 1318 | 1319 | \item 已知:如图,$\overline{AB}=\overline{DC}$, $\overline{AC}=\overline{DB}$ 1320 | 1321 | 求证:$\angle 1=\angle 2$. 1322 | 1323 | 证明:在$\triangle ABC$与$\triangle DCB$中 1324 | 1325 | $\because\quad \overline{AB}=\overline{DC},\quad \overline{AC}=\overline{DB}$(\qquad ) 1326 | 1327 | 又$\because\quad \overline{BC}=\overline{BC}$(\qquad) 1328 | 1329 | $\therefore\quad \triangle ABC\cong \triangle DCB$(\qquad) 1330 | 1331 | $\therefore\quad \angle 1=\angle 2$(\qquad ) 1332 | 1333 | \item 已知:如图,$\overline{AC}$与$\overline{BD}$交于$O$点,并且$\overline{OA}=\overline{OC}$, $\overline{OB}=\overline{OD}$. 1334 | 1335 | 求证:$\overline{AB}=\overline{CD}$. 1336 | 1337 | 证明:在$\triangle AOB$与$\triangle COD$中, 1338 | 1339 | $\because\quad \overline{OA}=\overline{OC},\quad \overline{OB}=\overline{OD}$(\qquad) 1340 | 1341 | 又$\because\quad \overline{AC}$与$\overline{BD}$交于$O$点(\qquad) 1342 | 1343 | $\therefore\quad \angle AOB=\angle COD$(\qquad) 1344 | 1345 | $\therefore\quad \triangle AOB\cong\triangle COD$(\qquad) 1346 | 1347 | $\therefore\quad \overline{AB}=\overline{CD}$ 1348 | 1349 | \begin{figure}[htp]\centering 1350 | \begin{minipage}[t]{0.48\textwidth} 1351 | \centering 1352 | \begin{tikzpicture}[>=latex, scale=1] 1353 | \tkzDefPoints{-2/0/B, 2/0/C, -1/3/A, 1/3/D} 1354 | \tkzDrawSegments(A,B A,C B,D B,C D,C) 1355 | \tkzLabelPoints[left](A,B) 1356 | \tkzLabelPoints[right](C,D) 1357 | \tkzMarkAngles[mark=none, size=.4](A,C,B C,B,D) 1358 | \tkzLabelAngles[pos=.6](A,C,B){2} 1359 | \tkzLabelAngles[pos=.6](C,B,D){1} 1360 | 1361 | \end{tikzpicture} 1362 | \caption*{第2(h)题} 1363 | \end{minipage} 1364 | \begin{minipage}[t]{0.48\textwidth} 1365 | \centering 1366 | \begin{tikzpicture}[>=latex, scale=1] 1367 | \draw(0,0)node[left]{$D$}--(3,0)node[right]{$C$}; 1368 | \draw(1,3)node[left]{$A$}--(4,3)node[right]{$B$}; 1369 | \draw(0,0)--(4,3); 1370 | \draw(1,3)--(3,0); 1371 | \node at (2,1.5)[left]{$O$}; 1372 | \end{tikzpicture} 1373 | \caption*{第2(i)题} 1374 | \end{minipage} 1375 | \end{figure} 1376 | 1377 | \item 已知:如图,$\angle 1=\angle 2$, $\angle 3=\angle 4$, 1378 | 1379 | 求证:$\overline{AC}=\overline{AD}$ 1380 | 1381 | 证明:$\because\quad \angle 1=\angle 2,\quad \angle 3=\angle 4$(\qquad ) 1382 | 1383 | 又$\because\quad \overline{AB}=\overline{AB}$(\qquad ) 1384 | 1385 | $\therefore\quad\triangle ABC\cong \triangle ABD$(\qquad ) 1386 | 1387 | $\therefore\quad \overline{AC}=\overline{AD}$(\qquad ) 1388 | 1389 | \begin{figure}[htp]\centering 1390 | \begin{minipage}[t]{0.48\textwidth} 1391 | \centering 1392 | \begin{tikzpicture}[>=latex, scale=1] 1393 | \tkzDefPoints{0/0/A, 1/1.5/C, 1/-1.5/D, 4/0/B} 1394 | \tkzDrawSegments(A,C C,B B,D A,D A,B) 1395 | \tkzLabelPoints[left](A,C,D) 1396 | \tkzLabelPoint[right](B){$B$} 1397 | \tkzMarkAngles[mark=none, size=.4](B,A,C A,B,D) 1398 | \tkzLabelAngles[pos=.6](B,A,C){1} 1399 | \tkzLabelAngles[pos=.6](A,B,D){4} 1400 | \tkzMarkAngles[mark=none, size=.5](C,B,A D,A,B) 1401 | \tkzLabelAngles[pos=.7](C,B,A){3} 1402 | \tkzLabelAngles[pos=.7](D,A,B){2} 1403 | \end{tikzpicture} 1404 | \caption*{第2(j)题} 1405 | \end{minipage} 1406 | \begin{minipage}[t]{0.48\textwidth} 1407 | \centering 1408 | \begin{tikzpicture}[>=latex, scale=.8] 1409 | \draw(-3,0)node[below]{$B$}--(0,3)node[above]{$A$}--(3,0)node[below]{$C$}; 1410 | \draw(-1,2)node[left]{$D$}--(3,0); 1411 | \draw(1,2)node[right]{$E$}--(-3,0); 1412 | \end{tikzpicture} 1413 | \caption*{第3(a)题} 1414 | \end{minipage} 1415 | \end{figure} 1416 | \end{enumerate} 1417 | 1418 | \item 去掉下列各题中推理的多余步骤. 1419 | \begin{enumerate} 1420 | \item 已知;如图,$\overline{AC}=\overline{AB}$, $\overline{AD}=\overline{AE}$. 1421 | 1422 | 求证:$\angle ADC=\angle AEB$. 1423 | 1424 | 证明:在$\triangle ADC$和$\triangle AEB$中, 1425 | 1426 | $\because\quad \overline{AC}=\overline{AB},\quad \overline{AD}=\overline{AE}$(已知) 1427 | 1428 | 又$\because\quad \angle A=\angle A$ (公共角) 1429 | 1430 | 又$\because\quad \angle B=\angle C$ 1431 | 1432 | $\therefore\quad \triangle ADC\cong \triangle AEB$(SAS) 1433 | 1434 | $\therefore\quad \angle ADC=\angle AEB$(全等三角形对应角相 1435 | 等) 1436 | 1437 | \item 已知:如图,$\overline{AB}=\overline{DC},\quad \angle A=\angle D,\quad \angle B=\angle C$ 1438 | 1439 | 求证:$\overline{BF}=\overline{CE}$. 1440 | 1441 | 证明:在$\triangle ABE$与$\triangle DCF$中, 1442 | 1443 | $\because\quad \overline{AB}=\overline{DC},\quad \angle A=\angle D,\quad \angle B=\angle C$(已知) 1444 | 1445 | 又$\because\quad \angle 1=\angle 2$ 1446 | 1447 | $\therefore\quad \triangle ABE\cong \triangle DCF$(ASA) 1448 | 1449 | $\therefore\quad \overline{BE}=\overline{CF}$(全等三角形的对应边相等) 1450 | 1451 | 又$\because\quad \overline{EF}=\overline{EF}$ 1452 | 1453 | $\therefore\quad \overline{BF}=\overline{CE}$(等量减等量差相等) 1454 | \end{enumerate} 1455 | 1456 | \begin{figure}[htp]\centering 1457 | \begin{minipage}[t]{0.48\textwidth} 1458 | \centering 1459 | \begin{tikzpicture}[>=latex, scale=1] 1460 | \tkzDefPoints{0/0/C, 2/0/D, 2/3/F, 1/1.5/E, 1/4.5/A, 3/4.5/B} 1461 | \tkzDrawSegments(B,C F,D A,E A,B C,D) 1462 | \tkzLabelPoints[left](A,C,E) 1463 | \tkzLabelPoints[right](B,F,D) 1464 | \tkzMarkAngles[mark=none, size=.5](C,F,D B,E,A) 1465 | \tkzLabelAngles[pos=.7](C,F,D){1} 1466 | \tkzLabelAngles[pos=.7](B,E,A){2} 1467 | \end{tikzpicture} 1468 | \caption*{第3(b)题} 1469 | \end{minipage} 1470 | \begin{minipage}[t]{0.48\textwidth} 1471 | \centering 1472 | \begin{tikzpicture}[>=latex, scale=1] 1473 | \tkzDefPoints{-2.5/0/D, 2.5/0/B, -1/2.5/A, 1/2.5/C} 1474 | \tkzDrawSegments(A,B A,D B,C C,D) 1475 | \tkzLabelPoints[left](A,D) 1476 | \tkzLabelPoints[right](C,B) 1477 | \node at (0,1.5){$O$}; 1478 | \end{tikzpicture} 1479 | \caption*{第4(a)题} 1480 | \end{minipage} 1481 | \end{figure} 1482 | 1483 | \item 补上下列各题推理中所缺少的必要步骤. 1484 | \begin{enumerate} 1485 | \item 已知;如图,$\overline{AB}$, $\overline{CD}$相交于$O$点,并且$\overline{AO}=\overline{OC}$,$\overline{OD}=\overline{OB}$. 1486 | 1487 | 求证:$\angle D=\angle B$, $\overline{AD}=\overline{BC}$. 1488 | 1489 | 证明:在$\triangle AOD$和$\triangle COB$中, 1490 | 1491 | $\because\quad \overline{AO}=\overline{OC}$(已知) 1492 | 1493 | 又$\because\quad \overline{AB}$, $\overline{CD}$相交于$O$点(已知) 1494 | 1495 | $\therefore\quad \angle AOD=\angle COB$(对顶角相等) 1496 | 1497 | $\therefore\quad \triangle AOD\cong \triangle COB$(SAS) 1498 | 1499 | $\therefore\quad \angle D=\angle B$(全等三角形的对应角相等), 1500 | $\overline{AD}=\overline{BC}$(全等三角形的对应边相等). 1501 | 1502 | \item 已知:如图,$\overline{AD}=\overline{BC}$, $\overline{AB}=\overline{CD}$. 1503 | 1504 | 求证:$\angle B=\angle D$. 1505 | 1506 | 证明:在$\triangle ABC$与$\triangle CDA$中 1507 | 1508 | $\because\quad \overline{AD}=\overline{BC},\quad \overline{AB}=\overline{CD}$(已知) 1509 | 1510 | $\therefore\quad \triangle ABC\triangle CDA$(SSS) 1511 | 1512 | $\therefore\quad \angle B=\angle D$(全等三角形的对应角相等). 1513 | 1514 | 1515 | \end{enumerate} 1516 | 1517 | \begin{figure}[htp]\centering 1518 | \begin{minipage}[t]{0.48\textwidth} 1519 | \centering 1520 | \begin{tikzpicture}[>=latex, scale=1] 1521 | \tkzDefPoints{0/0/B, 3/0/C, 1/2/A, 4/2/D} 1522 | \tkzDrawSegments(A,B A,C A,D B,C D,C) 1523 | \tkzLabelPoints[left](A,B) 1524 | \tkzLabelPoints[right](C,D) 1525 | \end{tikzpicture} 1526 | \caption*{第4(b)题} 1527 | \end{minipage} 1528 | \begin{minipage}[t]{0.48\textwidth} 1529 | \centering 1530 | \begin{tikzpicture}[>=latex, scale=1] 1531 | \tkzDefPoint(0:3){A} 1532 | \tkzDefPoint(60:3){C} 1533 | \tkzDefPoint(30:3){B} 1534 | \tkzDefPoint(20:3){A1} 1535 | \tkzDefPoint(50:3){A2} 1536 | \tkzDefPoint(0,0){O} 1537 | \tkzDrawSegments(O,A O,B O,C O,A1 O,A2) 1538 | \tkzLabelPoints[right](A,B,C) 1539 | \node at (-.2,-.2){$O$}; 1540 | 1541 | \draw(1,0) arc (0:20:1); 1542 | \draw(20:1.5) arc (20:30:1.5); 1543 | \draw(30:1) arc (30:50:1); 1544 | \draw(50:1.5) arc (50:60:1.5); 1545 | \node at (10:1.2){1}; 1546 | \node at (25:1.8){2}; 1547 | \node at (40:1.2){3}; 1548 | \node at (55:1.8){4}; 1549 | \end{tikzpicture} 1550 | \caption*{第5题} 1551 | \end{minipage} 1552 | \end{figure} 1553 | 1554 | \item 已知:如图,$OB$是$\angle AOC$的平分线,并且$\angle 1=\angle 4$, 1555 | 1556 | 求证:$\angle 2=\angle 3$. 1557 | \item 已知:如图,$\angle 2$与$\angle 3$是对顶角,且$\angle 1=\angle 3$, 1558 | 1559 | 求证:$\angle 1=\angle 2$. 1560 | \item 已知:如图,$\overline{AB}=\overline{AC}$, $D$是$\overline{BC}$的中点, 1561 | 1562 | 求证:$\triangle ABD\cong \triangle ACD$. 1563 | \begin{figure}[htp]\centering 1564 | \begin{minipage}[t]{0.48\textwidth} 1565 | \centering 1566 | \begin{tikzpicture}[>=latex, scale=1] 1567 | \tkzDefPoints{0/0/A, 4/0/B, 0/-1/C, 4/-1/D} 1568 | \tkzDrawSegments (A,B C,D) 1569 | \tkzDefPoints{2/0/P, 1.5/-1/Q, 2.5/1/E} 1570 | \tkzDrawLines[add =1 and 1](P,Q) 1571 | \tkzMarkAngles[mark=none, size=.4](A,P,Q D,Q,P B,P,E) 1572 | \tkzLabelAngles[pos=.6](A,P,Q){2} 1573 | \tkzLabelAngles[pos=.6](D,Q,P){1} 1574 | \tkzLabelAngles[pos=.6](B,P,E){3} 1575 | 1576 | \end{tikzpicture} 1577 | \caption*{第6题} 1578 | \end{minipage} 1579 | \begin{minipage}[t]{0.48\textwidth} 1580 | \centering 1581 | \begin{tikzpicture}[>=latex, scale=1] 1582 | \draw(-1.5,0)node[below]{$B$}--(0,0)node[below]{$D$}--(1.5,0)node[below]{$C$}--(0,3)--(-1.5,0); 1583 | \draw(0,3)node[above]{$A$}--(0,0); 1584 | \end{tikzpicture} 1585 | \caption*{第7题} 1586 | \end{minipage} 1587 | \end{figure} 1588 | 1589 | \item 已知:如图,$\overline{AB}\bot \overline{BD}$, $\overline{CD}\bot \overline{BD}$, $O$是$\overline{BD}$的中点, 1590 | $\overline{AC}$与$\overline{BD}$交于$O$. 1591 | 1592 | 求证:$\triangle ABO\cong \triangle CDO$. 1593 | \item 已知;如图,$BD$平分$\angle ABC$, 并且$\overline{AB}=\overline{BC}$, 1594 | 1595 | 求证:$\triangle ABD\cong \triangle CBD$. 1596 | \begin{figure}[htp]\centering 1597 | \begin{minipage}[t]{0.48\textwidth} 1598 | \centering 1599 | \begin{tikzpicture}[>=latex, scale=1] 1600 | \draw(0,0)node[below]{$B$}--(4,0)node[right]{$D$}--(4,-1.5)node[right]{$C$}--(2,0)node[below]{$O$}--(0,1.5)node[left]{$A$}--(0,0) ; 1601 | \end{tikzpicture} 1602 | \caption*{第8题} 1603 | \end{minipage} 1604 | \begin{minipage}[t]{0.48\textwidth} 1605 | \centering 1606 | \begin{tikzpicture}[>=latex, scale=1] 1607 | \draw(0,0)node[left]{$B$}--(4,0)node[right]{$D$}--(3,1.5)node[right]{$A$}--(0,0)--(3,-1.5)node[right]{$C$}--(4,0); 1608 | \end{tikzpicture} 1609 | \caption*{第9题} 1610 | \end{minipage} 1611 | \end{figure} 1612 | 1613 | 1614 | 1615 | 1616 | 1617 | \end{enumerate} 1618 | 1619 | 1620 | \section*{复习题二} 1621 | \addcontentsline{toc}{section}{复习题二} 1622 | \begin{enumerate} 1623 | \item 试举出五个集合的例子,并用列举法或描述法表示出来. 1624 | \item 写出下列集合的所有元素. 1625 | \begin{enumerate} 1626 | \item $A=\{\text{绝对值小于10的2的倍数}\}$ 1627 | \item $B=\{\text{绝对值小于40的5的倍数}\}$ 1628 | \item $C=\{\text{绝对值小于20的3的倍数}\}$ 1629 | \end{enumerate} 1630 | 1631 | \item 用特征性质描述法表示下列集合: 1632 | \begin{enumerate} 1633 | \item 一切偶数的集合. 1634 | \item 一切奇数的集合. 1635 | \item 大于等于0, 而小于等于1的实数集合. 1636 | \end{enumerate} 1637 | 1638 | \item 下列命题是否正确? 1639 | \begin{enumerate} 1640 | \item 一个正方形是正方形集合的一个元素. 1641 | \item 飞机是飞机场集合的一个元素. 1642 | \item 某校一成员是这校全体学生构成的集合的一个元素. 1643 | \end{enumerate} 1644 | 1645 | \item 指出下列集合中的元素: 1646 | \begin{enumerate} 1647 | \item $A=\{x|x<15\text{ 且 $x$ 是质数}\}$ 1648 | \item $B=\{x|x^2-2=0\text{ 且 $x$ 是实数}\}$ 1649 | \end{enumerate} 1650 | 1651 | \item 设$A=\{1,2,3,4\}$, $B=\{ 2,4,6,8\}$, $C=\{3,4,5,6\}$, 求: 1652 | \begin{multicols}{4} 1653 | \begin{enumerate} 1654 | \item $A\cup B$ 1655 | \item $A\cup C$ 1656 | \item $B\cup C$ 1657 | \item $(A\cup B)\cup C$ 1658 | \item $A\cup (B\cup C)$ 1659 | \item $A\cap B$ 1660 | \item $A\cap C$ 1661 | \item $B\cap C$ 1662 | \item $(A\cap B)\cap C$ 1663 | \item $A\cap (B\cap C)$ 1664 | \end{enumerate} 1665 | \end{multicols} 1666 | 1667 | 1668 | \item 如图,$A$、$B$、$C$三个集合相交成非空集,用阴影分别 1669 | 把下列各集合所表示的区域划出来. 1670 | \begin{multicols}{4} 1671 | \begin{enumerate} 1672 | \item $A\cap B$ 1673 | \item $A\cup B$ 1674 | \item $A\cap C$ 1675 | \item $A\cup C$ 1676 | \item $B\cap C$ 1677 | \item $B\cup C$ 1678 | \item $A\cap (B\cap C)$ 1679 | \item $(A\cap B)\cap C$ 1680 | \item $A\cup (B\cup C)$ 1681 | \item $(A\cup B)\cup C$ 1682 | \end{enumerate} 1683 | \end{multicols} 1684 | 1685 | \begin{figure}[htp]\centering 1686 | \begin{minipage}[t]{0.48\textwidth} 1687 | \centering 1688 | \begin{tikzpicture}[>=latex, scale=1] 1689 | \draw (0,0)node{$A$} circle (1); 1690 | \draw (1.7,0) node{$B$} circle (1.2); 1691 | \draw (.7,-1.2) node{$C$} circle (.8); 1692 | \end{tikzpicture} 1693 | \caption*{第7题} 1694 | \end{minipage} 1695 | \begin{minipage}[t]{0.48\textwidth} 1696 | \centering 1697 | \begin{tikzpicture}[>=latex, scale=1] 1698 | \tkzDefPoints{0/0/B, -1/2/A, 1/2/C, 0/3.5/E, 0/2.75/D} 1699 | \tkzDrawLines[add=0 and .5](B,A B,C B,D) 1700 | \tkzLabelPoints[left](A,B) 1701 | \tkzLabelPoints[right](C,D) 1702 | \draw(A)--(D)--(C); 1703 | \tkzMarkAngles[mark=none, size=.45](E,D,A E,B,A) 1704 | \tkzMarkAngles[mark=none, size=.5](C,D,E C,B,E) 1705 | \tkzLabelAngle[pos=.6](E,D,A){1} 1706 | \tkzLabelAngle[pos=.7](E,B,A){3} 1707 | \tkzLabelAngle[pos=.6](C,D,E){2} 1708 | \tkzLabelAngle[pos=.7](C,B,E){4} 1709 | \node at (0,3.8)[right]{$E$}; 1710 | \end{tikzpicture} 1711 | \caption*{第11题} 1712 | \end{minipage} 1713 | \end{figure} 1714 | 1715 | \item 设基集${\rm I}=\{1,2,3,4,5,6,7,8,9,10\}$, 1716 | $A=\{6,8,9\}$, $B=\{ 1, 3, 7,8,9\}$, 1717 | $C=\{2,6,8,9\}$.求: 1718 | \begin{multicols}{4} 1719 | \begin{enumerate} 1720 | \item $A^c$ 1721 | \item $B^c$ 1722 | \item $C^c$ 1723 | \item $(A\cap B)^c$ 1724 | \item $(A^c)\cap (B^c)$ 1725 | \item $(A\cup B)^c$ 1726 | \item $(A^c)\cup (B^c)$ 1727 | \end{enumerate} 1728 | \end{multicols} 1729 | 1730 | \item 写出列各命题中的“已知”部分和“求证”部分. 1731 | \begin{enumerate} 1732 | \item 三角形三个角的和等于$180^{\circ}$. 1733 | \item 角平分线上任一点到角的两边距离相等. 1734 | \end{enumerate} 1735 | 1736 | \item 在下列各题的(\qquad)中,适当地填上“必要不充分”, 1737 | “充分不必要”,“充分必要”等词. 1738 | \begin{enumerate} 1739 | \item $a=0$是$ab=0$的(\qquad)条件. 1740 | \item 两个角相等是两个角为对顶角的(\qquad)条件. 1741 | \item 形状和大小完全相同的两个图形是全等形的(\qquad) 1742 | 条件. 1743 | \item $a^2=b^2$是$a=b$的(\qquad)条件. 1744 | \item 多项式$P(x)$被$x-\alpha$整除是$P(\alpha)=0$的(\qquad) 1745 | 条件. 1746 | \item $a\ne b$ 是$a^2+b^2>2ab$的(\qquad)条件. 1747 | \end{enumerate} 1748 | 1749 | \item 已知:如图,$\angle 1=\angle 2$, $\angle 3=\angle 4$, $BE$是直线. 1750 | 1751 | 求证:$\overline{AD}=\overline{CD}$. 1752 | 1753 | \item 已知:如图,$\overline{AB}=\overline{DE}$, $\overline{AC}=\overline{DF}$, $\overline{BC}=\overline{EF}$. 1754 | 1755 | 求证:$\angle 1=\angle 2$, $AC\parallel DF$. 1756 | \item 已知:如图,$\angle 1=\angle 2$, $\overline{AD}=\overline{BC}$. 1757 | 1758 | 求证:$\angle B=\angle D$, $\overline{AB}=\overline{CD}$. 1759 | 1760 | \begin{figure}[htp]\centering 1761 | \begin{minipage}[t]{0.48\textwidth} 1762 | \centering 1763 | \begin{tikzpicture}[>=latex, scale=.8] 1764 | \draw (0,0)node[below]{$B$}--(4,0)node[below]{$C$}--(1.5,3)node[above]{$A$}--(0,0); 1765 | \draw (3,0)node[below]{$E$}--(7,0)node[below]{$F$}--(4.5,3)node[above]{$D$}--(3,0); 1766 | \foreach \x/\xtext in {4/1,7/2} 1767 | { 1768 | \draw (\x-.25,0) arc (180:130:.25); 1769 | \draw(\x-.5,0)node[above]{$\xtext$}; 1770 | } 1771 | \end{tikzpicture} 1772 | \caption*{第12题} 1773 | \end{minipage} 1774 | \begin{minipage}[t]{0.48\textwidth} 1775 | \centering 1776 | \begin{tikzpicture}[>=latex, scale=.7] 1777 | \draw(1,0)--(-3.5,-4)node[below]{$B$}--(-1,0)node[above]{$A$}--(1,0)node[above]{$C$}--(3.5,-4)node[below]{$D$}--(-1,0); 1778 | \draw (1-.5,0) arc (180:220:.5); 1779 | \draw (-1+.5,0) arc (0:-40:.5); 1780 | \draw(1-.75,0)node[below]{2};\draw(-1+.75,0)node[below]{1}; 1781 | \end{tikzpicture} 1782 | \caption*{第13题} 1783 | \end{minipage} 1784 | \end{figure} 1785 | 1786 | \item 证明:同角的补角相等. 1787 | \item 证明:等角的余角相等. 1788 | \item 证明:等角的补角相等. 1789 | \end{enumerate} -------------------------------------------------------------------------------- /6.tex: -------------------------------------------------------------------------------- 1 | \chapter{三角比与角边关系} 2 | 3 | 人们为了要确定空间各点之间的相互位置,就得做一番 4 | 测量,测量是几何学的起源,也是几何学最直接的实践. 5 | 6 | 测量学的最基本原理,就是相似形的性质及三角形的边 7 | 角关系.例如,我们在第三章末用相似形性质测量两点间的, 8 | 距离,物体的高度、测绘具有多边形形状的地段的平面图 9 | 等.我们知道,在两个直角三角形中,只要有一个锐角对应 10 | 相等,它们就相似了,这就是说,一个直角三角形的各边之. 11 | 间的比是被它的一个锐角的大小所决定,例如在图6.1中, 12 | 一些含有$30^{\circ}$角的直角三角形,$30^{\circ}$角所对的直角边与斜边的 13 | 比都是1:2. 14 | 15 | \begin{figure}[htp] 16 | \centering 17 | \begin{tikzpicture} 18 | \begin{scope} 19 | \tkzDefPoint(30:2){B_1} 20 | \tkzDefPoints{0/0/A_1, 1.732/0/C_1} 21 | \tkzDrawPolygon(A_1,B_1,C_1) 22 | \tkzMarkAngle[mark=none, size=.5](C_1,A_1,B_1) 23 | \tkzLabelPoints[right](C_1,B_1) 24 | \tkzLabelPoints[left](A_1) 25 | \tkzMarkRightAngle(B_1,C_1,A_1) 26 | \end{scope} 27 | \begin{scope}[xshift=3.5cm] 28 | \tkzDefPoint(30:3){B_2} 29 | \tkzDefPoints{0/0/A_2, 2.6/0/C_2} 30 | \tkzDrawPolygon(A_2,B_2,C_2) 31 | \tkzMarkAngle[mark=none, size=.5](C_2,A_2,B_2) 32 | \tkzLabelPoints[right](C_2,B_2) 33 | \tkzLabelPoints[left](A_2) 34 | \tkzMarkRightAngle(B_2,C_2,A_2) 35 | \end{scope} 36 | \begin{scope}[xshift=11cm] 37 | \tkzDefPoint(150:4){B_3} 38 | \tkzDefPoints{0/0/A_3, -3.464/0/C_3} 39 | \tkzDrawPolygon(A_3,B_3,C_3) 40 | \tkzMarkAngle[mark=none, size=.6](B_3,A_3,C_3) 41 | \tkzLabelPoints[below](C_3,A_3) 42 | \tkzLabelPoints[left](B_3) 43 | \tkzMarkRightAngle(A_3,C_3,B_3) 44 | \end{scope} 45 | 46 | \end{tikzpicture} 47 | \caption{} 48 | \end{figure} 49 | 50 | 这一章,我们首先向同学介绍的就是直角三角形中,边 51 | 与边的比与它所含锐角之间的关系.这些边与边的比值叫做 52 | \textbf{锐角三角比},它们是进行测量计算时的常用数据,也是从数 53 | 量方面研究几何学的基本工具. 54 | 55 | \section{锐角三角比} 56 | 57 | \subsection{定义} 58 | 59 | \begin{figure}[htp]\centering 60 | \begin{minipage}[t]{0.48\textwidth} 61 | \centering 62 | \begin{tikzpicture}[>=latex, scale=.8] 63 | \draw(30:6)node[right]{$Y$}--(0,0)node[left]{$A$}--(5.5,0)node[right]{$X$}; 64 | \draw(30:5)node[above]{$B$}--node[right]{$a$}+(0,-2.5)node[below]{$C$}; 65 | \node at (1.25*1.732,0)[below]{$b$}; 66 | \node at (30:2.5)[above]{$c$}; 67 | \draw(4.33,0) rectangle (4.33-.2,.2); 68 | \end{tikzpicture} 69 | \caption{} 70 | \end{minipage} 71 | \begin{minipage}[t]{0.48\textwidth} 72 | \centering 73 | \begin{tikzpicture}[>=latex, scale=.8] 74 | \draw(30:5.5)node[right]{$Y$}--(0,0)node[left]{$A$}--(5,0)node[right]{$X$}; 75 | \draw(30:2.5)node[above]{$B$}--+(0,-1.25)node[below]{$C$}; 76 | \draw(30:3.5)node[above]{$B'$}--+(0,-1.75)node[below]{$C'$}; 77 | \draw(30:4.5)--+(0,-2.25); 78 | \draw(2.165,0) rectangle (2.165-.2,.2); 79 | \draw(3.03,0) rectangle (3.03-.2,.2); 80 | \draw(3.9,0) rectangle (3.9-.2,.2); 81 | \end{tikzpicture} 82 | \caption{} 83 | \end{minipage} 84 | \end{figure} 85 | 86 | 87 | 取任意锐角$\angle XAY$, 在边$AY$上任取一点$B$, 作$\overline{BC}\bot AX$ 88 | 于$C$(图6.2). 在直角$\triangle ABC$中,设$\angle A$、$\angle B$、$\angle C$的 89 | 对边分别用$a$、$b$、$c$表示,对 90 | 锐角$A$来说,$a$叫做$\angle A$的\textbf{对 91 | 边},$b$叫做$\angle A$的相邻的直角 92 | 边(简称\textbf{邻边})我们定义: 93 | \begin{enumerate} 94 | \item $\angle A$的对边与斜边的比值,叫做$\angle A$的正弦,用符 95 | 号$\sin A$来表示,即 96 | \[\sin A=\frac{\angle A\text{的对边}}{\text{斜边}}=\frac{a}{c}\] 97 | \item $\angle A$的邻边与斜边的比值,叫做$\angle A$的余弦.用符 98 | 号$\cos A$来表示,即 99 | \[\cos A=\frac{\angle A\text{的邻边}}{\text{斜边}}=\frac{b}{c}\] 100 | \item $\angle A$的对边与邻边的比值,叫做$\angle A$的正切,用符 101 | 号$\tan A$来表示,即 102 | \[\tan A=\frac{\angle A\text{的对边}}{\angle A\text{的邻边}}=\frac{a}{b}\] 103 | \item $\angle A$的邻边与对边的比值,叫做 $\angle A$的余切,用符 104 | 号$\cot A$来表示,即 105 | \[\cot A=\frac{\angle A\text{的邻边}}{\angle A\text{的对边}}=\frac{b}{a}\] 106 | \end{enumerate} 107 | 108 | 我们知道,只要$\angle A$的大 109 | 小定了,不管$B$点在边$AY$上 110 | 的位置如何(图6.3),以上 111 | 的四个比值都是不变的,只有 112 | 当$\angle A$变化时,这些比值才随着变化.这四个比都叫做锐角$A$的三角比. 113 | 114 | 有了以上定义,我们就在直角三角形的角与边之间建立 115 | 了联系,知道了角的大小,相应的四个三角比就被唯一地确 116 | 定了.反过来,如果我们知道了一个角的四个三角比中的任 117 | 何一个,我们也就能确定这个角的大小. 118 | 119 | \begin{example} 120 | 在直角$\triangle ABC$中,$\angle C=90^{\circ}$, $\overline{BC}=3$cm, $\overline{AC}= 121 | 4$cm, 求$\angle A$的4个三角比(图6.4). 122 | \end{example} 123 | 124 | \begin{figure}[htp]\centering 125 | \begin{minipage}[t]{0.48\textwidth} 126 | \centering 127 | \begin{tikzpicture}[>=latex, scale=1] 128 | \draw(0,0)node[left]{$B$}--node[below]{3}(3,0)node[right]{$C$}--node[right]{4}(3,4)node[above]{$A$}--node[left]{5}(0,0); 129 | \draw(3,0) rectangle (3-.2,.2); 130 | \draw(3,4-.5) arc (-90:-36.9-90:.5); 131 | 132 | \end{tikzpicture} 133 | \caption{} 134 | \end{minipage} 135 | \begin{minipage}[t]{0.48\textwidth} 136 | \centering 137 | \begin{tikzpicture}[>=latex, scale=.8] 138 | \draw(40:7)node[right]{$Y$}--(0,0)node[left]{$A$}--(5,0)node[right]{$X$}; 139 | \draw(4.2,0)node[below]{$C$}--node[right]{$1.8$}(4.2,3.56)node[above]{$B$}; 140 | \node at (40:3)[left]{$3$}; 141 | \node at (2.1,0)[below]{$2.1$}; 142 | \draw(4.2,0) rectangle (4,.2); 143 | \draw(.5,0) arc (0:40:.5); 144 | \end{tikzpicture} 145 | \caption{} 146 | \end{minipage} 147 | \end{figure} 148 | 149 | \begin{solution} 150 | 根据勾股定理, 151 | \[ \overline{AB}=\sqrt{\overline{BC}^2+\overline{AC}^2}=\sqrt{3^2+4^2}=5{\rm (cm)}\] 152 | 根据各三角比的定义有, 153 | \[\begin{split} 154 | \sin A=\frac{\overline{BC}}{\overline{AB}}=\frac{3}{5},&\qquad \cos A=\frac{\overline{AC}}{\overline{AB}}=\frac{4}{5}\\ 155 | \tan A=\frac{\overline{BC}}{\overline{AC}}=\frac{3}{4},&\qquad \cot A=\frac{\overline{AC}}{\overline{BC}}=\frac{4}{3}\\ 156 | \end{split}\] 157 | \end{solution} 158 | 159 | 160 | \begin{example} 161 | 求$40^{\circ}$角的四个三角比. 162 | \end{example} 163 | 164 | \begin{solution} 165 | 用量角器画$\angle XAY=40^{\circ}$(图6.5). 在边$AY$上截 166 | 取$\overline{AB}=3$cm(为计算方便,我们尽量取整数),作$\overline{BC}\bot AY$于$C$点,量得$\overline{BC}=1.8$cm, $\overline{AC}=2.1$cm, 在直角 167 | $\triangle ABC$中,根据三角比的定义可得: 168 | \[\sin40^{\circ}=\frac{1.8}{3}\approx 0.6,\qquad \cos40^{\circ}=\frac{2.1}{3}\approx 0.7 \] 169 | \[\tan40^{\circ}=\frac{1.8}{2.1}\approx 0.8,\qquad \cot40^{\circ}=\frac{2.1}{1.8}\approx 1.1\] 170 | \end{solution} 171 | 172 | \begin{example} 173 | 已知$\tan A=\frac{1}{2}$, 求$\angle A$. 174 | \end{example} 175 | 176 | \begin{figure}[htp] 177 | \centering 178 | \begin{tikzpicture} 179 | \draw(0,0)node[left]{$A$}--(4,0)node[right]{$C$}--(4,2)node[above]{$B$}--(0,0); 180 | \draw (4,0)rectangle(4-.2,.2); 181 | \end{tikzpicture} 182 | \caption{} 183 | \end{figure} 184 | 185 | \begin{solution} 186 | 作一个直角$\triangle ABC$, 使直角边$\overline{AC}=2$个单位长, 187 | $\overline{CB}=1$个单位长(图6.6),于是, 188 | \[\tan A=\frac{1}{2}\] 189 | 用量角器量$\angle A$, 得之$\angle A\approx 26^{\circ}$. 190 | \end{solution} 191 | 192 | \begin{ex} 193 | \begin{enumerate} 194 | \item 已知直角$\triangle ABC$, $\angle C=90^{\circ}$, $\overline{BC}=5$个单位长,$\overline{AC}= 195 | 12$个单位长,求$\angle A$与$\angle B$的四个三角比. 196 | \item 用作图法求出表中各角的四个三角比的近似值,填入表 197 | 中: 198 | \begin{center} 199 | \begin{tabular}{c|cccc} 200 | \hline 201 | $\alpha$ & $20^{\circ}$ & $40^{\circ}$ & $50^{\circ}$ & $80^{\circ}$\\ 202 | \hline 203 | $\sin\alpha$\\ 204 | $\cos\alpha$\\ 205 | $\tan\alpha$\\ 206 | $\cot\alpha$\\ 207 | \hline 208 | \end{tabular} 209 | \end{center} 210 | \item 已知 $\sin\alpha=\frac{3}{4}$, 211 | 用作图法求$\angle\alpha$. 212 | \item 已知$\cos\beta=\frac{2}{5}$, 213 | 用作图法求$\angle \beta$. 214 | \item 已知$\tan A=\frac{3}{5}$, 215 | 用作图法求$\angle A$. 216 | \end{enumerate} 217 | \end{ex} 218 | 219 | \subsection{$0^{\circ}$到$90^{\circ}$角的三角比的变化} 220 | 半径等于1个单位长的圆叫做\textbf{单位圆}.下面我们利用单 221 | 位圆来研究锐角三角比的变化规律: 222 | 223 | \begin{figure}[htp] 224 | \centering 225 | \begin{tikzpicture}[>=latex,scale=1.5] 226 | \draw[->](-1.5,0)--(1.5,0)node[right]{$x$}; 227 | \draw[->](0,-1.5)--(0,2)node[right]{$y$}; 228 | 229 | \draw(0,0)node[below left]{$O$} circle (1); 230 | \draw(1,0)node[above right]{$A$}--(1,2)node[above]{$A'$}; 231 | \draw[very thick](0,0)--(60:1)node[below left]{$P$}--(60:2)node[right]{$Q$}; 232 | \draw[very thick](60:1)--(0.5,0)node[below]{$M$}; 233 | \draw[very thick](0,1)node[above left]{$B$}--(1,1)node[right]{$B'$}--(1,0)node[below right]{1}; 234 | \node at (.5,1)[above]{$T$}; 235 | \draw (.25,0) arc (0:60:.25)node[right]{$\alpha$}; 236 | \end{tikzpicture} 237 | \caption{} 238 | \end{figure} 239 | 240 | 241 | 画单位圆$\odot O$(图6.7), 通过单位圆的圆心$O$作互相 242 | 垂直的两条直线,其中一条是 243 | 水平的,另一条是铅直的,以 244 | $O$为原点,单位圆的半径为长 245 | 度单位,在两条直线上建立数 246 | 轴,其中水平轴向右为正,铅 247 | 直轴向上为正;水平轴用$x$表 248 | 示,又叫做$x$轴,铅直的轴用 249 | $y$表示,又叫做$y$轴.以$O$为 250 | 顶点,$x$轴的正方向为一边,作$\angle AOP$等于已知角$\alpha$, $\angle AOP$ 251 | 的两边分别与单位圆相交于$A$、$P$两点,过$P$点作$\overline{PM}\bot OA$于$M$点, 252 | 253 | $\because\quad \overline{OP}=1$ 254 | 255 | $\therefore\quad \sin\alpha=\frac{\overline{MP}}{\overline{OP}}=\overline{MP}\text{的量数},\quad \cos\alpha=\frac{\overline{OM}}{\overline{OP}}=\overline{OM}\text{的量数}$ 256 | 257 | 这样,对于任一锐角$\alpha$, 我们可直接用$\overline{MP}$和$\overline{OM}$的量 258 | 数来分别表示$\sin\alpha$, $\cos\alpha$的值.我们把$\overline{MP}$和$\overline{OM}$分别叫做 259 | 角$\alpha$的\textbf{正弦线}和\textbf{余弦线}.下面我们用正弦线和余弦线来研究 260 | $\sin\alpha$与$\cos\alpha$的变化规律. 261 | 262 | \begin{itemize} 263 | \item 当$\alpha=0^{\circ}$时,$\overline{MP}=0$, 264 | $\overline{OM}=1$ 265 | \item 当$\alpha=90^{\circ}$时,$\overline{MP}=1$, 266 | $\overline{OM}=0$ 267 | \end{itemize} 268 | 我们就说, 269 | \[\sin0^{\circ}=0,\qquad 270 | \cos0^{\circ}=1,\qquad 271 | \sin90^{\circ}=1,\qquad 272 | \cos90^{\circ}=0.\] 273 | 我们使角$\alpha$从$0^{\circ}$逐渐增加到$90^{\circ}$, 于是从角$\alpha$的正弦线 274 | 和余弦线的变化规律可以看到,\textbf{当 275 | $\alpha$增大时,$\sin\alpha$随着增 276 | 大,而$\cos\alpha$随着减小;反之,当$\alpha$减小时,$\sin\alpha$随着减小, 277 | 而$\cos\alpha$随着增大.} 278 | 279 | 在图6.7中,过$A$点作$AA'\bot OA$, 与角$\alpha$的一边$OP$相 280 | 交于$Q$点,于是, 281 | \[\tan\alpha=\frac{\overline{AQ}}{\overline{OA}}=\overline{AQ}\text{的量数}\] 282 | $\overline{AQ}$叫做角$\alpha$的\textbf{正切线}. 283 | 284 | 在图6.7中,过单位圆与$y$轴的交点$B$作$BB'\bot OB$, 角 285 | $\alpha$的一边$OP$与$BB'$相交于$T$点,于是, 286 | \[\cot\alpha=\frac{\overline{BT}}{\overline{OB}}=\overline{BT}\text{的量数}\] 287 | $\overline{BT}$叫做角$\alpha$的\textbf{余切线}. 288 | 289 | 下面我们用正切线和余切线来说明角$\alpha$的正切和余切 290 | 随着角$\alpha$的变化规律. 291 | 292 | \begin{itemize} 293 | \item 当$\alpha =0^{\circ}$时,$\overline{AQ}=0$, 边$OP$与$BB'$不相交,我们就说$\tan0^{\circ}=0$, 294 | $\cot 0^{\circ}$不存在. 295 | \item 当$\alpha =90^{\circ}$时,$AQ$与$OP$不相交,$\overline{BT}=0$, 296 | 我们就说, 297 | $\tan90^{\circ}$不存在, 298 | $\cot 90^{\circ}=0$. 299 | \end{itemize} 300 | 301 | 我们使角$\alpha$从$0^{\circ}$增加到$90^{\circ}$, 于是从角$\alpha$的正切线和余 302 | 切线的变化规律可以看到,\textbf{当$\alpha$增大时,$\tan\alpha$也随着增大, 303 | 而$\cot\alpha$测随着减小.反之当$\alpha$减小时,$\tan\alpha$也随着减小, 304 | 而$\cot\alpha$则随着增大.} 305 | 306 | 307 | \begin{ex} 308 | \begin{enumerate} 309 | \item 在横线上填入不等号($\alpha$、$\beta$都是锐角). 310 | \begin{enumerate} 311 | \item 当$\alpha>\beta$时,$\sin\alpha\underline{\quad}\sin\beta$,$\cos\alpha\underline{\quad}\cos\beta$, 312 | $\tan\alpha\underline{\quad}\tan\beta$,$\cot\alpha\underline{\quad}\cot\beta$. 313 | \item 当$\alpha<\beta$时,$\sin\alpha\underline{\quad}\sin\beta$,$\cos\alpha\underline{\quad}\cos\beta$, 314 | $\tan\alpha\underline{\quad}\tan\beta$,$\cot\alpha\underline{\quad}\cot\beta$. 315 | \end{enumerate} 316 | 317 | \item 指出下列差的符号: 318 | \begin{multicols}{2} 319 | \begin{enumerate} 320 | \item $\sin34^{\circ}-\sin33^{\circ}$ 321 | \item $ \sin27^{\circ}-\sin26^{\circ}$ 322 | \item $\cos83^{\circ}-\cos84^{\circ}$ 323 | \item $\cos10^{\circ}-\cos9^{\circ}$ 324 | \item $\tan 5^{\circ}-\tan 6^{\circ}$ 325 | \item $\cot 14^{\circ}-\cot 13^{\circ}$ 326 | \item $\tan 46^{\circ}-\tan 44^{\circ}$ 327 | \item $\cot 44^{\circ}-\cot 47^{\circ}$ 328 | \end{enumerate} 329 | \end{multicols} 330 | \end{enumerate} 331 | \end{ex} 332 | 333 | \subsection{$30^{\circ}$、$45^{\circ}$、$60^{\circ}$角的三角比} 334 | 我们根据锐角三角比的定义和直角三角形中的一些边角 335 | 特殊关系,可以计算出$30^{\circ}$、$45^{\circ}$、$60^{\circ}$角的三角比的精确 336 | 值. 337 | 338 | 作$\triangle ABC$, 使$\angle C=90^{\circ}$, 339 | $\angle A=30^{\circ}$ (图6.8), 那么 340 | $\angle B=60^{\circ}$, 设$\overline{BC}=a$, 则$\overline{AB}=2a$, $\overline{AC}=\sqrt{\overline{AB}^2-\overline{BC}^2}= 341 | \sqrt{(2a)^2-a^2}=\sqrt{3}a$. 342 | 由此得: 343 | \[\begin{split} 344 | \sin 30^{\circ}=\frac{a}{2a}=\frac{1}{2},&\qquad \sin 60^{\circ}=\frac{\sqrt{3}a}{2a}=\frac{\sqrt{3}}{2}\\ 345 | \cos 30^{\circ}=\frac{\sqrt{3}a}{2a}=\frac{\sqrt{3}}{2},&\qquad \cos 60^{\circ}=\frac{a}{2a}=\frac{1}{2}\\ 346 | \end{split}\] 347 | \[\begin{split} 348 | \tan 30^{\circ}=\frac{a}{\sqrt{3}a}=\frac{1}{\sqrt{3}}=\frac{\sqrt{3}}{3},&\qquad \tan 60^{\circ}=\frac{\sqrt{3}a}{a}=\sqrt{3}\\ 349 | \cot 30^{\circ}=\frac{\sqrt{3}a}{a}=\sqrt{3},&\qquad \cot 60^{\circ}=\frac{a}{\sqrt{3}a}=\frac{1}{\sqrt{3}}=\frac{\sqrt{3}}{3}\\ 350 | \end{split}\] 351 | 352 | \begin{figure}[htp]\centering 353 | \begin{minipage}[t]{0.48\textwidth} 354 | \centering 355 | \begin{tikzpicture}[>=latex, scale=1] 356 | \draw(0,0)node[left]{$A$}--node[below]{$\sqrt{3}a$}(2*1.732,0)node[right]{$C$}--node[right]{$a$}(2*1.732,2)node[above]{$B$}--node[left]{$2a$}(0,0); 357 | \draw(2*1.732,0)rectangle (2*1.732-.2,.2); 358 | \end{tikzpicture} 359 | \caption{} 360 | \end{minipage} 361 | \begin{minipage}[t]{0.48\textwidth} 362 | \centering 363 | \begin{tikzpicture}[>=latex, scale=1] 364 | \draw(0,0)node[left]{$A$}--node[below]{$a$}(3,0)node[right]{$C$}--node[right]{$a$}(3,3)node[above]{$B$}--node[left]{$\sqrt{2}a$}(0,0); 365 | \draw(3,0)rectangle (3-.2,.2); 366 | \end{tikzpicture} 367 | \caption{} 368 | \end{minipage} 369 | \end{figure} 370 | 371 | 作$\triangle ABC$, 使$\angle C=90^{\circ}$, $\angle A=45^{\circ}$ (图6.9), 那么, 372 | $\angle B=45^{\circ}$, 设$\overline{BC}=a$, 则$\overline{AC}=a$, $\overline{AB}=\sqrt{a^2+a^2}=\sqrt{2}a$ 373 | 374 | 由此得: 375 | \[\sin 45^{\circ}=\frac{a}{\sqrt{2}a}=\frac{1}{\sqrt{2}}=\frac{\sqrt{2}}{2},\qquad \cos 45^{\circ}=\frac{a}{\sqrt{2}a}=\frac{1}{\sqrt{2}}=\frac{\sqrt{2}}{2}\] 376 | \[\tan 45^{\circ}=\frac{a}{a}=1,\qquad \cot 45^{\circ}=\frac{a}{a}=1\] 377 | 为了便于记忆,我们把$30^{\circ},45^{\circ},60^{\circ}$的三角比列成表. 378 | \begin{center} 379 | \begin{tabular}{c|ccc} 380 | \hline 381 | $\alpha$&$30^{\circ}$&$45^{\circ}$&$60^{\circ}$\\ 382 | \hline 383 | $\sin\alpha$ & $\frac{1}{2}$ & $\frac{\sqrt{2}}{2}$& $\frac{\sqrt{3}}{2}$\\ 384 | $\cos\alpha$ & $\frac{\sqrt{3}}{2}$ & $\frac{\sqrt{2}}{2}$& $\frac{1}{2}$\\ 385 | $\tan\alpha$ & $\frac{\sqrt{3}}{3}$ &1&$\sqrt{3}$\\ 386 | $\cot\alpha$ & $\sqrt{3}$ &1&$\frac{\sqrt{3}}{3}$\\ 387 | \hline 388 | \end{tabular} 389 | \end{center} 390 | 391 | \begin{example} 392 | 计算 $4\cot 30^{\circ}-2\sin60^{\circ}+2\cos60^{\circ}$ 393 | \end{example} 394 | 395 | \begin{solution} 396 | \[4\cot 30^{\circ}-2\sin60^{\circ}+2\cos60^{\circ} =4\x \sqrt{3}-2\x \frac{\sqrt{3}}{2}+2\x \frac{1}{2}=3\sqrt{3}+1 397 | \] 398 | \end{solution} 399 | 400 | 401 | \begin{example} 402 | 计算 403 | $\cos^2 30^{\circ}+\sin^2 45^{\circ}-\tan^2 45^{\circ}$ 404 | 其中:$\sin^\alpha$、$\cos^2\alpha$ 405 | $\tan^2\alpha$、$\cot^2\alpha$分别表示$(\sin\alpha)^2$、$(\cos \alpha)^2$、$(\tan\alpha)^2$、$(\cot \alpha)^2$ 406 | \end{example} 407 | 408 | 409 | \begin{solution} 410 | \[\cos^2 30^{\circ}+\sin^2 45^{\circ}-\tan^2 45^{\circ}=\left(\frac{\sqrt{3}}{2}\right)^2+\left(\frac{\sqrt{2}}{2}\right)^2-1^2=\frac{3}{4}+\frac{2}{4}-1=\frac{1}{4}\] 411 | \end{solution} 412 | 413 | \begin{ex} 414 | 求下列各式之值: 415 | \begin{enumerate} 416 | \item $\sin^2 60^{\circ}+\cos^2 30^{\circ}$ 417 | \item $\sin^2 60^{\circ}+\cos^2 60^{\circ}$ 418 | \item $\sin^2 45^{\circ}+\cos^2 45^{\circ}$ 419 | \item $2\sin30^{\circ}+2\cos60^{\circ}+4\tan 45^{\circ}$ 420 | \item $5\tan 30^{\circ}+\cot 45^{\circ}-2\tan 45^{\circ}+2\cos 60^{\circ}$ 421 | \item $\frac{2\sin30^{\circ}}{2\cos30^{\circ}-1}$ 422 | \item $\frac{\sin60^{\circ}-\sin30^{\circ}}{\sin60^{\circ}+\sin30^{\circ}}$ 423 | \end{enumerate} 424 | \end{ex} 425 | 426 | \subsection{三角比值表} 427 | 在前面的内容中,我们讲的只是特殊角的三角比,为了应用方 428 | 便,人们早已制定了任意锐角的三角比值表,下面就来介绍 429 | 四位三角比值表的用法. 430 | 431 | \subsubsection{正弦表,余弦表} 432 | 在正弦、余弦表里左右各有一列排度数,左列上端和右 433 | 列下端都有$A$字,在左列$A$的下面,由上到下排着度数, 434 | 在右列$A$的上面,由下到上排着度数,在顶行$A$的右边, 435 | 由左至右依次排着$0',6',\ldots,60'$, 在表的顶上写着正弦表, 436 | 说明查正弦时用左列$A$下面的度数和顶行的分数,表的底下 437 | 写着余弦表,说明查余弦时,用右列$A$上的度数和底行的分 438 | 数.例如要查$26^{\circ}18'$的正弦,在表中左列$A$的下面先找到 439 | $26^{\circ}$, 顺着$26^{\circ}$所在的这一行往右,在顶行$18'$所在的这一列 440 | 里找到了一个数0.4431, 就是$26^{\circ}18'$的正弦,即$\sin26^{\circ}18'= 441 | 0.4431$. 换句话说左列$A$下面的$26^{\circ}$所在的行和顶行$18'$所 442 | 在的列的交点处的0.4431, 就是$\sin26^{\circ}18'$. 要查$\cos27^{\circ}24'$, 443 | 在表中右列$A$的上面找到$27^{\circ}$, 底行里找到$24'$, $27^{\circ}$所在的 444 | 行和$24'$所在的列的交点处的0.8878, 就是$\cos27^{\circ}24'$, 即 445 | $\cos27^{\circ}24'=0.8878$. 446 | 447 | \subsubsection{正切表、余切表} 448 | 正切的查法和正弦相同,余切的查法和余弦相同,例如 449 | 我们在正切表、余切表中可以查到: 450 | \[\tan 54^{\circ}30'=1.4019,\qquad \cot 4^{\circ}6'=13.95\] 451 | 452 | \begin{ex} 453 | 查表求下列各三角比: 454 | \begin{enumerate} 455 | \item $\sin14^{\circ},\quad \sin20^{\circ}24',\quad \sin65^{\circ}30',\quad \sin82^{\circ}12'$ 456 | \item $\cos7^{\circ},\quad \cos32^{\circ}6',\quad \cos60^{\circ}54',\quad \cos83^{\circ}18'$ 457 | \item $\tan 18^{\circ},\quad \tan 78^{\circ}36',\quad \tan 80^{\circ}24',\quad \tan 83^{\circ}$ 458 | \item $\cot 42^{\circ}42',\quad \cot20^{\circ}48', \quad \cot9^{\circ}36', \quad \cot 5^{\circ}30'$ 459 | \end{enumerate} 460 | \end{ex} 461 | 462 | 在三角比值表中,最右边的三列是修正值,它是用来求 463 | 在左边表里找不到的角的三角比,这三列的上端和下端都标 464 | 有$1'$、$2'$、$3'$, 三列中的数是小数的简写,每一个数都代表 465 | 一个小数,它的末位数相当于表中间同一行小数的末位数, 466 | $1'$、$2'$、$3'$各列中的各数,分别是它所在的行的角度分别相 467 | 差$1'$、$2'$、$3'$时的三角比的修正值,下面举列说明查法: 468 | 469 | 例如,要求$\sin20^{\circ}19'$, 先从表中查得$\sin20^{\circ}18'$的值是 470 | 0.3469, 因为$20^{\circ}19'$比$20^{\circ}18'$大$1'$, 查修正值是0.0003. 因 471 | 为角度大,它的正弦值也大,所以$\sin20^{\circ}19'$就比$\sin20^{\circ}18'$ 472 | 大0.0003, 因此,$\sin20^{\circ}19'=0.3469+0.0003=0.3472$. 要 473 | 求$\sin20^{\circ}46'$, 先从表中查得$\sin20^{\circ}48'$的值是0.3551, $20^{\circ}46'$ 474 | 比$20^{\circ}48'$小$2'$, 查得$2'$的修正值是0.0005, 角度小,正弦值 475 | 也小,所以$\sin20^{\circ}46'=0.3551-0.0005=0.3546$. 要求 476 | $\cos28^{\circ}26'$, 先从表中查得$\cos28^{\circ}24'=0.8796$, $28^{\circ}26'$比 477 | $28^{\circ}24'$大$2'$, 查表得修正值是0.0003, 角大余弦值反而小, 478 | 所以$\cos28^{\circ}26'=0.8796-0.0003=0.8793$. 479 | 480 | 正切的查法和正弦相同,余切的查法和余弦相同. 481 | 482 | 例如,我们可以求得 483 | \[\tan 69^{\circ}25'=2.662,\qquad \cot 70^{\circ}45'=0.3492\] 484 | 485 | \begin{ex} 486 | 查表求下列各三角函数: 487 | \begin{multicols}{2} 488 | \begin{enumerate} 489 | \item $\sin18^{\circ}19',\quad \sin63^{\circ}40'$ 490 | \item $\cos65^{\circ}2',\quad \cos10^{\circ}34'$ 491 | \item $\tan 9^{\circ}19',\quad \tan64^{\circ}10'$ 492 | \item $\cot25^{\circ}28',\quad \cot10^{\circ}25'$ 493 | \end{enumerate} 494 | \end{multicols} 495 | \end{ex} 496 | 497 | 从三角比值表里不但可以查得任何锐角的三角比,反过 498 | 来,也可以根据已知的三角比值查到未知的锐角. 499 | 500 | \begin{example} 501 | 已知$\sin x=0.9966$, 求锐角 502 | $x$. 503 | \end{example} 504 | 505 | \begin{solution} 506 | 在正弦表里找到0.9966, 因为它是正弦的值,要用到左 507 | 列的度和顶行的分,在0.9966这一行的左端是$85^{\circ}$, 在0.5966 508 | 的上端是$18'$. 所以 509 | $ \sin85^{\circ}18'=0.9966$, 因此$x=85^{\circ}18'$. 510 | \end{solution} 511 | 512 | \begin{example} 513 | 已知$\cos y=0.9966$, 求锐角$y$. 514 | \end{example} 515 | 516 | \begin{solution} 517 | 在表中找到0.9966, 因为它是余弦的值,余弦要用到右 518 | 列的度数和底行的分,在0.9966这一行的右端是$4^{\circ}$, 在 519 | 0.9966这一列的下端是$42'$. 520 | 521 | 所以$\cos4^{\circ}42'=0.9966$, 因此$y=4^{\circ}42'$. 522 | \end{solution} 523 | 524 | \begin{example} 525 | $\tan x=14.30$, $\cot y=1.4715$, 求$x$、$y$. 526 | \end{example} 527 | 528 | \begin{solution} 529 | 倒查正切表(查法和例6.6相同)可得:$x=86^{\circ}$. 530 | 531 | 倒查余切表(查法和例6.7相同)可得:$y=34^{\circ}12'$. 532 | \end{solution} 533 | 534 | 由三角比值求角度,有时要用到修正值,用修正值时, 535 | 必须注意到,对于正弦、正切的值越大,角度也越大;对于 536 | 余弦、余切的值越大,角度反而越小,下面举例说明用修正 537 | 值查法. 538 | 539 | \begin{example} 540 | $\sin x=0.2493$, 求$x$. 541 | \end{example} 542 | 543 | \begin{solution} 544 | 在正弦表里,和0.2493最接近的正弦值是0.2487, 它是 545 | $14^{\circ}24'$的正弦,$0.2493-0.2487=0.0006$, 在$14^{\circ}$这一行里 546 | 正弦值相差0.0006时,角度的修正值是$2'$, $\sin x$比$\sin14^{\circ}24'$ 547 | 大0.0006, $x$就比$14^{\circ}24'$大$2'$, 因此 548 | \[x=14^{\circ}24'+2'=14^{\circ}26'\] 549 | \end{solution} 550 | 551 | \begin{example} 552 | $\cos y=0.9841$, 求$y$. 553 | \end{example} 554 | 555 | \begin{solution} 556 | 在余弦表中和0.9841最接近的余弦值是0.9842, 它是 557 | $10^{\circ}12'$的余弦,$0.9842-0.9841=0.0001$, 在$10^{\circ}$这一行里 558 | 余弦值相差0.0001时,角度的修正值是$1'$或$2'$, $\cos y$比 559 | $\cos10^{\circ}12'$小0.0001, $y$比$12^{\circ}12'$大$1'$或$2'$, 因此 560 | \[y=10^{\circ}12'+1'=10^{\circ}13'\] 561 | 或\[y=10^{\circ}12'+2'=10^{\circ}14'\] 562 | 这里$y$有两个答案, 563 | 一个是不足近似值,一个过剩近似值. 564 | \end{solution} 565 | 566 | \begin{example} 567 | $\tan x=1.3773$, $\cot=0.1950$, 求$x$、$y$. 568 | \end{example} 569 | 570 | \begin{solution} 571 | 倒查正切表,得$x=54^{\circ}1'$(查法和例6.9相同). 572 | 573 | 倒查余切表,得$y=78^{\circ}51'$(查法和例6.10相同). 574 | \end{solution} 575 | 576 | \begin{ex} 577 | 由三角比值表求锐角$x$: 578 | \begin{enumerate} 579 | \item $\sin x=0.9816,\quad 580 | \sin x=0.6639,\quad 581 | \tan x=9.595,\quad 582 | \tan x=0.1890$ 583 | \item $\cos x=0.8607,\quad 584 | \cos x=0.9893,\quad 585 | \cot x=2.106,\quad 586 | \cot x=67.40$ 587 | \item $\sin x=0.2476,\quad 588 | \sin x=0.9709,\quad 589 | \cos x=0.3372$ 590 | 591 | $ 592 | \cos x=0.4174,\quad 593 | \tan x=0.365,\quad 594 | \cot x=0.1614$ 595 | \end{enumerate} 596 | \end{ex} 597 | 598 | \subsection{互为余角的三角比间的关系} 599 | \begin{figure}[htp] 600 | \centering 601 | \begin{tikzpicture}[>=latex, scale=.8] 602 | \draw(0,0)node[left]{$A$}--node[below]{$b$}(2,0)node[right]{$C$}--node[right]{$a$}(2,2*1.732)node[above]{$B$}--node[left]{$c$}(0,0); 603 | \draw(2,0)rectangle (2-.2,.2); 604 | \end{tikzpicture} 605 | \caption{} 606 | \end{figure} 607 | 608 | 在直角$\triangle ABC$中,如果$\angle C=90^{\circ}$(图6.10), 则 609 | \[\angle A+\angle B=90^{\circ},\qquad \angle B=90^{\circ}-\angle A\] 610 | 由于$\angle A$的对边是$\angle B$的邻边, 611 | $\angle B$的对边是$\angle A$的邻边,那么,根 612 | 据三角比的定义,我们便得出$\angle A$和 613 | $\angle B$这两个互为余角的三角比之间有 614 | 下面的关系: 615 | \[\begin{split} 616 | \sin A=\frac{a}{c}=\cos B=\cos(90^{\circ}-A),&\qquad \cos A=\frac{b}{c}=\sin B=\sin(90^{\circ}-A)\\ 617 | \tan A=\frac{a}{b}=\cot B=\cot(90^{\circ}-A),&\qquad \cot A=\frac{b}{a}=\tan B=\tan(90^{\circ}-A)\\ 618 | \end{split}\] 619 | 620 | 这就是说,\textbf{互为余角的两个角中,任一角的正弦等于另 621 | 一角的余弦;任一角的正切等于另一角的余切.} 622 | 623 | 有了这个关系,我们就可把任意大于$45^{\circ}$的锐角的三角 624 | 比化为小于$45^{\circ}$的锐角的三角比. 625 | 626 | \begin{example} 627 | 把下面各角的三角比化为小于$45^{\circ}$的锐角的三角 628 | 比. 629 | \begin{multicols}{2} 630 | \begin{enumerate} 631 | \item $\sin75^{\circ}$ 632 | \item $\cos62^{\circ}22'$ 633 | \item $\tan 80^{\circ}$ 634 | \item $\cot56^{\circ}18'$ 635 | \end{enumerate} 636 | \end{multicols} 637 | \end{example} 638 | 639 | \begin{solution} 640 | \begin{enumerate} 641 | \item $\sin75^{\circ}=\cos(90^{\circ}-75^{\circ})=\cos 15^{\circ}$ 642 | \item $\cos62^{\circ}22'=\sin(90^{\circ}-62^{\circ}22')=\sin27^{\circ}38'$ 643 | \item $\tan 80^{\circ}=\cot(90^{\circ}-80^{\circ})=\cot10^{\circ}$ 644 | \item $\cot 56^{\circ}18'=\tan(90^{\circ}-56^{\circ}18')=\tan 33^{\circ}42'$ 645 | \end{enumerate} 646 | \end{solution} 647 | 648 | 649 | \begin{example} 650 | 下列等式是否成立: 651 | \begin{enumerate} 652 | \item $\sin(60^{\circ}+\alpha )=\cos(30^{\circ}-\alpha ) \qquad (0\le \alpha \le 30^{\circ})$ 653 | \item $\sin(45^{\circ}+\alpha )=\cos(45^{\circ}-\alpha ) \qquad (0\le \alpha \le 45^{\circ})$ 654 | \item $\tan(50^{\circ}+\alpha )=\cot (40^{\circ}-\alpha ) \qquad (0\le \alpha <40^{\circ})$ 655 | \end{enumerate} 656 | \end{example} 657 | 658 | 659 | \begin{solution} 660 | 由于 661 | \[\begin{split} 662 | (60^{\circ}+\alpha )+(30^{\circ}-\alpha )&=90^{\circ}\\ 663 | (45^{\circ}+\alpha )+(45^{\circ}-\alpha )&=90^{\circ}\\ 664 | (50^{\circ}+\alpha )+(40^{\circ}-\alpha )&=90^{\circ} 665 | \end{split}\] 666 | 667 | 而$\alpha $角的取值范围使得各式中的三角比都有意义,所以 668 | 根据互为余角的三角比之间的关系可知,各等式都成立. 669 | \end{solution} 670 | 671 | \begin{ex} 672 | \begin{enumerate} 673 | \item 为什么三角比值表中,锐角$\alpha$的正弦和$90^{\circ}-\alpha$角的余弦共 674 | 用一个表. 675 | \item 把下列各角的三角比化为小于$45^{\circ}$的锐角的三角比. 676 | \begin{multicols}{2} 677 | \begin{enumerate} 678 | \item $\sin73^{\circ},\qquad \sin77^{\circ}18'$ 679 | \item $\cos57^{\circ},\qquad \cos52^{\circ}38'$ 680 | \item $\tan78^{\circ},\qquad \tan79^{\circ}5'$ 681 | \item $\cot48^{\circ},\qquad \cot78^{\circ}31'$ 682 | \end{enumerate} 683 | \end{multicols} 684 | \item 下列各式中的$x$应为多少度? 685 | \begin{multicols}{2} 686 | \begin{enumerate} 687 | \item $\sin75^{\circ}=\cos x$ 688 | \item $\cos18^{\circ}=\sin x$ 689 | \item $\tan5^{\circ}=\cot x$ 690 | \item $\cot83^{\circ}=\tan x$ 691 | \end{enumerate} 692 | \end{multicols} 693 | \item 下列等式是否成立($x$、$\alpha$的取值都使各三角比有意义). 694 | \begin{multicols}{2} 695 | \begin{enumerate} 696 | \item $\sin(75^{\circ}+\alpha )=\cos(15^{\circ}-\alpha )$ 697 | \item $\sin(15^{\circ}-\alpha )=\sin(30^{\circ}+\alpha )$ 698 | \item $ \tan (30^{\circ}+x)=\cot (60^{\circ}-x)$ 699 | \item $ \cot (89^{\circ}+\alpha )=\tan (1^{\circ}-\alpha )$ 700 | \end{enumerate} 701 | \end{multicols} 702 | \end{enumerate} 703 | \end{ex} 704 | 705 | \subsection{同一锐角的各三角比间的关系} 706 | \begin{blk}{定理} 707 | 同一锐角$\alpha$ 的四个三角比之间有下列关系: 708 | \begin{enumerate} 709 | \item $\sin^2\alpha +\cos^2\alpha =1$ 710 | \item $\tan\alpha=\frac{\sin\alpha}{\cos\alpha },\qquad \cot\alpha=\frac{\cos\alpha }{\sin\alpha }$ 711 | \item $\tan\alpha\cdot \cot\alpha =1$ 712 | \end{enumerate} 713 | \end{blk} 714 | 715 | \begin{proof} 716 | 作直角$\triangle ABC$, 使$\angle C=90^{\circ}$, $\angle A=\alpha$ (图6.11). 717 | \begin{figure}[htp] 718 | \centering 719 | \begin{tikzpicture}[>=latex, scale=1.2] 720 | \draw(0,0)node[left]{$A$}--node[below]{$b$}(2,0)node[right]{$C$}--node[right]{$a$}(2,1.5)node[above]{$B$}--node[left]{$c$}(0,0); 721 | \draw(2,0)rectangle (2-.2,.2); 722 | \draw (.5,0) arc (0:36.87:.5)node[right]{$\alpha$}; 723 | \end{tikzpicture} 724 | \caption{} 725 | \end{figure} 726 | 727 | \begin{enumerate} 728 | \item $\because\quad a^2+b^2=c^2$ 729 | 730 | $\therefore\quad \left(\frac{a}{c}\right)^2+\left(\frac{b}{c}\right)^2=1$ 731 | 732 | 又$\because\quad \frac{a}{c}=\sin\alpha,\quad \frac{b}{c}=\cos\alpha$, 733 | 734 | $\therefore\quad \sin^2\alpha +\cos^2\alpha =1$,即: 735 | \textbf{一锐角的正弦和余弦的平方和等于1.} 736 | 该式也就是勾股定理的三角比的表示式. 737 | 738 | \item $\because\quad \tan\alpha=\frac{a}{b},\quad \frac{\sin\alpha}{\cos\alpha}=\frac{\frac{a}{c}}{\frac{b}{c}}=\frac{a}{b}$, 739 | 740 | $\therefore\quad \tan\alpha=\frac{\sin\alpha}{\cos\alpha}$ 741 | 742 | 又$\because\quad \cot\alpha=\frac{b}{a},\quad \frac{\cos\alpha}{\sin\alpha}=\frac{\frac{b}{c}}{\frac{a}{c}}=\frac{b}{a}$, 743 | 744 | $\therefore\quad \cot\alpha=\frac{\cos\alpha}{\sin\alpha}$ 745 | 746 | \item $\because\quad \tan\alpha=\frac{a}{b},\quad \cot\alpha=\frac{b}{a}$ 747 | 748 | $\therefore\quad \tan\alpha\cdot \cot\alpha=\frac{a}{b}\cdot \frac{b}{a}=1$ 749 | \end{enumerate} 750 | \end{proof} 751 | 752 | 上面的定理,表示了同一锐角的三角比之间的关系,我 753 | 们可利用定理中的三个公式,由已知锐角的一个三角比,去 754 | 计算这个角的其它的三个三角比;也可以利用它们来化简含 755 | 有三角比的式子. 756 | 757 | 758 | \begin{example} 759 | 已知:$\sin\alpha=\frac{3}{5}$, 760 | 求$\alpha$角($\alpha$为锐角)的其它 761 | 的三个三角比. 762 | \end{example} 763 | 764 | 765 | \begin{solution} 766 | 从公式$\sin^2\alpha +\cos^2\alpha =1$ 可得 767 | \[ \cos\alpha =\pm\sqrt{1-\sin^2\alpha}\] 768 | 由于锐角的三角比都是正 769 | 数,所以根号前应取正号,把$\sin\alpha =\frac{3}{5}$ 770 | 代入上式,得 771 | \[\cos\alpha=\sqrt{1-\left(\frac{3}{5}\right)^2}=\sqrt{\frac{25-9}{5^2}}=\frac{4}{5}\] 772 | 根据$\tan\alpha=\frac{\sin\alpha}{\cos\alpha}$,可得 773 | \[\tan\alpha=\frac{\frac{3}{5}}{\frac{4}{5}}=\frac{3}{4}\] 774 | 再根据$\tan\alpha\cdot \cot\alpha=1$,可得 775 | \[\cot\alpha=\frac{4}{3}\] 776 | \end{solution} 777 | 778 | \begin{example} 779 | 化简 780 | $\sin^2 54^{\circ}+\sin^2 36^{\circ}-\tan 45^{\circ}$ 781 | \end{example} 782 | 783 | \begin{solution} 784 | \[\begin{split} 785 | \sin^2 54^{\circ}+\sin^2 36^{\circ}-\tan 45^{\circ}&= 786 | \cos^2(90^{\circ}-54^{\circ})+\sin^2 36^{\circ}-\tan 45^{\circ}\\ 787 | &=\cos^2 36^{\circ}+\sin^2 36^{\circ}-1\\ 788 | &=1-1=0 789 | \end{split}\] 790 | \end{solution} 791 | 792 | 793 | \begin{example} 794 | 化简$\frac{\sqrt{1-\sin^2\alpha}}{\sin\alpha}\cdot \tan\alpha$ 795 | \end{example} 796 | 797 | \begin{solution} 798 | \[\frac{\sqrt{1-\sin^2\alpha}}{\sin\alpha}\cdot \tan\alpha=\frac{\cos\alpha}{\sin\alpha}\cdot \tan\alpha=\cot\alpha\cdot \tan\alpha=1\] 799 | \end{solution} 800 | 801 | \begin{example} 802 | 化简$(\sin\alpha+\cos\alpha)^2+(\sin\alpha-\cos\alpha)^2$ 803 | \end{example} 804 | 805 | \begin{solution} 806 | \[\begin{split} 807 | & (\sin\alpha+\cos\alpha)^2+(\sin\alpha-\cos\alpha)^2\\ 808 | &\quad =\sin^2\alpha+2\sin\alpha\cdot \cos\alpha+\cos^2\alpha 809 | +\sin^2\alpha-2\sin\alpha\cdot\cos\alpha+\cos^2\alpha\\ 810 | &\quad =1+1=2 811 | \end{split}\] 812 | \end{solution} 813 | 814 | \begin{ex} 815 | \begin{enumerate} 816 | \item 证明$\sin^2\alpha+\cos^2\alpha=1$, $\tan\alpha=\frac{\sin\alpha}{\cos\alpha}$ 817 | \item 已知$\sin\alpha=\frac{5}{13}$,求$\cos\alpha, \tan\alpha, \cot\alpha$ 818 | \item 已知$\cos\alpha=\frac{4}{5}$,求$\sin\alpha, \tan\alpha, \cot\alpha$ 819 | \item 已知$\tan\alpha=\frac{3}{4}$,求$\sin\alpha,\cos\alpha$. 820 | 821 | 提示:$\tan^2\alpha =\frac{\sin^2\alpha}{1-\sin^2\alpha}=\frac{9}{16}$,令$\sin \alpha =x$,解方程. 822 | 823 | \item 化简 824 | 825 | \begin{enumerate}\begin{multicols}{2} 826 | \item $\frac{1-\sin^2\alpha}{\cos^2\alpha}$ 827 | \item $\tan\alpha\cdot \cos\alpha$ 828 | \item $\frac{\cos\alpha}{\sqrt{1-\cos^2\alpha}}$ 829 | \item $\frac{\sin^2\alpha}{1+\cos\alpha}$ 830 | \item $\frac{\cos^2\alpha}{1-\sin\alpha}$\end{multicols} 831 | \item $(\tan\alpha+\cot\alpha)^2-(\tan\alpha-\cot\alpha)^2$ 832 | \end{enumerate} 833 | 834 | \end{enumerate} 835 | \end{ex} 836 | 837 | \section*{习题6.1} 838 | \addcontentsline{toc}{subsection}{习题6.1} 839 | \begin{enumerate} 840 | \item 在直角$\triangle ABC$中,$\angle C=90^{\circ}$, $a=3$, $b=2$, 求$\angle A$的四 841 | 个三角比. 842 | \item 在直角$\triangle ABC$中,$\angle C=90^{\circ}$,$\overline{AB}=10$, $\overline{BC}=8$, 求: 843 | $ \sin A$、$\cos A$. 844 | \item 在直角$\triangle ABC$中,$\angle C=90^{\circ}$, $\overline{CD}$为$\overline{AB}$边上的高,问 845 | 图中哪些线段的比可以表 846 | 示$\angle A$的正弦?哪些线段 847 | 的比可以表示$\angle B$的正弦? 848 | \begin{figure}[htp] 849 | \centering 850 | \begin{tikzpicture} 851 | \draw(0,0)node[below]{$A$}--(5,0)node[below]{$B$}--(3.2,2.4)node[above]{$C$}--(0,0); 852 | \draw(3.2,2.4)--(3.2,0)node[below]{$D$}; 853 | \draw(3.2,0) rectangle (3.2+.2,.2); 854 | \draw (0.4,0) arc (0:32:.4); 855 | \draw (3.2,2) arc (-90:-90+32:.4); 856 | \end{tikzpicture} 857 | \caption*{第3题} 858 | \end{figure} 859 | 860 | \item 判断下列等式是否正确: 861 | \begin{enumerate} 862 | \item $\sin35^{\circ} 30'=\cos54^{\circ} 30'$ 863 | \item $\sin(30^{\circ} -\alpha )=\cos(60^{\circ} +\alpha )$ 864 | \item $\cos(2\alpha +14^{\circ} )=\sin(76^{\circ} -2\alpha )$ 865 | \end{enumerate} 866 | 867 | \item 化简下列各式: 868 | \begin{enumerate} 869 | \item $\sin^4\alpha +2\sin^2\alpha \cos^2\alpha +\cos^4\alpha $ 870 | \item $\sin^4\alpha -\cos^4\alpha +\cos^2\alpha $ 871 | \item $\sin(45^{\circ} +\alpha )-\cos(45^{\circ} -\alpha )$ 872 | \item $\frac{1-\sin^2\alpha }{1-\cos^2\alpha }$ 873 | \item $\frac{\tan \alpha +\tan \beta}{\cot\alpha +\cot \beta}$ 874 | \item $\sin(90^{\circ} -\alpha )\cdot \cot(90^{\circ} -\alpha )$ 875 | \end{enumerate} 876 | 877 | \item 已知 $\tan \alpha =2$, 求 $\tan(90^{\circ} -\alpha )$. 878 | \item 已知 879 | $\sin A=\frac{1}{3}$ 880 | 求 $\cos A$, $\tan A$. 881 | \item 已知互为余角的两角的正切的和等于3, 求两角正切的 882 | 平方和. 883 | \item 求下列各题中的未知锐角$x$: 884 | \begin{multicols}{2} 885 | \begin{enumerate} 886 | \item $\sin x=\frac{1}{2}$ 887 | \item $\cos x=\frac{\sqrt{2}}{2}$ 888 | \item $\tan x=\sqrt{3}$ 889 | \item $\sin x=\frac{\sqrt{3}}{2}$ 890 | \item $\cos x=\frac{1}{2}$ 891 | \item $\tan x=\frac{1}{\sqrt{3}}$ 892 | \end{enumerate} 893 | \end{multicols} 894 | 895 | \item 已知 $\cos^2 x=\frac{1}{4}$,求$x$. 896 | \item 已知 $\cos^2x-\sin^2x=\frac{1}{2}$, 897 | 求$x$. 898 | \item 回答下列问题: 899 | \begin{enumerate} 900 | \item 当$0^{\circ}\le \alpha\le 90^{\circ}$时,$\sin\alpha$的最大值是多少?最小值 901 | 是多少? 902 | \item 当$0^{\circ}\le \alpha\le 90^{\circ}$时,$\cos\alpha$的最大值是多少?最小值 903 | 是多少? 904 | \end{enumerate} 905 | 906 | 907 | \item 当$0^{\circ}\le \alpha\le 45^{\circ}$, $\alpha=45^{\circ}$, $45^{\circ}<\alpha\le 90^{\circ}$时,分别比较 908 | $\sin\alpha$与$\cos\alpha$的大小. 909 | \end{enumerate} 910 | 911 | \section{解直角三角形} 912 | \subsection{直角三角形中的边角关系} 913 | 914 | 我们把学过的直角三角形中 915 | 的边角基本关系总结如下: 916 | \begin{figure}[htp] 917 | \centering 918 | \begin{tikzpicture}[>=latex, scale=.8] 919 | \draw(0,0)node[left]{$A$}--node[below]{$b$}(2,0)node[right]{$C$}--node[right]{$a$}(2,2.5)node[above]{$B$}--node[left]{$c$}(0,0); 920 | \draw(2,0)rectangle (2-.2,.2); 921 | \end{tikzpicture} 922 | \caption{} 923 | \end{figure} 924 | 925 | 已知$\triangle ABC$, $\angle C=90^{\circ}$ (图6.12). 926 | \begin{enumerate} 927 | \item 勾股定理:$a^2+b^2=c^2$ 928 | \item 两锐角互余:$\angle A+\angle B=90^{\circ}$ 929 | \item 四个三角比: 930 | \[\sin A=\frac{\angle A\text{的对边}}{\angle A\text{斜边}},\qquad \cos A=\frac{\angle A\text{的邻边}}{\angle A\text{斜边}}\] 931 | \[\tan A=\frac{\angle A\text{的对边}}{\angle A\text{的邻边}},\qquad \cot A=\frac{\angle A\text{的邻边}}{\angle A\text{的对边}}\] 932 | \end{enumerate} 933 | 为了应用方便,我们把3中的四个公式写成下面的形 934 | 式: 935 | \[\angle A\text{的对边}=\text{斜边}\x \sin A,\qquad \angle A\text{的邻边}=\text{斜边}\x \cos A\] 936 | \[\angle A\text{的对边}=\text{邻边}\x \tan A,\qquad \angle A\text{的邻边}=\text{对边}\x \cot A\] 937 | 938 | 改用语言叙述,就是: 939 | \begin{blk}{} 940 | 在直角三角形中: 941 | \begin{enumerate} 942 | \item 一条直角边等于斜边乘上这条直角边所对锐角的 943 | 正弦. 944 | \item 一条直角边等于斜边乘上这条直角边相邻锐角的 945 | 余弦. 946 | \item 一条直角边等于另一条直角边乘上这条直角边所 947 | 对锐角的正切. 948 | \item 一条直角边等于另一条直角边乘上这条直角边相 949 | 邻锐角的余切. 950 | \end{enumerate} 951 | \end{blk} 952 | 953 | \begin{ex} 954 | \begin{enumerate} 955 | \item 在直角$\triangle ABC$中,$\angle C=90^{\circ}$, 求证: 956 | \[\overline{AB}=\frac{\overline{BC}}{\sin A},\qquad \overline{AB}=\frac{\overline{AC}}{\cos A}\] 957 | 并把这两个式子用语言叙述出来. 958 | \item 在直角$\triangle ABC$中,$\angle C=90^{\circ}$,求证: 959 | \begin{enumerate} 960 | \item $\sin A=\cos B,\qquad \sin B=\cos A$ 961 | \item $\tan A=\cot B,\qquad \cot A=\tan B$ 962 | \end{enumerate} 963 | \end{enumerate} 964 | 965 | \end{ex} 966 | 967 | \subsection{解直角三角形} 968 | 根据三角形的某些已知元素求出它的未知元素,这种过 969 | 程叫做\textbf{解三角形}.在直角三角形中,直角总是已知的.除直 970 | 角外,只要再知道两个元素,其中至少有一边,就可以求出 971 | 直角三角形的其它各元素.因此,解直角三角形,只有下面 972 | 四种情况: 973 | \begin{enumerate} 974 | \item 已知斜边与一锐角, 975 | \item 已知一条直角边与一锐角; 976 | \item 已知斜边与一条直角边; 977 | \item 已知两条直角边. 978 | \end{enumerate} 979 | 980 | 上述四种情况,都可用前面中所列出的关系式,并利用 981 | 三角比值表来求解. 982 | 983 | \begin{example} 984 | 已知$c=18$, $\angle A=62^{\circ}20'$ (图6.13), 985 | 求:$\angle B, a, b$ 986 | \end{example} 987 | 988 | \begin{solution} 989 | \[\begin{split} 990 | \angle B&=90^{\circ}-\angle A=90^{\circ}-62^{\circ}20' 991 | =27^{\circ}40'\\ 992 | a&=c\sin A=18\x \sin62^{\circ}20'=18\x0.8857 993 | \approx 15.9\\ 994 | b&=c\cos A=18\x\cos62^{\circ}20'=18\x0.4643\approx 8.3600 995 | \end{split}\] 996 | \end{solution} 997 | 998 | \begin{figure}[htp]\centering 999 | \begin{minipage}[t]{0.48\textwidth} 1000 | \centering 1001 | \begin{tikzpicture}[>=latex, scale=2] 1002 | \draw(0,0)node[left]{$A$}--node[below]{$b$}(.8358,0)node[right]{$C$}--node[right]{$a$}(62.33:1.8)node[above]{$B$}--node[left]{$c$}(0,0); 1003 | \draw(.8358,0)rectangle (.8358-.1,.1); 1004 | \draw(.2,0) arc (0:62.33:.2); 1005 | \end{tikzpicture} 1006 | \caption{} 1007 | \end{minipage} 1008 | \begin{minipage}[t]{0.48\textwidth} 1009 | \centering 1010 | \begin{tikzpicture}[>=latex, scale=1.3] 1011 | \draw(0,0)node[left]{$A$}--node[below]{$b$}(1.567,0)node[right]{$C$}--node[right]{$a$}(55.27:2.75)node[above]{$B$}--node[left]{$c$}(0,0); 1012 | \draw(1.567,0)rectangle (1.567-.2,.2); 1013 | \end{tikzpicture} 1014 | \caption{} 1015 | \end{minipage} 1016 | \end{figure} 1017 | 1018 | \begin{example} 1019 | 已知$c=27.5$, $a=22.6$, 求$b$、$\angle A$、$\angle B$ (图6.14). 1020 | \end{example} 1021 | 1022 | \begin{solution} 1023 | \[b=\sqrt{c^2-a^2}=\sqrt{27.5^2-22.6^2}\approx 15.7\] 1024 | 由于 1025 | $\sin A=\frac{a}{c}=\frac{22.6}{27.5}\approx 0.8218$, 1026 | 因此: 1027 | \[\begin{split} 1028 | \angle A&\approx 55^{\circ}16'\\ 1029 | \angle B&=90^{\circ}-\angle A=90^{\circ}-55^{\circ}16'=34^{\circ}44' 1030 | \end{split}\] 1031 | \end{solution} 1032 | 1033 | 1034 | \begin{example} 1035 | 已知$a=3.8$, $\angle A=42^{\circ}$, 求$\angle B$, $C$, $b$ (图6.15). 1036 | \end{example} 1037 | 1038 | \begin{solution} 1039 | \[\begin{split} 1040 | B&=90^{\circ} -\angle A=48^{\circ} \\ 1041 | b&=a \cot A=3.8\x\cot 42^{\circ}\approx 4.22\\ 1042 | c&=\frac{a}{\sin A}=\frac{3.8}{\sin 42^{\circ}}\approx 5.68 1043 | \end{split}\] 1044 | 1045 | \end{solution} 1046 | 1047 | \begin{figure}[htp]\centering 1048 | \begin{minipage}[t]{0.48\textwidth} 1049 | \centering 1050 | \begin{tikzpicture}[>=latex, scale=.7] 1051 | \draw(0,0)node[left]{$A$}--node[below]{$b$}(4.22,0)node[right]{$C$}--node[right]{$a$}(42:5.68)node[above]{$B$}--node[left]{$c$}(0,0); 1052 | \draw(4.22,0)rectangle (4.22-.2,.2); 1053 | \draw(.6,0) arc (0:42:.6); 1054 | \end{tikzpicture} 1055 | \caption{} 1056 | \end{minipage} 1057 | \begin{minipage}[t]{0.48\textwidth} 1058 | \centering 1059 | \begin{tikzpicture}[>=latex, scale=.8] 1060 | \draw(0,0)node[left]{$B$}--node[below]{$c$}(5,0)node[right]{$A$}--node[right]{$b$}(90-36.87:3)node[above]{$C$}--node[left]{$a$}(0,0); 1061 | 1062 | \end{tikzpicture} 1063 | \caption{} 1064 | \end{minipage} 1065 | \end{figure} 1066 | 1067 | 1068 | \begin{example} 1069 | 已知$a=3$、$b=4$, 求$c$、$\angle A$、$\angle B$ (图6.16) 1070 | \end{example} 1071 | 1072 | \begin{solution} 1073 | \[c=\sqrt{a^2+b^2}=\sqrt{3^2+4^2}=5\] 1074 | 由于$\tan A=\frac{3}{4}=0.75$, 1075 | 因此:$$\angle A\approx 36^{\circ} 52',\qquad 1076 | \angle B\approx 90^{\circ} -36^{\circ} 52'=53^{\circ} 8'$$ 1077 | \end{solution} 1078 | 1079 | \begin{ex} 1080 | \begin{enumerate} 1081 | \item 解下列直角三角形: 1082 | \begin{enumerate} 1083 | \item 已知$c=58.5,\quad \angle A=45^{\circ} 13'$ 1084 | \item 已知$c=14,\quad \angle B=62^{\circ}$ 1085 | \item 已知$c=28,\quad \angle A=34^{\circ} $ 1086 | \item 已知$c=195,\quad \angle B=78^{\circ} 47'$ 1087 | \item 已知$a=87,\quad \angle A=55^{\circ} $ 1088 | \item 已知$b=99,\quad \angle B=83^{\circ} $ 1089 | \item 已知$c=32,\quad a=18$ 1090 | \item 已知$a=14,\quad \angle B=78^{\circ}$ 1091 | \item 已知$c=79,\quad b=56$ 1092 | \item 已知$a=12.8,\quad b=15.6$ 1093 | \item 已知$c=73,\quad \angle B=66.2^{\circ}$ 1094 | \item 已知$c=350,\quad \angle A=3.8^{\circ} $ 1095 | \end{enumerate} 1096 | 1097 | \item 已知直角$\triangle ABC$, $\angle C=90^{\circ}$, $\angle A=\alpha$和$\angle A$对的直角边是 1098 | $a$ 1099 | 1100 | 求证:直角$\triangle ABC$的面积$S=\frac{a^2}{2}\tan\alpha$ 1101 | 1102 | \item 已知直角三角形的一个锐角为$\alpha$, 面积等于$S$, 1103 | 求它的外接圆的面积. 1104 | \end{enumerate} 1105 | \end{ex} 1106 | 1107 | \subsection{解直角三角形的应用} 1108 | 下面我们举例说明解直角三角形在实际中的应用. 1109 | \begin{example} 1110 | 如图6.17, 已知在测点$C$处,测得一铁塔顶端$A$的 1111 | 仰角$\angle ACE=\aleph$, $\overline{BD}=a$, 仪器的高度$\overline{CD}=b$, 求铁塔的高$\overline{AB}$. 1112 | 1113 | \end{example} 1114 | 1115 | 1116 | \begin{solution} 1117 | \[\overline{AB}=\overline{AE}+\overline{EB}\] 1118 | 已知$\overline{EB}=\overline{CD}=b$, 在$\triangle ACE$中, 1119 | \[\overline{AE}=\overline{CE}\x\tan\alpha\] 1120 | 又$\overline{CE}=\overline{BD}=a$. 1121 | 1122 | $\therefore\quad \overline{AB}=a\cdot \tan\alpha+b$ 1123 | \end{solution} 1124 | 1125 | \begin{figure}[htp]\centering 1126 | \begin{minipage}[t]{0.48\textwidth} 1127 | \centering 1128 | \includegraphics[scale=.5]{fig/6-17.png} 1129 | \caption{} 1130 | \end{minipage} 1131 | \begin{minipage}[t]{0.48\textwidth} 1132 | \centering 1133 | \begin{tikzpicture}[>=latex, scale=1] 1134 | \begin{scope} 1135 | \draw(30:2)node[right]{$B$}--(0,0)node[left]{$A$}--(2,0)node[right]{$C$}; 1136 | \node at (1,-.5){(1)}; 1137 | \draw(0.5,0) arc (0:30:.5); 1138 | \end{scope} 1139 | \begin{scope}[xshift=3cm] 1140 | \draw(2,1)node[right]{$C$}--(0,1)node[left]{$A$}--+(-30:2)node[right]{$B$}; 1141 | \node at (1,-.5){(2)}; 1142 | \draw(0.5,1) arc (0:-30:.5); 1143 | \end{scope} 1144 | \end{tikzpicture} 1145 | \caption{} 1146 | \end{minipage} 1147 | \end{figure} 1148 | 1149 | \begin{rmk} 1150 | 如图6.18(1)连结测点$A$和目的物$B$, 并且经过$A$点画和 1151 | $AB$在同一铅直平面内的水平线$AC$, 如$AB$在$AC$的上方,那么 1152 | $\angle BAC$叫做仰角;如图6.18(2), 如果$AB$在$AC$的下方,那么 1153 | $\angle CAB$叫做俯角. 1154 | \end{rmk} 1155 | 1156 | 1157 | \begin{example} 1158 | 如图6.19, 已知$C$、$D$两点与物体“$\overline{AB}$”的底端$B$ 1159 | 点共线,且 1160 | $\overline{CD}=a$, 仪器$\overline{CC'}$、$\overline{DD'}$的高都等于$b$, 在$C'$、 1161 | $D'$两点测得物体的顶端$A$的仰角分别是$\alpha$、$\beta$, 求物体的高 1162 | $\overline{AB}$ 1163 | \end{example} 1164 | 1165 | \begin{figure}[htp]\centering 1166 | \begin{minipage}[t]{0.48\textwidth} 1167 | \centering 1168 | \includegraphics[scale=.5]{fig/6-19.png} 1169 | \caption{} 1170 | \end{minipage} 1171 | \begin{minipage}[t]{0.48\textwidth} 1172 | \centering 1173 | \begin{tikzpicture}[>=latex, scale=1] 1174 | \draw(0,0)--node[below]{$\ell$}(4,0)--node[right]{$h$}(4,.8)--(0,0); 1175 | \draw(1,0) arc (0:11.3:1)node[right]{$\alpha$}; 1176 | \end{tikzpicture} 1177 | \caption{} 1178 | \end{minipage} 1179 | \end{figure} 1180 | 1181 | 1182 | \begin{solution} 1183 | 在直角$\triangle AEC'$与直角$\triangle AED'$中, 1184 | \begin{align} 1185 | \overline{C'E}&=\overline{AE}\x \cot\alpha\\ 1186 | \overline{D'E}&=\overline{AE}\x \cot\beta 1187 | \end{align} 1188 | $(6.1)-(6.2)$得: 1189 | \[\overline{D'E}-\overline{C'E}=\overline{AE} \x(\cot\beta-\cot\alpha)\] 1190 | 即:$\overline{C'D'}=\overline{AE} \x(\cot\beta-\cot\alpha)$ 1191 | 1192 | 已知$\overline{C'D'}=\overline{CD}=a$ 1193 | 1194 | $\therefore\quad \overline{AE}=\frac{a}{\cot\beta-\cot\alpha}$ 1195 | 1196 | 但$\overline{AB}=\overline{AE}+\overline{EB}=\overline{AE} +b$, 1197 | 1198 | $\therefore\quad \overline{AB}=b+\frac{a}{\cot\beta-\cot\alpha}$ 1199 | 1200 | \end{solution} 1201 | 1202 | 如果要测底部可以到达的物体的高度,可用例6.22所介绍 1203 | 的方法,如要测底部不能到达的的物体的高度,可用例6.23所 1204 | 介绍的方法. 1205 | 1206 | \begin{example} 1207 | 一条公路的路面升高$h$与水平距离$\ell$的比值$i$叫做 1208 | 路面的坡度.即$i=h:\ell$ (图6.20). 已知某段公路,每前进 1209 | 100米就升高4米,求路面的坡度及路面对水平面的倾角$\alpha$. 1210 | \end{example} 1211 | 1212 | \begin{solution} 1213 | 路面的坡度$i=\frac{h}{\ell}$ 1214 | ,已知$h=4$米,$\ell=100$米, 1215 | 1216 | $\therefore\quad i=\frac{4}{100}=0.04$ 1217 | 1218 | 从图6.20中的直角三角形可 1219 | 见,$h:\ell$正好是角$\alpha$的正切,即:$i=\tan\alpha$, 1220 | 1221 | $\therefore\quad \tan\alpha=0.04$. 倒查正切表得: 1222 | $\alpha=2^{\circ}17'$. 1223 | 1224 | 答:路面的坡度是0.04, 路面对水平面的倾角是$2^{\circ}17'$. 1225 | \end{solution} 1226 | 1227 | \begin{example} 1228 | 已知一门式起重 1229 | 机(图6.21),机身高21 1230 | 米,吊杆长$\overline{AB}=36$米,吊 1231 | 杆的倾角$A$(即吊杆与水平 1232 | 线的夹角)可以从$30^{\circ}$转到 1233 | $80^{\circ}$, 求这门起重机工作时 1234 | 的最大高度和最大水平距 1235 | 离. 1236 | \end{example} 1237 | 1238 | \begin{figure}[htp] 1239 | \centering 1240 | \includegraphics[scale=.6]{fig/6-21.png} 1241 | \caption{} 1242 | \end{figure} 1243 | 1244 | \begin{solution} 1245 | 当吊杆$\overline{AB}$的倾角 1246 | $A$达到最大限度$80^{\circ}$时,这时 1247 | 起重机吊的最高位置如图6. 1248 | 21中$\overline{AB}$的位置.在直角 1249 | $\triangle ABC$中,$\angle A=80^{\circ}$, 1250 | $\overline{AB}=36$米,因此: 1251 | \[ \overline{EC}=\overline{AB} \x \sin80^{\circ}=36\x\sin80^{\circ} 1252 | =36\x0.9848\approx 35.45{\rm m}\] 1253 | \[\text{起重机的最大高度}=\text{机身高}+\overline{BC}=21+35.45=56.45{\rm m}\] 1254 | 1255 | 当倾角$A$达到最小角时,起吊的水平距离最远,这时, 1256 | 如图6.21中$\overline{AB'}$的位置.在直角$\triangle AB'C'$中,$\angle A=30^{\circ}$, 1257 | $\overline{AB'}=36$米. 1258 | 1259 | 因此:起重机起吊的最远水平距离 1260 | \[\overline{AC'}=\overline{AB'}\x\cos30^{\circ}=36\x0.8660 1261 | \approx 31.18{\rm m}\] 1262 | 1263 | 答:起重机工作的最大高度是56.45米,最远水平距离 1264 | 是31.18米. 1265 | \end{solution} 1266 | 1267 | \begin{ex} 1268 | \begin{enumerate} 1269 | \item 测量学校旗杆的高度. 1270 | \item 选择底部不能到达的建筑物或树本,测量它的高度. 1271 | \item 如图,要求河两岸$B$、$C$两点间的距离,在$B$点这一岸垂 1272 | 直于$BC$方向上找一点$A$, 测出$\angle BAC=58^{\circ}12'$, 1273 | $\overline{AB}=25$米,求$B$、$C$两点间的距离. 1274 | \item 如图,从山顶$D$测得地平面上同一方向的两点$A$和$B$的俯 1275 | 角分别是$18^{\circ}$和$23^{\circ}$, 已知$\overline{AB}=140$米,求山高 1276 | $\overline{CD}$(得数保留整数米). 1277 | \item 已知传送带和地面的夹角是$25^{\circ}$, 它把物件从地面运到 1278 | 离地面9米高的地方,求物件所走的路程. 1279 | \item 工件上有V形槽,测出上口宽200mm,深19.2mm, 1280 | 求V形角$\alpha$多大. 1281 | \item 如图,在离地面高5米处引拉线固定电线杆,拉线和地 1282 | 面成$60^{\circ}$角,求每根拉线多长,拉线底端离杆底多远. 1283 | \end{enumerate} 1284 | \end{ex} 1285 | 1286 | \begin{figure}[htp]\centering 1287 | \begin{minipage}[t]{0.3\textwidth} 1288 | \centering 1289 | \includegraphics[scale=.7]{fig/6-3ti.PNG} 1290 | \caption*{第3题} 1291 | \end{minipage} 1292 | \begin{minipage}[t]{0.6\textwidth} 1293 | \centering 1294 | \includegraphics[scale=.7]{fig/6-4ti.PNG} 1295 | \caption*{第4题} 1296 | \end{minipage} 1297 | \end{figure} 1298 | 1299 | \begin{figure}[htp]\centering 1300 | \begin{minipage}[t]{0.48\textwidth} 1301 | \centering 1302 | \begin{tikzpicture}[>=latex, scale=1] 1303 | \tkzDefPoints{0/0/A, 4/0/B, 4/2/C, 2.7/2/D, 2/1/E, 1.3/2/F, 0/2/G} 1304 | \tkzDrawPolygon(A,B,C,D,E,F,G) 1305 | \draw[dashed](F)--node[above]{20}(D); 1306 | \draw[<->](3.5,1)--node[fill=white]{19.2}(3.5,2); 1307 | \draw(E)--(3.7,1); 1308 | \tkzMarkAngle[mark=none, size=.3](D,E,F) 1309 | \node at (E)[above=.2]{$\alpha$}; 1310 | \end{tikzpicture} 1311 | \caption*{第6题} 1312 | \end{minipage} 1313 | \begin{minipage}[t]{0.48\textwidth} 1314 | \centering 1315 | \begin{tikzpicture}[>=latex, scale=1] 1316 | \fill[pattern=north east lines](-2,-.25) rectangle (2.5,0); 1317 | \draw(-2,0)--(2.5,0); 1318 | \draw[very thick](-1.75,0)--(0,3.03)--(1.75,0); 1319 | \draw(-.04,0) rectangle (.04,3.5); 1320 | \draw(0,3.03)--(2.5,3.03); 1321 | \draw[<->](2.25,0)--node[fill=white]{5m}(2.25,3.03); 1322 | \draw(-1.25,0) arc (0:60:.5)node[right]{$60^{\circ}$}; 1323 | \end{tikzpicture} 1324 | \caption*{第7题} 1325 | \end{minipage} 1326 | \end{figure} 1327 | 1328 | \section*{习题6.2} 1329 | \addcontentsline{toc}{subsection}{习题6.2} 1330 | \begin{enumerate} 1331 | \item 已知等腰$\triangle ABC$, 腰长为5cm, 底边长为8cm, 求 1332 | $\angle A$、$\angle B$、$\angle C$. 1333 | \item 把2米的竹杆,垂直于地面的时候,影长1.6米,这时 1334 | 太阳的仰角大约是多少度? 1335 | \item 已知等腰$\triangle ABC$, 腰长$\overline{AB}=\overline{AC}=10$cm, 底角为$40^{\circ}$, 1336 | 求高$\overline{AD}$, 底边$\overline{BC}$和面积$S$. 1337 | \item 在$\triangle ABC$中,$\angle B$、 1338 | $\angle C$是锐角,$\overline{AB}=c$, $\overline{AC}=b$, $\overline{AD}$是 1339 | $\overline{BC}$边上的高,求证: 1340 | \begin{enumerate} 1341 | \item $\overline{AD}=b\sin C=c\sin B$ 1342 | \item $\frac{b}{\sin B}=\frac{c}{\sin C}$ 1343 | \end{enumerate} 1344 | \item 已知正$n$边形的半径是$r$,周长是$\ell$,求证: 1345 | \[\ell=2nr\sin\frac{180^{\circ}}{n}\] 1346 | \item 一架战斗机从3300米高空以每秒150米的速度向轰炸目 1347 | 标俯冲,俯冲角为$42^{\circ}$, 在离地面1300米时投弹,击中 1348 | 目标,问飞机开始俯冲到投弹共用几秒钟? 1349 | \begin{figure}[htp]\centering 1350 | \begin{minipage}[t]{0.48\textwidth} 1351 | \centering 1352 | \begin{tikzpicture}[>=latex, scale=1] 1353 | \draw(-2,0)--(4,0); 1354 | \draw(-1.5,0) rectangle (-.8,.5); 1355 | \draw(-1.3,.5) rectangle (-1,.9); 1356 | \draw[dashed](0,3.3)--(4,3.3); 1357 | \draw[dashed](0,1.3)--(3,1.3); 1358 | \draw[dashed](3,0)--(3,3.3); 1359 | \draw[very thick, ->](3,3.3)--(0,1.3); 1360 | \draw[<->] (3.5,3.3)--node[fill=white]{3300}(3.5,0); 1361 | \draw[<->] (1,1.3)--node[fill=white]{1300}(1,0); 1362 | \end{tikzpicture} 1363 | \caption*{第6题} 1364 | \end{minipage} 1365 | \begin{minipage}[t]{0.48\textwidth} 1366 | \centering 1367 | \begin{tikzpicture}[>=latex, scale=.8] 1368 | \draw(0,0)node[left]{$A$}--node[above]{2km}(4,0)node[right]{$B$}; 1369 | \draw[dashed](0,-2)--(0,0)--(4,-4.767)node[below]{$C$}--(4,0); 1370 | \draw(0,-.5) arc (-90:-50:.5)node[below=4pt]{$40^{\circ}$}; 1371 | \end{tikzpicture} 1372 | \caption*{第7题} 1373 | \end{minipage} 1374 | \end{figure} 1375 | 1376 | \item 东西两炮台$A$、$B$相距2km, 同时发现入侵敌舰$C$, 炮台 1377 | $A$测得敌舰$C$在它的南偏东$40^{\circ}$的方向,炮台$B$测得敌舰$C$ 1378 | 在它的正南方,试求敌舰与两炮台的距离. 1379 | 1380 | \item 1381 | 从圆外一点向圆引两条切线,已知切线长等于21.8cm, 1382 | 圆的半径等于10.6cm, 求这两条切线间的夹角. 1383 | \end{enumerate} 1384 | 1385 | \section{任意三角形中的边角关系} 1386 | \subsection{正弦定理} 1387 | 1388 | 为了解任意的三角形,下面我们用三角比来研究任意三 1389 | 角形中的边角关系. 1390 | 1391 | 我们约定在$\triangle ABC$中,$\angle A$、$\angle B$、$\angle C$分别用$\alpha$、$\beta$、$\gamma$ 1392 | 来表示;$\angle A$、$\angle B$、$\angle C$的对边分别用$a$、$b$、$c$来表示;外 1393 | 接圆的半径用$r$来表示. 1394 | 1395 | 假定$\alpha$是锐角(图6.22),作$\triangle ABC$的外接圆$O$, 再作$\odot O$的直径$\overline{CA'}$, 弦$\overline{BA'}$, 在$\triangle A'BC$中,$\angle A'BC$是直角,因此$a=2r\sin A'$ 1396 | 1397 | $\because\quad \angle A'=\angle A=\alpha$ 1398 | 1399 | $\therefore\quad a=2r\sin\alpha$,因此: 1400 | \begin{equation} 1401 | \frac{a}{\sin\alpha}=r 1402 | \end{equation} 1403 | 1404 | \begin{figure}[htp]\centering 1405 | \begin{minipage}[t]{0.48\textwidth} 1406 | \centering 1407 | \begin{tikzpicture}[>=latex, scale=1] 1408 | \draw[thick](0,0) circle(2); 1409 | \tkzDefPoints{0/0/O} 1410 | \tkzDefPoint(40:2){B}\tkzDefPoint(-30:2){C}\tkzDefPoint(-170:2){A}\tkzDefPoint(150:2){A'} 1411 | \tkzDrawPolygon(A,B,C) 1412 | \tkzDrawPolygon[dashed](A',B,C) 1413 | \tkzAutoLabelPoints[center=O](A,B,C,A') 1414 | \tkzLabelPoints[right](O) 1415 | \tkzDrawPoints(O) 1416 | \tkzMarkAngles[mark=none, size=.5](C,A,B C,A',B) 1417 | \node at (5:2)[left]{$a$}; 1418 | 1419 | \end{tikzpicture} 1420 | \caption{} 1421 | \end{minipage} 1422 | \begin{minipage}[t]{0.48\textwidth} 1423 | \centering 1424 | \begin{tikzpicture}[>=latex, scale=1] 1425 | \draw[thick](0,0) circle(2); 1426 | \tkzDefPoint(120:2){B}\tkzDefPoint(-120:2){C}\tkzDefPoint(180:2){A}\tkzDefPoint(60:2){A'} 1427 | \tkzLabelPoints[right](O) 1428 | \tkzDrawPoints(O) 1429 | \tkzDefPoints{0/0/O} 1430 | \tkzDrawPolygon(A,B,C) 1431 | \tkzDrawPolygon[dashed](A',B,C) 1432 | \tkzAutoLabelPoints[center=O](A,B,C,A') 1433 | \tkzMarkAngles[mark=none, size=.4](C,A,B B,A',C) 1434 | 1435 | \end{tikzpicture} 1436 | \caption{} 1437 | \end{minipage} 1438 | \end{figure} 1439 | 1440 | 假定$\angle A$是钝角(图6.23),作$\triangle ABC$的外接圆$\odot O$, 再 1441 | 作直径$\overline{CA'}$, 弦$\overline{BA'}$ 1442 | 1443 | $\because\quad \angle A+\angle A'=180^{\circ}$ 1444 | 1445 | $\therefore\quad \angle A'=180^{\circ}-\angle A$, $\angle A'$是一个锐角, 1446 | 1447 | 在直角$\triangle A'BC$中,$\angle A'BC$是直角. 1448 | 1449 | $\therefore\quad a=2r\sin A'=2r\sin(180^{\circ}-A)=2r\sin(180^{\circ}-\alpha)$,因此: 1450 | \begin{equation} 1451 | \frac{a}{\sin(180^{\circ}-\alpha)}=2r 1452 | \end{equation} 1453 | 1454 | 我们比较(6.3)、(6.4)两式,可以 1455 | 看出,如果我们定义一个钝角的正弦 1456 | 等于它的补角的正弦,即 1457 | \[\sin\alpha =\sin(180^{\circ}-\alpha )\] 1458 | 其中:$\alpha$ 是钝角. 1459 | 那么(6.4)式在形式上就与(6.3)式相同 1460 | 了,这就是说,$\alpha$ 不论是锐角还是钝角 1461 | 都有: 1462 | \[\frac{a}{\sin\alpha}=2r\] 1463 | 1464 | \begin{figure}[htp] 1465 | \centering 1466 | \begin{tikzpicture} 1467 | \draw[thick](0,0) circle(2); 1468 | \tkzDefPoint(100:2){B}\tkzDefPoint(-80:2){C}\tkzDefPoint(-130:2){A} 1469 | \tkzDefPoints{0/0/O} 1470 | \tkzLabelPoints[right](O) \tkzDrawPoints(O) 1471 | \tkzDrawPolygon(A,B,C) 1472 | \tkzAutoLabelPoints[center=O](A,B,C) 1473 | \tkzMarkRightAngle[size=.2](C,A,B) 1474 | \end{tikzpicture} 1475 | \caption{} 1476 | \end{figure} 1477 | 1478 | 当$\alpha$是直角时(图6.24),作直角$\triangle ABC$的外接圆$\odot O$ 1479 | 这时斜边$\alpha$正好是$O$的直径. 1480 | 1481 | $\because\quad \sin90^{\circ}=1$ 1482 | 1483 | $\therefore\quad \frac{a}{\sin\alpha}=2r$ 1484 | 仍然成立. 1485 | 1486 | 同理,我们还可得出, 1487 | \[\frac{b}{\sin\beta}=2r,\qquad \frac{c}{\sin\gamma}=2r\] 1488 | 1489 | 总结上面的讨论,我们得到: 1490 | 1491 | \begin{blk} 1492 | {正弦定理} 1493 | 在任意$\triangle ABC$中,边角关系满足: 1494 | \[\frac{a}{\sin\alpha}=\frac{b}{\sin\beta}=\frac{c}{\sin\gamma}=2r\] 1495 | \end{blk} 1496 | 1497 | \begin{example} 1498 | 已知等边$\triangle ABC$的边长是$a$, 求$r$. 1499 | \end{example} 1500 | 1501 | \begin{solution} 1502 | $\because\quad \frac{a}{\sin\alpha}=2r,\quad \alpha=60^{\circ}$ 1503 | 1504 | $\therefore\quad \frac{a}{\sin60^{\circ}}=2r$ 1505 | \[r=\frac{a}{2\sin60^{\circ}}=\frac{a}{2\cdot \frac{\sqrt{3}}{2}}=\frac{\sqrt{3}}{3}a\] 1506 | \end{solution} 1507 | 1508 | \begin{example} 1509 | 在$\triangle ABC$中,已知$b\sin\beta=c\sin\gamma$, 求证$\triangle ABC$是等腰三角形. 1510 | \end{example} 1511 | 1512 | \begin{proof} 1513 | $\because\quad b\sin\beta=c\sin\gamma$ 1514 | 1515 | $\therefore\quad \frac{b}{c}=\frac{\sin\gamma}{\sin\beta}$ 1516 | 1517 | 又$\because\quad \frac{b}{\sin\beta}=\frac{c}{\sin\gamma}$ 1518 | 1519 | $\therefore\quad \frac{c}{b}=\frac{\sin\gamma}{\sin\beta},\quad \frac{b}{c}=\frac{c}{b},\quad b^2=c^2$ 1520 | 1521 | $\because\quad b>0,\quad c>0$ 1522 | 1523 | $\therefore\quad b=c$,$\triangle ABC$是等腰三角形. 1524 | \end{proof} 1525 | 1526 | \begin{example} 1527 | 已知$P$点是$\triangle ABC$的$\overline{BC}$边上的任一点,求证: 1528 | \[\frac{\overline{BP}}{\overline{PC}}=\frac{c\sin \angle PAB}{b\sin\angle PAC}\] 1529 | \end{example} 1530 | 1531 | \begin{proof} 1532 | 如图6.25所示,在$\triangle ABP$与$\triangle APC$中,根据正弦 1533 | 定理有: 1534 | \[\frac{\overline{BP}}{\sin \angle PAB}=\frac{c}{\sin \angle APB},\qquad \frac{\overline{PC}}{\sin \angle PAC}=\frac{b}{\sin\angle APC}\] 1535 | 即: 1536 | \[\overline{BP}=\frac{c\sin \angle PAB}{\sin \angle APB},\qquad \overline{PC}=\frac{b\sin \angle PAC}{\sin \angle APC}\] 1537 | 1538 | $\because\quad \angle APB+\angle APC=\pi$ 1539 | 1540 | $\therefore\quad \sin \angle APB=\sin\angle APC,\qquad \frac{\overline{BP}}{\overline{PC}}=\frac{c\sin\angle PAB}{b\sin\angle PAC}$ 1541 | \end{proof} 1542 | 1543 | \begin{figure}[htp] 1544 | \centering 1545 | \begin{tikzpicture}[scale=.8] 1546 | \tkzDefPoints{0/0/B, 5/0/C, 2/3/A, 1.5/0/P} 1547 | \tkzDrawPolygon(A,B,C) 1548 | \tkzDrawSegments(A,P) 1549 | \tkzLabelPoints[above](A) 1550 | \tkzLabelPoints[below](B,C,P) 1551 | \node at (1,1.5)[left]{$c$}; 1552 | \node at (3.5,1.5)[right]{$b$}; 1553 | \end{tikzpicture} 1554 | \caption{} 1555 | \end{figure} 1556 | 1557 | \begin{ex} 1558 | \begin{enumerate} 1559 | \item 求证在$\triangle ABC$中, 1560 | \begin{enumerate} 1561 | \item $\sin\alpha=\frac{a}{b}\sin\beta=\frac{a}{c}\sin\gamma$ 1562 | \item $a+b+c=2r(\sin\alpha +\sin\beta+\sin\gamma)$ 1563 | \item $\sin\alpha+\sin\beta<\sin\gamma$ 1564 | \end{enumerate} 1565 | 1566 | \item 在$\odot O$中,量得一个$30^{\circ}$的圆周角所对的弦长是4cm, 1567 | 求$\odot O$的半径$r$. 1568 | \item 求下列各角的正弦: 1569 | \[90^{\circ},\quad 120^{\circ},\quad 150^{\circ},\quad 135^{\circ},\quad 132^{\circ}\] 1570 | \item 在半径是5cm的圆中,$120^{\circ}$的圆心角所对的弦长是多 1571 | 少? 1572 | \item 在半径是15cm的圆中,一条长18cm的弦所对的圆心角 1573 | 是多少度(精确到度). 1574 | \item 应用正弦定理证明三角形内角平分线定理(提示:模仿 1575 | 例6.28的证法). 1576 | \end{enumerate} 1577 | \end{ex} 1578 | 1579 | \subsection{余弦定理} 1580 | 在$\triangle ABC$中,如果$\beta$、$\gamma$都是锐角(图6.26),作$\overline{BC}$边 1581 | 上的高$\overline{AD}$, 则$a=\overline{BD}+\overline{DC}$. 1582 | 1583 | 但$\overline{BD}=c\cos\beta$, $\overline{DC}=b\cos\gamma$,因此: 1584 | \begin{equation} 1585 | a=c\cos\beta+b\cos\gamma 1586 | \end{equation} 1587 | 1588 | 如果$\beta$、$\gamma$中有一个是钝角,设$\gamma$是钝角(图6.27)作 1589 | $\overline{BC}$边上的高$\overline{AD}$, 这时垂足$D$落在$\overline{BC}$的延长线上,则$a=\overline{BD}-\overline{CD}$.但$\overline{BD}=c\cos\beta$, $\overline{CD}=b\cos(180^{\circ}-\gamma)$,因此: 1590 | \begin{equation} 1591 | a=c\cos\beta -b\cos(180^{\circ}-\gamma) 1592 | \end{equation} 1593 | 1594 | 比较(6.5)、(6.6)两式,为了使它们在形式上得到一致, 1595 | 我们定义钝角Y的余弦为: 1596 | \[\cos\gamma=-\cos(180^{\circ}-\gamma)\qquad \text{($\gamma$为钝角)}\] 1597 | 这样,不论$\gamma$是锐角还是钝角都有关系式 1598 | \[a=c\cos\beta +b\cos\gamma\] 1599 | 如果$\beta$ 是钝角,由$\cos\beta =-\cos(180^{\circ}-\beta )$同样可以得到: 1600 | \[a=c\cos\beta +b\cos\gamma\] 1601 | 1602 | \begin{figure}[htp]\centering 1603 | \begin{minipage}[t]{0.32\textwidth} 1604 | \centering 1605 | \begin{tikzpicture}[>=latex, scale=1] 1606 | \tkzDefPoints{0/0/B, 3/0/C, 2/2/A, 2/0/D} 1607 | \tkzLabelPoints[below](B,C,D) 1608 | \tkzLabelPoints[above](A) 1609 | \tkzDrawPolygon(A,B,C) 1610 | \node at (1,1)[left]{$c$}; \node at (2.5,1)[right]{$b$}; 1611 | \node at (1,0)[below]{$a$}; \draw(A)--(D); 1612 | \tkzMarkRightAngle[size=.2](A,D,B) 1613 | \tkzMarkAngles[size=.3, mark=none](A,C,D C,B,A) 1614 | \tkzLabelAngle[pos=.5pt](A,C,D){$\gamma$} 1615 | \tkzLabelAngle[pos=.5pt](C,B,A){$\beta$} 1616 | 1617 | \end{tikzpicture} 1618 | \caption{} 1619 | \end{minipage} 1620 | \begin{minipage}[t]{0.32\textwidth} 1621 | \centering 1622 | \begin{tikzpicture}[>=latex, scale=1] 1623 | \tkzDefPoints{0/0/B, 2/0/C, 3/2/A, 3/0/D} 1624 | \tkzLabelPoints[below](B,C,D) 1625 | \tkzLabelPoints[above](A) 1626 | \tkzDrawPolygon(A,B,C) 1627 | \draw[dashed](A)--(D)--(C); 1628 | \tkzMarkRightAngle[size=.2](A,D,B) 1629 | \node at (1.5,1)[left]{$c$}; \node at (2.5,1)[right]{$b$}; 1630 | \node at (1,0)[below]{$a$}; 1631 | \tkzMarkAngles[size=.3, mark=none](A,C,B) 1632 | \tkzLabelAngle[pos=.5pt](A,C,B){$\gamma$} 1633 | \end{tikzpicture} 1634 | \caption{} 1635 | \end{minipage} 1636 | \begin{minipage}[t]{0.32\textwidth} 1637 | \centering 1638 | \begin{tikzpicture}[>=latex, scale=1] 1639 | \tkzDefPoints{0/0/B, 2/0/C, 2/2.5/A} 1640 | \tkzDrawPolygon(A,B,C) 1641 | \tkzLabelPoints[below](B,C) 1642 | \tkzLabelPoints[above](A) 1643 | \tkzMarkRightAngle[size=.2](A,C,B) 1644 | 1645 | \end{tikzpicture} 1646 | \caption{} 1647 | \end{minipage} 1648 | \end{figure} 1649 | 1650 | 1651 | 如果$\beta$、$\gamma$中有一个是直角,设$\gamma=90^{\circ}$ (图6.28), 1652 | 1653 | $\because\quad \cos\gamma=\cos90^{\circ}=0$ 1654 | 1655 | $\therefore\quad a=c\cos\beta +b\cos\gamma$ 仍然成立. 1656 | 1657 | 若分别作$\overline{AB}$、$\overline{AC}$边上的高,同样还可证明 1658 | \[\begin{split} 1659 | b&=a\cos\gamma+c\cos\alpha \\ 1660 | c&=b\cos\alpha+a\cos\beta 1661 | \end{split}\] 1662 | 1663 | 总结上面的讨论,我们得到: 1664 | 1665 | \begin{blk} 1666 | {定理} 1667 | 在任意$\triangle ABC$中,边角关系满足: 1668 | \begin{align} 1669 | a&=c\cos\beta +b\cos\gamma\\ 1670 | b&=a\cos\gamma+c\cos\alpha \\ 1671 | c&=b\cos\alpha +a\cos\beta 1672 | \end{align} 1673 | \end{blk} 1674 | 1675 | 由上面的定理,我们就可推出余弦定理 1676 | 1677 | \begin{blk} 1678 | {余弦定理} 1679 | 在任意的$\triangle ABC$中,边角关系满足 1680 | \[\begin{split} 1681 | a^2&=b^2+c^2-2bc\cos\alpha\\ 1682 | b^2&=c^2+a^2-2ca \cos\beta\\ 1683 | c^2&=a^2+b^2-2ab\cos\gamma 1684 | \end{split}\] 1685 | \end{blk} 1686 | 1687 | \begin{proof} 1688 | 由上面的定理的(6.7)式得, 1689 | \[\cos\beta=\frac{a-b\cos\gamma}{c}\] 1690 | 由(6.8)式得 1691 | \[\cos\alpha=\frac{b-a\cos\gamma}{c}\] 1692 | 把$\cos\beta$、$\cos\alpha$代入(6.9)得 1693 | \[c=a\frac{a-b\cos\gamma}{c}+b\frac{b-a\cos\gamma}{c}=\frac{a^2+b^2-2ab\cos\gamma}{c}\] 1694 | $\therefore\quad c^2=a^2+b^2-2ab\cos\gamma$ 1695 | 1696 | 用同样的办法我们可证: 1697 | \[\begin{split} 1698 | b^2&=c^2+a^2-2ca \cos\beta\\ 1699 | a^2&=b^2+c^2-2bc\cos\alpha 1700 | \end{split}\] 1701 | \end{proof} 1702 | 1703 | 请同学们自证如下两个推论: 1704 | 1705 | \begin{blk}{推论1} 1706 | 在$\triangle ABC$中, 1707 | \[\begin{split} 1708 | \cos\alpha&=\frac{b^2+c^2-a^2}{2bc}\\ 1709 | \cos\beta&=\frac{a^2+c^2-b^2}{2ac}\\ 1710 | \cos\gamma&=\frac{a^2+b^2-c^2}{2ab} 1711 | \end{split}\] 1712 | \end{blk} 1713 | 1714 | \begin{blk}{推论2} 1715 | 一个三角形两边平方的和如果等于第三边的平 1716 | 方,那么第三边所对的角是直角;如果小于第三边的平方, 1717 | 那么第三边所对的角是钝角;如果大于第三边的平方,那么 1718 | 第三边所对的角是锐角. 1719 | \end{blk} 1720 | 1721 | 1722 | \begin{example} 1723 | 分别求出$120^{\circ}$、$135^{\circ}$、$150^{\circ}$、$140^{\circ}$的余弦. 1724 | \end{example} 1725 | 1726 | \begin{solution} 1727 | 由定义知,一个钝角的余弦等于它的补角的余弦的 1728 | 相反数,所以有: 1729 | \[\begin{split} 1730 | \cos120^{\circ}&=-\cos(180^{\circ}-120^{\circ})=-\cos60^{\circ}=-\frac{1}{2}\\ 1731 | \cos135^{\circ}&=-\cos(180^{\circ}-135^{\circ})=-\cos45^{\circ}=-\frac{\sqrt{2}}{2}\\ 1732 | \cos150^{\circ}&=-\cos(180^{\circ}-150^{\circ})=-\cos30^{\circ}=- 1733 | \frac{\sqrt{3}}{2}\\ 1734 | \cos140^{\circ}&=-\cos(180^{\circ}-140^{\circ})=-\cos40^{\circ}\approx -0.7660 1735 | \end{split}\] 1736 | \end{solution} 1737 | 1738 | \begin{example} 1739 | 用余弦定理证明广义勾股定理. 1740 | 1741 | 已知:$\parallelogram ABCD$ (图6.29) 1742 | 1743 | 求证: 1744 | $\overline{AC}^2+\overline{BD}^2=2(\overline{AB}^2+\overline{AD}^2)$ 1745 | \end{example} 1746 | 1747 | \begin{figure}[htp] 1748 | \centering 1749 | \begin{tikzpicture} 1750 | \tkzDefPoints{0/0/A, 3/0/B, 3.75/1.5/C, .75/1.5/D} 1751 | \tkzDrawPolygon[thick](A,B,C,D) 1752 | \tkzDrawSegments[thick](A,C B,D) 1753 | \tkzLabelPoints[below](A,B) 1754 | \tkzLabelPoints[above](C,D) 1755 | \end{tikzpicture} 1756 | \caption{} 1757 | \end{figure} 1758 | 1759 | \begin{proof} 1760 | 在$\triangle ABC$与$\triangle ABD$中, 1761 | \begin{align} 1762 | \overline{AC}^2&=\overline{AB}^2+\overline{BC}^2-2\overline{AB}\cdot \overline{BC} \cos\angle ABC \\ 1763 | \overline{BD}^2&=\overline{AB}^2+\overline{AD}^2-2\overline{AB}\cdot \overline{AD}\cos\angle BAD 1764 | \end{align} 1765 | 1766 | $\because\quad \angle BAD+\angle ABC =180^{\circ}$ 1767 | 1768 | $\therefore\quad \cos \angle ABC=-\cos\angle BAD$ 1769 | 1770 | 又$\because\quad \overline{BC}=\overline{AD}$ 1771 | 1772 | $(6.10)+(6.11)$则得: 1773 | \[\overline{AC}^2+\overline{BD}^2=\overline{AB}^2+\overline{BC}^2+\overline{AB}^2+\overline{AD}^2=2\left(\overline{AB}^2+\overline{AD}^2\right)\] 1774 | \end{proof} 1775 | 1776 | \begin{ex} 1777 | \begin{enumerate} 1778 | \item 在$\triangle ABC$中,已知$a=4$, $b=7$, $\gamma=40^{\circ}$, 求$c$. 1779 | \item 在$\triangle ABC$中,已知$a=2$, $b=3$, $c=4$, 求$\cos\alpha$、$\cos\beta$、 1780 | $\cos\gamma$. 1781 | \item 已知$\triangle ABC$的三边: 1782 | \begin{enumerate} 1783 | \item 56, 65, 33, 求最大角; 1784 | \item 7、$4\sqrt{3}$、$\sqrt{13}$, 求最小角. 1785 | \end{enumerate} 1786 | 1787 | \item 在$\triangle ABC$中,$\angle A=120^{\circ}$, 求证: 1788 | \begin{enumerate} 1789 | \item $a^2-b^2=c(b+c)$ 1790 | \item $ b(a^2-b^2)=c(a^2-c^2)$ 1791 | \end{enumerate} 1792 | 1793 | \item 在$\triangle ABC$中,已知$\cos\beta=\frac{\sin\alpha}{2\sin\gamma}$, 求证$\triangle ABC$是等腰三角形. 1794 | \item 在$\triangle ABC$中,求证: 1795 | \[\frac{\cos A}{a}+\frac{\cos B}{b}+\frac{\cos C}{c}=\frac{a^2+b^2+c^2}{2abc}\] 1796 | \end{enumerate} 1797 | \end{ex} 1798 | 1799 | \subsection{解斜三角形} 1800 | 在一个三角形中,如果没有一个角是直角,那么,这个三角 1801 | 形叫做斜三角形.斜三角形的解法可以分成下面的四种情形: 1802 | \begin{enumerate} 1803 | \item 已知一边和两角; 1804 | \item 已知两边和它们的夹角; 1805 | \item 已知三边; 1806 | \item 已知两边和其中一边的对角. 1807 | \end{enumerate} 1808 | 1809 | 下面我们分别举例说明每种情形的解法. 1810 | 1811 | \begin{example} 1812 | 已知:$c=100$、$\alpha=40^{\circ}$, $\beta=60^{\circ}$ (图6.30), 求 1813 | $\gamma$、$a$、$b$. 1814 | \end{example} 1815 | 1816 | \begin{figure}[htp] 1817 | \centering 1818 | \begin{tikzpicture} 1819 | \tkzDefPoints{0/0/A, 4/0/B, 2.8/2/C} 1820 | \tkzDrawPolygon(A,B,C) 1821 | \tkzMarkAngles[mark=none, size=.3](B,A,C C,B,A) 1822 | \tkzLabelPoints[below](A,B) 1823 | \tkzLabelPoints[above](C) 1824 | \end{tikzpicture} 1825 | \caption{} 1826 | \end{figure} 1827 | 1828 | \begin{solution} 1829 | \[\begin{split} 1830 | \gamma&=180^{\circ}-(40^{\circ}+60^{\circ})=80^{\circ}\\ 1831 | a&=\frac{c\sin\alpha}{\sin\gamma}\approx \frac{100\x \sin 40^{\circ}}{\sin 80^{\circ}}=\frac{100\x 0.6428}{0.9848}=65.27\\ 1832 | b&=\frac{c\sin\beta}{\sin\gamma}=\frac{100\x \sin 60^{\circ}}{\sin 80^{\circ}}=\frac{100\x 0.8660}{0.9848}=87.94 1833 | \end{split}\] 1834 | \end{solution} 1835 | 1836 | 例6.31告诉我们,如果已知两角和一边,先用内角和定理 1837 | 求出未知的第三个角,然后再用正弦定理便可求出未知的两 1838 | 边了. 1839 | 1840 | \begin{ex} 1841 | \begin{enumerate} 1842 | \item 已知下列条件,解三角形,并求其外接圆的半径. 1843 | \begin{enumerate} 1844 | \item $b=4,\qquad \alpha=30^{\circ} ,\qquad \beta=120^{\circ} $ 1845 | \item $c=13,\qquad \alpha=45^{\circ},\qquad \beta=60^{\circ} $ 1846 | \end{enumerate} 1847 | \item 解三角形: 1848 | \begin{enumerate} 1849 | \item $\alpha=62^{\circ} ,\qquad \beta=48^{\circ} ,\qquad c=24$ 1850 | \item $\alpha=55^{\circ} 12',\qquad \beta=29^{\circ} 18',\qquad c=18$ 1851 | \end{enumerate} 1852 | \end{enumerate} 1853 | \end{ex} 1854 | 1855 | 1856 | \begin{example} 1857 | 已知$b=60$、$c=34$、$\alpha=41^{\circ}$ (图6.31).求$\alpha$, $\beta$, 1858 | $\gamma$. 1859 | \end{example} 1860 | 1861 | \begin{figure}[htp] 1862 | \centering 1863 | \begin{tikzpicture} 1864 | \tkzDefPoints{0/0/B, 4/0/C, 1.5/2/A} 1865 | \tkzDrawPolygon(A,B,C) 1866 | \tkzMarkAngles[mark=none, size=.3](B,A,C) 1867 | \tkzLabelPoints[below](C,B) 1868 | \tkzLabelPoints[above](A) 1869 | \node at (2,0)[below]{$a$}; 1870 | \node at (5.5/2,1)[right]{$b$}; 1871 | \node at (.75,1)[left]{$c$}; 1872 | \end{tikzpicture} 1873 | \caption{} 1874 | \end{figure} 1875 | 1876 | \begin{solution} 1877 | \[\begin{split} 1878 | a^2&=b^2+c^2-2bc\cos\alpha\\ 1879 | &=602+342-2\x60\x34\cos41^{\circ}\\ 1880 | &=3600+1156-4080\x0.755\approx 1676 1881 | \end{split}\] 1882 | 查平方根表,得:$a\approx 41$. 1883 | \[\sin\gamma=\frac{c\sin\alpha}{a}=\frac{34\x\sin41^{\circ}}{41}=\frac{34\x 0.655}{41}\approx 0.5441\] 1884 | 1885 | $\therefore\quad \gamma\approx 32^{\circ}58'$ 1886 | 1887 | \[\beta=180^{\circ}-(\alpha+\gamma) 1888 | =180^{\circ}-(41^{\circ}+32^{\circ}58') 1889 | =106^{\circ}2'\] 1890 | \end{solution} 1891 | 1892 | 1893 | 例6.32告诉我们,如果已知两边及夹角,可由余弦定理先求 1894 | 出未知的一边,然后由正弦定理就可算出未知的一个角了, 1895 | 再由内角和定理,就可求出未知的另一个角了.再求出未知 1896 | 边后,最好先求较短边所对的角,因为较短边所对的角一定 1897 | 是锐角,这样就可避免求钝角的情况. 1898 | 1899 | \begin{ex} 1900 | 已知下列条件,解三角形 1901 | \begin{enumerate} 1902 | \item $a=22,\qquad b=26,\qquad \gamma=78^{\circ}$ 1903 | \item $b=10,\qquad c=10,\qquad \alpha=102^{\circ}$ 1904 | \item $a=0.8,\qquad c=0.6,\qquad \beta=50^{\circ}$ 1905 | \item $a=4,\qquad b=5,\qquad \gamma=60^{\circ}$ 1906 | \end{enumerate} 1907 | \end{ex} 1908 | 1909 | \begin{example} 1910 | 已知$a=134.6$, $b=87.8$, $c=161.7$(图6.32), 1911 | 求$\alpha$、$\beta$、$\gamma$. 1912 | \end{example} 1913 | 1914 | \begin{figure}[htp] 1915 | \centering 1916 | \begin{tikzpicture} 1917 | \tkzDefPoints{0/0/C, 3/0/B, 0/2.5/A} 1918 | \tkzDrawPolygon(A,B,C) 1919 | \tkzLabelPoints[below](B,C) 1920 | \tkzLabelPoints[above](A) 1921 | \node at (1.5,0)[below]{$a$}; 1922 | \node at (0,1.25)[left]{$b$}; 1923 | \node at (1.5,1.25)[right]{$c$}; 1924 | \end{tikzpicture} 1925 | \caption{} 1926 | \end{figure} 1927 | 1928 | \begin{solution} 1929 | \textbf{解法一:} 先用余弦定理求两个较短的边所对的角. 1930 | \[\cos\alpha=\frac{87.8^2+161.7^2-134.6^2}{2\x 87.8\x 161.7}\approx 0.5549\] 1931 | $\therefore\quad \alpha=56^{\circ}18'$ 1932 | 1933 | \[\cos\beta=\frac{134.6^2+161.7^2-87.8^2}{2\x134.6\x161.7}\approx 0.8400\] 1934 | $\therefore\quad \beta=32^{\circ}51',\quad 1935 | \gamma=180^{\circ}-(56^{\circ}18'+32^{\circ}51')=90^{\circ}51'$ 1936 | 1937 | \textbf{解法二:} 1938 | \[\cos\alpha=\frac{87.8^2+161.7^2-134.6^2}{2\x 87.8\x 161.7}\approx 0.5549\] 1939 | $\therefore\quad \alpha=56^{\circ}18'$ 1940 | 1941 | \[\sin\beta=\frac{b}{a}\sin\alpha=\frac{87.8\x \sin 56^{\circ}18'}{134.6}\approx 0.5427\] 1942 | $\therefore\quad \beta=32^{\circ}51', \quad \gamma=180^{\circ}-(56^{\circ}18'+32^{\circ}51')=90^{\circ}51'$ 1943 | \end{solution} 1944 | 1945 | 由例6.33可知,如果已知三边$a$、$b$、$c$, 可用余弦定理求 1946 | 出两个较短的边所对的角的余弦;或由余弦定理求出一角 1947 | 后,再改用正弦定理求另一角的正弦.一般地说后一种方法 1948 | 比较方便.我们所以要先求较短边所对的角,主要还是避免 1949 | 在计算中出现求钝角的情况. 1950 | 1951 | \begin{ex} 1952 | 已知下列条件,解三角形 1953 | \begin{enumerate} 1954 | \item $a=40,\qquad b=19,\qquad c=41$ 1955 | \item $a=1.5,\qquad b=2.5,\qquad c=1.8$ 1956 | \item $a=6,\qquad b=3,\qquad c=3\sqrt{3}$ 1957 | \item $a=\sqrt{10},\qquad b=4,\qquad c=\sqrt{2}$ 1958 | \end{enumerate} 1959 | \end{ex} 1960 | 1961 | \begin{example} 1962 | 在锐角$\triangle ABC$中,已知$a=12$, $b=20$, $\alpha=34^{\circ}$, 1963 | 求:$\beta$、$\gamma$和$c$ (图6.33). 1964 | \end{example} 1965 | 1966 | \begin{figure}[htp] 1967 | \centering 1968 | \begin{tikzpicture} 1969 | \tkzDefPoints{0/0/A, 4/0/B, 3.2/2.5/C, 2.4/0/B'} 1970 | \tkzDrawPolygon(A,B,C) 1971 | \tkzLabelPoints[below](A,B,B') 1972 | \tkzLabelPoints[above](C) 1973 | \tkzDrawSegments[dashed](C,B') 1974 | \node at (1.6,1.25)[left]{$b$}; 1975 | \node at (3.6,1.25)[right]{$a$}; 1976 | \tkzDrawArc[delta=10](C,B')(B) 1977 | \end{tikzpicture} 1978 | \caption{} 1979 | \end{figure} 1980 | 1981 | 1982 | \begin{solution} 1983 | $\because\quad \frac{a}{\sin\alpha}=\frac{b}{\sin\beta}$ 1984 | 1985 | $\therefore\quad \sin\beta=\frac{b\sin\alpha}{a}=\frac{20\x \sin 34^{\circ}}{12}=\frac{20\x 0.5592}{12}=0.9320$ 1986 | 1987 | 由于$\beta$是锐角,反查正弦表得:$\beta=68^{\circ}45'$ 1988 | 1989 | $\therefore\quad \gamma=180^{\circ}-(\alpha+\beta)=180^{\circ}-(34^{\circ}+68^{\circ}45')=77^{\circ}15'$ 1990 | 1991 | 又$\because\quad \frac{c}{\sin\gamma}=\frac{a}{\sin\alpha}$ 1992 | 1993 | $\therefore\quad c=\frac{a\sin\gamma}{\sin\alpha}=\frac{12\x \sin 77^{\circ}15'}{\sin 34^{\circ}}=\frac{12\x 0.9753}{0.5582}=20.93$ 1994 | \end{solution} 1995 | 1996 | 如果在例6.34中,不限$\triangle ABC$是锐角三角形,那么$\beta$角就 1997 | 可能是钝角(图6.33中的$\angle AB'C$),这样$\beta$角就有两个解. 1998 | 对于已知$a$、$b$、$\alpha$解三角形这种情形,我们详细讨论如下: 1999 | \begin{enumerate} 2000 | 2001 | 2002 | \item 如果已知$\alpha$角是钝角或直角,那么必须$a>b$才能有 2003 | 解(为什么?),这时从$\sin\beta=\frac{b\sin\alpha}{a}$ 2004 | 求$\beta$角的时候,只能 2005 | 取锐角的值,因此只有一个解. 2006 | \item 如果已知的$\alpha$角是锐角,并且$a>b$或者$a=b$. 这时 2007 | 从$\sin\beta=\frac{b\sin\alpha}{a}$ 2008 | 求$\beta$角的时候,也只能取锐角的值,因此都 2009 | 只有一个解(图6.34和图6.35). 2010 | 2011 | \begin{figure}[htp]\centering 2012 | \begin{minipage}[t]{0.48\textwidth} 2013 | \centering 2014 | \begin{tikzpicture}[>=latex, scale=1] 2015 | \tkzDefPoints{0/0/A, 3/0/B, 1/2/C} 2016 | \tkzLabelPoints[below](A,B) 2017 | \tkzLabelPoints[above](C) 2018 | \tkzDrawPolygon(A,B,C) 2019 | \node at (1.5,0)[below]{$c$}; 2020 | \node at (2,1)[right]{$a>b$}; 2021 | \node at (.5,1)[left]{$b$}; 2022 | \tkzDefPointBy[rotation= center C angle -100](B) 2023 | \tkzGetPoint{C'} 2024 | \tkzDrawArc[delta=10](C,C')(B) 2025 | \end{tikzpicture} 2026 | \caption{} 2027 | \end{minipage} 2028 | \begin{minipage}[t]{0.48\textwidth} 2029 | \centering 2030 | \begin{tikzpicture}[>=latex, scale=1] 2031 | \tkzDefPoints{0/0/A, 3/0/B, 1.5/2/C} 2032 | \tkzLabelPoints[below](A,B) 2033 | \tkzLabelPoints[above](C) 2034 | \tkzDrawPolygon(A,B,C) 2035 | \node at (1.5,0)[below]{$c$}; 2036 | \node at (.75,1)[left]{$b$}; 2037 | \node at (2.25,1)[right]{$a=b$}; 2038 | \tkzDrawArc[delta=10](C,A)(B) 2039 | \end{tikzpicture} 2040 | \caption{} 2041 | \end{minipage} 2042 | \end{figure} 2043 | 2044 | 2045 | 2046 | \item 如果已知的$\alpha$角是锐角,并且$ab\sin \alpha$, 这时从$\sin\beta=\frac{b\sin\alpha}{a}$求得 $\sin\beta<1$, 2050 | $\beta$可以取一个锐角的值和一个钝角的值,因此可以有两个解 2051 | (图6.36). 2052 | \begin{figure}[htp] 2053 | \centering 2054 | \begin{tikzpicture} 2055 | \tkzDefPoints{0/0/A, 4/0/B, 3/2.5/C, 2/0/B'} 2056 | \tkzLabelPoints[below](A,B,B') 2057 | \tkzLabelPoints[above](C) 2058 | \draw[dashed](C)--node[fill=white]{$b\sin\alpha$}(3,0); 2059 | \tkzDrawPolygon(A,B,C) 2060 | \tkzDrawSegments(B',C) 2061 | \node at (1.5,1.25)[left]{$b$}; 2062 | \node at (3.5,1.25)[right]{$\begin{array}{cc} 2063 | a>b\sin\alpha\\ 2064 | a1$, 但是 2077 | 一角的正弦值不能大于1, 因此没有解(图6.38). 2078 | \end{enumerate} 2079 | \end{enumerate} 2080 | 2081 | \begin{figure}[htp]\centering 2082 | \begin{minipage}[t]{0.48\textwidth} 2083 | \centering 2084 | \begin{tikzpicture}[>=latex, scale=1] 2085 | \tkzDefPoints{0/0/A, 4/0/B, 4/3/C} 2086 | \tkzLabelPoints[below](A,B) 2087 | \tkzLabelPoints[above](C) 2088 | \node at (2,1.5)[left]{$b$}; 2089 | \node at (4,1.5)[right]{$a=b\sin\alpha$}; 2090 | \tkzCompass(C,B) 2091 | \tkzDrawPolygon(A,B,C) 2092 | \end{tikzpicture} 2093 | \caption{} 2094 | \end{minipage} 2095 | \begin{minipage}[t]{0.48\textwidth} 2096 | \centering 2097 | \begin{tikzpicture}[>=latex, scale=1] 2098 | \tkzDefPoints{0/0/A, 4/0/B, 3/2.5/C, 3.75/.5/D} 2099 | \tkzLabelPoints[below](A) 2100 | \tkzLabelPoints[above](C) 2101 | \draw[dashed](C)--node[fill=white]{$b\sin\alpha$}(3,0); 2102 | \node at (3.5,1.5)[right]{$ab$& 一解&一解\\ \hline 2117 | $a=b$&无解&一解\\ 2118 | \hline 2119 | & & $a>b\sin\alpha$时,两解\\ 2120 | $a=latex, scale=1] 2164 | \tkzDefPoints{0/0/A, 4/0/B, 3/2.5/C, 3/0/D} 2165 | \tkzLabelPoints[below](A,B,D) 2166 | \tkzLabelPoints[above](C) 2167 | \tkzDrawPolygon(A,C,B) 2168 | \tkzDrawSegments[dashed](C,D) 2169 | \tkzMarkAngles[mark=none, size=.35](D,A,C C,B,D) 2170 | \tkzLabelAngle[pos=.5](D,A,C){$\alpha$} 2171 | \tkzLabelAngle[pos=.5](C,B,D){$\beta$} 2172 | \tkzMarkRightAngle(B,D,C) 2173 | \end{tikzpicture} 2174 | \caption{} 2175 | \end{minipage} 2176 | \begin{minipage}[t]{0.48\textwidth} 2177 | \centering 2178 | \begin{tikzpicture}[>=latex, scale=1] 2179 | \tkzDefPoints{0/0/D, 4/0/B, 0/2.5/C, 1.3/0/A} 2180 | \tkzLabelPoints[below](A,B,D) 2181 | \tkzLabelPoints[above](C) 2182 | \tkzDrawPolygon(A,C,B) 2183 | \tkzDrawSegments[dashed](C,D A,D) 2184 | 2185 | \end{tikzpicture} 2186 | \caption{} 2187 | \end{minipage} 2188 | \end{figure} 2189 | 2190 | \item 当$\alpha$为钝角时 2191 | (图6.40),仍设$\overline{AB}$边上的高 2192 | $\overline{CD}=h$, 那么: 2193 | \[S_{\triangle}=\frac{1}{2}ch\] 2194 | 在直角$\triangle ACD$中,$h=b\sin(180^{\circ}-\alpha)=b\sin\alpha$ 2195 | 2196 | $\therefore\quad S_{\triangle}=\frac{1}{2}bc\sin\alpha$ 2197 | 2198 | 当$\beta$为钝角时,同理也可证明: 2199 | \[S_{\triangle}=\frac{1}{2}bc\sin\alpha\] 2200 | \item 当$\alpha$为直角时,那么, 2201 | \[S_{\triangle}=\frac{1}{2}bc \] 2202 | 但$\sin\alpha=\sin 90^{\circ}=1$ 2203 | 2204 | $\therefore\quad S_{\triangle}=\frac{1}{2}bc\sin\alpha$仍然成立. 2205 | 2206 | 同理可证: 2207 | \[S_{\triangle}=\frac{1}{2} ac\sin\beta,\qquad 2208 | S_{\triangle}=\frac{1}{2}ab\sin\gamma\] 2209 | \end{enumerate} 2210 | \end{proof} 2211 | 2212 | 2213 | \subsubsection{已知三角形的三边求面积} 2214 | 2215 | 如果已知三角形的三边,则 2216 | \[S_{\triangle}=\sqrt{p(p-a)(p-b)(p-c)} \] 2217 | 其中:$p=\frac{1}{2}(a+b+c)$ 2218 | 2219 | \begin{proof} 2220 | 由余弦定理得: 2221 | \[\cos\alpha=\frac{b^2+c^2-a^2}{2bc}\] 2222 | 因此: 2223 | \[\begin{split} 2224 | \sin\alpha&=\sqrt{1-\cos^2\alpha}=\sqrt{1-\left(\frac{b^2+c^2-a^2}{2bc}\right)^2}\\ 2225 | &=\sqrt{\left(1+\frac{b^2+c^2-a^2}{2bc}\right)\left(1-\frac{b^2+c^2-a^2}{2bc}\right)}\\ 2226 | &=\sqrt{\frac{(b+c)^2-a^2}{2bc}\cdot \frac{a^2-(b-c)^2}{2bc}}\\ 2227 | &=\frac{1}{2bc}\cdot \sqrt{(b+c+a)(b+c-a)(a+b-c)(a-b+c)} 2228 | \end{split}\] 2229 | 2230 | 令$p=\frac{1}{2}(a+b+c)$,则$2p=a+b+c$ 2231 | \[b+c-a=(a+b+c)-2a=2p-2a=2(p-a)\] 2232 | 同理: 2233 | \[a+c-b=2(p-b),\qquad a+b-c=2(p-c)\] 2234 | 因此: 2235 | \[\begin{split} 2236 | \sin\alpha&=\frac{1}{2bc}\sqrt{16p(p-a)(p-b)(p-c)}\\ 2237 | &=\frac{2}{bc}\sqrt{p(p-a)(p-b)(p-c)} 2238 | \end{split}\] 2239 | 2240 | 由于$S_{\triangle}=\frac{1}{2}bc\sin\alpha$,因此: 2241 | \[\begin{split} 2242 | S_{\triangle}&=\frac{1}{2}bc\x \frac{2}{bc}\sqrt{p(p-a)(p-b)(p-c)}\\ 2243 | &=\sqrt{p(p-a)(p-b)(p-c)} 2244 | \end{split}\] 2245 | 2246 | 这个公式,我国叫做\textbf{三斜求积公式},国外又叫做\textbf{海伦公 2247 | 式}. 2248 | \end{proof} 2249 | 2250 | \subsubsection{已知三角形二角一夹边,求三角形的面积} 2251 | 2252 | 如果已知三角形的二角一夹边,则 2253 | \[\begin{split} 2254 | S_{\triangle}&=\frac{a^{2} \sin \beta \cdot \sin \gamma}{2 \sin (\beta+\gamma)} \\ 2255 | S_{\triangle}&=\frac{b^{2} \sin \alpha \cdot \sin \gamma}{2 \sin (\alpha+\gamma)} \\ 2256 | S_{\triangle}&=\frac{c^{2} \sin \alpha \cdot \sin \beta}{2 \sin (\alpha+\beta)} 2257 | \end{split}\] 2258 | 2259 | \begin{proof} 2260 | 已知$S_{\triangle}=\frac{1}{2}bc\sin\alpha$,由正弦正理, 2261 | \[b=\frac{a\sin \beta}{\sin\alpha},\qquad c=\frac{a\sin\gamma}{\sin\alpha}\] 2262 | 2263 | 因此: 2264 | \[S_{\triangle}=\frac{1}{2}\left(\frac{a\sin \beta}{\sin\alpha}\right)\cdot\left(\frac{a\sin\gamma}{\sin\alpha}\right)\cdot \sin\alpha=\frac{a^2\sin\beta\cdot \sin\gamma}{2\sin\alpha}\] 2265 | 但$\sin\alpha=\sin[180^{\circ}-(\beta+\gamma)]=\sin(\beta+\gamma)$ 2266 | 2267 | $\therefore\quad S_{\triangle}=\frac{a^{2} \sin \beta \cdot \sin \gamma}{2 \sin (\beta+\gamma)}$ 2268 | 2269 | 同理可证明其它两个公式. 2270 | \end{proof} 2271 | 2272 | 2273 | \begin{ex} 2274 | \begin{enumerate} 2275 | \item 在三角形中,已知 2276 | \begin{enumerate} 2277 | \item 两边长分别为4、7, 夹角是$40^{\circ}$, 2278 | \item 三边长各为2、4、4. 2279 | \item $\alpha=60^{\circ}$、$\beta=80^{\circ}$、$c=12$, 2280 | \end{enumerate} 2281 | 求它们的面积各等于多少? 2282 | \item 求边长等于$a$的等边三角形的面积. 2283 | 2284 | \item $r$为$\triangle ABC$的外接圆的半径,求证: 2285 | \[S_{\triangle}=2r \sin\alpha\cdot\sin\beta\cdot \sin\gamma\] 2286 | 2287 | \item 已知三角形的三边$a$、$b$、$c$, 且$p=\frac{1}{2}(a+b+c)$, 2288 | 求证:内切圆半径 2289 | \[r=\sqrt{\frac{(p-a)(p-b)(p-c)}{p}}\] 2290 | 2291 | \item 设$h_a$、$h_b$、$h_c$分别表示$\triangle ABC$的三边$\overline{BC}$、$\overline{CA}$、$\overline{AB}$ 2292 | 上的高,求证: 2293 | \[\begin{split} 2294 | h_a&=\frac{2}{a}\sqrt{p(p-a)(p-b)(p-c)}\\ 2295 | h_b&=\frac{2}{b}\sqrt{p(p-a)(p-b)(p-c)}\\ 2296 | h_c&=\frac{2}{c}\sqrt{p(p-a)(p-b)(p-c)}\\ 2297 | \end{split}\] 2298 | \end{enumerate} 2299 | \end{ex} 2300 | 2301 | \subsection{解三角形在测量中的应用} 2302 | 2303 | 利用解三角形的原理,可 2304 | 以解许多实际测量问题.下面 2305 | 我们来研究如何计算不便测量 2306 | 的两点间的距离. 2307 | 2308 | 测量工作者,为了测量远 2309 | 方某个目标$C$的距离(图6.41), 2310 | 总先选定适当长的基线 2311 | $\overline{AB}$, 然后从基线两端$A$、$B$, 分别测 2312 | 得目标$C$的方向与$AB$的夹角为$\alpha$、$\beta$. 那么,根据基线$\overline{AB}$的 2313 | 长度和$\alpha$、$\beta$的值就可算出$C$点与$A$点、$B$点的距离.以及$C$点 2314 | 与基线$\overline{AB}$的距离,计算方法如下: 2315 | 2316 | $\because\quad \frac{\overline{AC}}{\sin\beta}=\frac{\overline{AB}}{\sin(\alpha+\beta)},\qquad \frac{\overline{BC}}{\sin\alpha}=\frac{\overline{AB}}{\sin(\alpha+\beta)}$ 2317 | 2318 | $\therefore\quad \overline{AC}=\frac{\overline{AB}\sin\beta}{\sin(\alpha+\beta)},\qquad \overline{BC}=\frac{\overline{AB}\sin\alpha}{\sin(\alpha+\beta)}$ 2319 | 2320 | 设$\overline{CD}$是$\overline{AB}$边上的高且$\overline{CD}=x$, 则$\overline{AD}=x\cot\alpha$, $\overline{BD}=x\cot\beta$ 2321 | \[\overline{AB}=\overline{AD} +\overline{BD} =x(\cot\alpha +\cot\beta)\] 2322 | 2323 | $\therefore\quad x=\frac{\overline{AB}}{\cot\alpha +\cot\beta}$ 2324 | 2325 | 2326 | \begin{figure}[htp]\centering 2327 | \begin{minipage}[t]{0.4\textwidth} 2328 | \centering 2329 | \begin{tikzpicture}[>=latex, scale=1] 2330 | \tkzDefPoints{0/0/A, 4/0/B, 2.5/2.5/C, 2.5/0/D} 2331 | \tkzLabelPoints[below](A,B,D) 2332 | \tkzLabelPoints[above](C) 2333 | \tkzDrawPolygon(A,C,B) 2334 | \tkzDrawSegments(C,D) 2335 | \tkzMarkAngles[mark=none, size=.35](D,A,C C,B,D) 2336 | \tkzLabelAngle[pos=.5](D,A,C){$\alpha$} 2337 | \tkzLabelAngle[pos=.5](C,B,D){$\beta$} 2338 | \tkzMarkRightAngle(B,D,C) 2339 | \node at (1.25,1.25)[left]{$b$}; 2340 | \node at (3.25,1.25)[right]{$a$}; 2341 | \end{tikzpicture} 2342 | \caption{} 2343 | \end{minipage} 2344 | \begin{minipage}[t]{0.55\textwidth} 2345 | \centering 2346 | \begin{tikzpicture}[>=latex, scale=1] 2347 | \draw(0,0) circle (1.5); 2348 | \tkzDefPoint(30:1.5){A} 2349 | \tkzDefPoint(-30:1.5){B} 2350 | \tkzDefPoints{0/0/O, 5/0/C} 2351 | \tkzDrawPolygon(A,B,C) 2352 | \tkzDrawSegments[dashed](A,O B,O) 2353 | \tkzMarkAngles[mark=none, size=.3](B,A,C C,B,A) 2354 | \tkzLabelAngle[pos=.6](B,A,C){$\alpha$} 2355 | \tkzLabelAngle[pos=.6](C,B,A){$\beta$} 2356 | \tkzLabelPoints[above](A) 2357 | \tkzLabelPoints[below](B) 2358 | \end{tikzpicture} 2359 | \caption{} 2360 | \end{minipage} 2361 | \end{figure} 2362 | 2363 | 2364 | 2365 | 在测量中,基线越长,量得越准确,测的精确度就越 2366 | 高. 2367 | 2368 | 天文学家曾用了这个方法测出了地球和月亮间的距离. 2369 | 1671年两个法国天文学家拉让 2370 | 德和拉卡伊,一个在柏林,一 2371 | 个在好望角(这两个城市差不 2372 | 多位于同一子午线上),测出了 2373 | $\alpha$、$\beta$的大小和$AB$的长度(图 2374 | 6.42).从而确切地算出了地 2375 | 月平均距离为385400公里. 2376 | 2377 | 在地球上,我们可使用的最长基线是地球的直径,由 2378 | 于地球绕太阳按椭圆形轨道运行,当我们要测量其它星球到 2379 | 地球的距离时,可使用的最长基线是地球椭圆轨道的长轴 2380 | (图6.43). 2381 | 2382 | \begin{figure}[htp] 2383 | \centering 2384 | \includegraphics[scale=.6]{fig/6-43.png} 2385 | \caption{} 2386 | \end{figure} 2387 | 2388 | 从以上的分析我们知道,当我们应用解三角形原理在地 2389 | 球上测量地球与其它星球的距离时,可使用的基线是有限 2390 | 的,所以星球离地球越远、视角也越小,相对基线就越短, 2391 | 测的结果也就越不精确了. 2392 | 2393 | \begin{ex} 2394 | \begin{enumerate} 2395 | \item 要测量一条河两岸的两点$A$、$B$之间的距离,在$A$点所在 2396 | 的岸边选择一条基线$\overline{AC}=380$米,测出 2397 | $\angle BAC=75^{\circ}32'$, $\angle BCA=45^{\circ}35'$, 2398 | 求$A$、$B$间的距离. 2399 | \item 如图,$A$、$B$两地不能直接测量,选同时能看到$A$、$B$的一点 2400 | $C$, 测得$\overline{AC}=280$米,$\overline{BC}=470$米,$\angle ACB=80^{\circ}21'$, 2401 | 求$A$、$B$两地的距离. 2402 | \item 河对岸有两个目标$A$、$B$, 若 2403 | 不准过河,如何测量才能 2404 | 算出$A$、$B$两点间的距离? 2405 | \end{enumerate} 2406 | \end{ex} 2407 | 2408 | \begin{figure}[htp]\centering 2409 | \begin{minipage}[t]{0.48\textwidth} 2410 | \centering 2411 | \includegraphics[scale=.7]{fig/6-ti1.PNG} 2412 | \caption*{第1题} 2413 | \end{minipage} 2414 | \begin{minipage}[t]{0.48\textwidth} 2415 | \centering 2416 | \includegraphics[scale=.7]{fig/6-ti2.PNG} 2417 | \caption*{第2题} 2418 | \end{minipage} 2419 | \end{figure} 2420 | 2421 | \begin{figure}[htp] 2422 | \centering 2423 | \includegraphics[scale=.7]{fig/6-ti3.PNG} 2424 | \caption*{第3题} 2425 | \end{figure} 2426 | 2427 | \section*{习题6.3} 2428 | \addcontentsline{toc}{subsection}{习题6.3} 2429 | \begin{enumerate} 2430 | \item 在$\triangle ABC$中, 2431 | \begin{enumerate} 2432 | \item 已知$a=8$、$\beta=40^{\circ}$、$\gamma=90^{\circ}$, 求$b$. 2433 | \item 已知$a=25$、$b=31$、$\gamma=90^{\circ}$, 求$a$、$c$. 2434 | \item 已知$a=6$、$\beta=40^{\circ}$、$\gamma=80^{\circ}$, 求b. 2435 | \item 已知$a=8$、$b=6$、$\alpha=75^{\circ}$, 求$\beta$、$c$. 2436 | \item 已知$a=3$、$b=4$、$c=5$, 求$\gamma$. 2437 | \end{enumerate} 2438 | \item 已知下列条件,求$S_{\triangle}$. 2439 | \begin{enumerate} 2440 | \item $a=6,\qquad c=12,\qquad \beta =135^{\circ}$ 2441 | \item $a=10,\qquad \beta =45^{\circ},\qquad \gamma=60^{\circ}$ 2442 | \item $a=3,\qquad b=5,\qquad c=10$ 2443 | \end{enumerate} 2444 | 2445 | \item 在$\triangle ABC$中,$a=10$cm, $\beta =54^{\circ}16'$, $\alpha=63^{\circ}6'$, 求$\triangle ABC$的面积和$\angle C$的平分线长. 2446 | \item 设$D$把$\triangle ABC$的边$\overline{BC}$分成$m:n$, 求证: 2447 | \[n\overline{AB}^2+m \overline{AC}^2=(m+n)\overline{AD}^2+\overline{BD}^2+m\overline{CD}^2\] 2448 | \item 已知$\triangle ABC$, 且 2449 | $\frac{a+b}{6}=\frac{b+c}{4}=\frac{c+a}{5}$, 2450 | 求证: 2451 | \begin{enumerate} 2452 | \item $\frac{\sin\alpha}{7}=\frac{\sin\beta}{5}=\frac{\sin\gamma}{3}$ 2453 | \item $\frac{\cos\alpha}{-7}=\frac{\cos\beta}{11}=\frac{\cos\gamma}{13}$ 2454 | \item $\alpha=120^{\circ}$ 2455 | \end{enumerate} 2456 | \item 已知在$\triangle ABC$中,$\gamma=60^{\circ}$, 求证:$a^2+b^2=c^2+ab$. 2457 | \item 求证在$\triangle ABC$中, 2458 | $a^2-b^2=ac\cos\beta -bc\cos\alpha$ 2459 | \item 已知三角形三边的比是$3:5:7$, 求最大角. 2460 | \item 如果$\triangle ABC$的边角之间满足下列关系,那么这个三角形 2461 | 是什么三角形, 2462 | \begin{enumerate} 2463 | \item $\sin^2\alpha=\sin^2\beta +\sin^\gamma$ 2464 | \item $a\cos A=b\cos B$ 2465 | \end{enumerate} 2466 | 2467 | \item 等腰梯形$ABCD$的上底$\overline{AD}=18$cm, 下底$\overline{BC}=22$cm, 2468 | $\angle ABC=60^{\circ}$, 求它的面积. 2469 | \item 已知梯形$ABCD$, $AD\parallel BC$. $\overline{AB}=13$, $\overline{BC}=18$, 2470 | $\overline{CD}=15$, $\overline{DA}=4$, 求它的面积. 2471 | \item 如果已知四边形$ABCD$的$\angle A=90^{\circ}$, $\overline{AB}=32$, $\overline{BC}= 2472 | 27$, $\overline{CD}=35$, $\overline{DA}=24$, 那么它的面积是多少? 2473 | \item 已知一四边形的两条对角线的长分别为$x$、$y$, 夹角为$\theta$, 2474 | 面积为$S$, 求证: 2475 | \[S=\frac{1}{2}xy\sin\theta\] 2476 | \end{enumerate} 2477 | 2478 | \section*{复习题六} 2479 | \addcontentsline{toc}{section}{复习题六} 2480 | \begin{enumerate} 2481 | \item 用作图法求锐角$x$. 2482 | \begin{multicols}{2} 2483 | \begin{enumerate} 2484 | \item $\sin x=\frac{4}{5}$ 2485 | \item $\cos x=\frac{1}{3}$ 2486 | \item $\tan x=\sqrt{3}+\sqrt{2}$ 2487 | \item $\sin x=2\cos x$ 2488 | \end{enumerate} 2489 | \end{multicols} 2490 | 2491 | \item 已知:$\tan\alpha=m,\; m\ge 0$, 求$\cos\alpha$、$\sin\alpha$. 2492 | \item 已知 2493 | $ \sin\alpha+\cos\alpha=1.2$, 求$\sin\alpha$、$\cos\alpha$的值. 2494 | \item 计算: 2495 | \begin{enumerate} 2496 | \item $3\sin90^{\circ}+2\cos0^{\circ}-3\sin45^{\circ}$ 2497 | \item $\sin^2 30^{\circ}-\frac{1}{2}\cos 90^{\circ}+\cos^2 30^{\circ}$ 2498 | \item $\sin^2 45^{\circ}+\tan^2 30^{\circ}$ 2499 | \end{enumerate} 2500 | 2501 | \item 已知$\triangle ABC$, $\angle C=90^{\circ}$, 求证 2502 | \[ \tan A+\tan B=\frac{c^2}{ab}\] 2503 | \item 已知在$\triangle ABC$中,$\angle C=90^{\circ}$, $\overline{CE}\overline{CB}$, 延长$CA$到$D$ 2504 | 使$\overline{AD}=\overline{AB}$, 作$\overline{DB}$, 求$\angle D$的度数,并根据图形求 2505 | $\angle D$的正弦、余弦、正切和余切的值. 2506 | 2507 | \item 在正方形$ABCD$中,已知$E$是$\overline{BC}$的中点,求$\angle AEC$的正 2508 | 弦和余弦. 2509 | \item 在四边形$ABCD$中,$\overline{AB}=2a$, $BC=(\sqrt{3}+1)a$, 2510 | $\overline{CD}=\sqrt{2}a$, $\angle B=60^{\circ}$, $\angle C=75^{\circ}$, 求$\overline{AC}$的长和四边 2511 | 形$ABCD$的面积. 2512 | \item 已知在$\triangle ABC$中,$a=4$、$b=5$、$c=6$, 求$\cos\alpha$、$\sin\gamma$、 2513 | $\sin\beta$、$\sin\alpha$, 问$\alpha$、$\beta$、$\gamma$是锐角还是钝角? 2514 | \item 一个三角形的三边的长分别为3尺,4尺及$\sqrt{37}$尺,求 2515 | 此三角形的最大角的度数. 2516 | \item 在$\triangle ABC$中,求证: 2517 | \[\frac{a}{\sin\alpha}=\frac{b+c}{\sin\beta+\sin\gamma}=\frac{b-c}{\sin\beta-\sin\gamma}\] 2518 | \item 设$P$是等边三角形$ABC$外接圆的$\wideparen{BC}$上的一点,求证 2519 | \[\overline{PA}^2=\overline{AB}^2+\overline{PB}\cdot \overline{PC}\] 2520 | (提示:求$\cos\angle ABP$, $\cos\angle ACP$,利用$\cos\angle ABP=-\cos\angle ACP$化简即得). 2521 | \item 在$\triangle ABC$中,已知$\gamma=60^{\circ}$, $\overline{AC}=4$, 面积为$\sqrt{3}$, 求 2522 | $\overline{AB}$及$\overline{BC}$的长. 2523 | \item 如图,为求得河对岸某建 2524 | 筑物的高$\overline{AB}$, 在地面上 2525 | 引一条基线$\overline{CD}=a$, 测得 2526 | $\angle ACB=\alpha$, $\angle BCD=\beta$, 2527 | $\angle BDC=\gamma$, 求$\overline{AB}$. 2528 | \item 已知$\odot (A,\sqrt{3})$、 2529 | $\odot (B,2-\sqrt{3})$、$\odot (c,1)$. 2530 | $\odot A$分别与$\odot B$和$\odot C$相外 2531 | 切.$\angle BAC=60^{\circ}$, 求$\overline{BC}$ 2532 | 的长和$\angle ACB$的度数. 2533 | 2534 | \begin{figure}[htp] 2535 | \centering 2536 | \includegraphics[scale=.7]{fig/6-14ti.png} 2537 | \caption*{第14题} 2538 | \end{figure} 2539 | 2540 | 2541 | \item 外国船只除特许者外,不得进入离我海岸$d$海里以内的 2542 | 区域,设$A$和$B$是我们的两个观测站,$A$与$B$之间的距离为 2543 | $S$海里,海岸线是过$A$点及$B$点的直线,一外国船在$P$点, 2544 | 在$A$站测得$\angle BAP=\alpha$, 同时在$B$站测得$\angle ABP=\beta$, 问$\alpha$ 2545 | 及$\beta$满足什么简单的三角比的不等式,就应当向此未经 2546 | 特许的外国船发出警告,命令退出我海域. 2547 | \item 已知$I$是$\triangle ABC$的内心,$I_a$、$I_b$、$I_c$为$\triangle ABC$的三个旁 2548 | 心,$r,r',r_a,r_b$分别为$\triangle ABC$的外接圆、内切圆和三个 2549 | 旁切圆的半径;设$p=\frac{1}{2}(a+b+c)$, $S_{\triangle}$为$\triangle ABC$的面积;求证: 2550 | \begin{enumerate} 2551 | \item $S_{\triangle}=r'p=r_a(p-a)=r_b(p-b)=r_c(p-c)$ 2552 | \item $4rS_{\triangle}=abc$ 2553 | \item $4r=r_a+r_b+r_c-r'$ 2554 | \end{enumerate} 2555 | 2556 | \begin{figure}[htp] 2557 | \centering 2558 | \begin{tikzpicture} 2559 | \tkzDefPoints{0/0/B, 3/0/C, 2.4/2/A} 2560 | \tkzDrawPolygon[very thick](A,B,C) 2561 | \tkzDefTriangleCenter[in](A,B,C) \tkzGetPoint{I} 2562 | \tkzDefTriangleCenter[ex](A,B,C) \tkzGetPoint{I_b} 2563 | \tkzDefTriangleCenter[ex](B,C,A) \tkzGetPoint{I_c} 2564 | \tkzDefTriangleCenter[ex](C,A,B) \tkzGetPoint{I_a} 2565 | \tkzDrawLines(I_a,I_b I_a,I_c I_b,I_c) 2566 | \tkzDrawLines[add = .5 and .5](A,B A,C B,C) 2567 | \tkzDrawLines[add = 0 and .1](A,I_a B,I_b C,I_c) 2568 | \tkzLabelPoints[above left](B) 2569 | \tkzLabelPoints[below right](C,I_b,I_a) 2570 | \tkzLabelPoints[above](A, I_c,I) 2571 | \tkzInterLL(B,I_b)(A,C) \tkzGetPoint{X} 2572 | \tkzDrawCircle(I_b,X) 2573 | \end{tikzpicture} 2574 | \caption*{第17题} 2575 | \end{figure} 2576 | 2577 | 2578 | \end{enumerate} --------------------------------------------------------------------------------